You are on page 1of 124

Given below are three circles with radii 1, 2 and 3 ft.

They are tangent to one


another. Let $\angle$B equal to $\theta$. Then the area of the shaded region will
be $$\text{Area}\ = \dfrac{1}{2} r^2 \theta$$ where $r$ is the radius of smallest
circle and is equal to $1$ feet.\\
\\
\\
Now, for the triangle $ABC$, we have $$AB = 4\ \text{ft},\ BC = 3\ \text{ft},\ CA =
5\ \text{ft}$$
For the above segments, we have \begin{align*}
AB^2 & = 16\ .\ .\ .\ .\ .\ .\ (1)\\
BC^2 & = 9\ .\ .\ .\ .\ .\ .\ .\ (2)\\
CA^2 & = 25\ .\ .\ .\ .\ .\ .\ (3)
\end{align*}
Upon adding (1) and (2), we get $$AB^2 + BC^2 = 16 + 9 = 25 = CA^2$$\\
We see that the sides follow Pythagoras theorem and so $\Delta$ABC is a right angle
triangle with $$\angle ABC = \angle B = \theta = 90^o = \dfrac{\pi}{2}\
\text{radian}$$\\
Therefore, the area of the sector is \begin{align*}
\text{Area} & = \dfrac{1}{2} (1)^2 \bigg(\dfrac{\pi}{2}\bigg)\\
& = \dfrac{\pi}{4}\\
& \approx 0.7854\ \text{radian}
\end{align*}

For the first figure, the triangle with $\angle$P = $\theta_1$, the length of all
sides are equal to 1 which means that the triangle is an equilateral triangle.
Therefore, the measure of angle $\theta_1$ will be equal to $$\theta_1 = 60^o\
\text{or}\ \dfrac{\pi}{3} \approx 1.0472\ \text{radian}$$\\
\\
We know that for a circle of radius $r$, when the length $s$ of an arc subtends a
central angle of $\theta$ radians, then $$\theta = \dfrac{s}{r}$$
Therefore, the measure of angle $\theta_2$ will be equal to
$$\theta_2 = \dfrac{1}{1} = 1\ \text{radian}$$
And so, \begin{align*}
\theta_1 - \theta_2 & = 1.0472 - 1\\
& = 0.0472\ \text{radian}
\end{align*}\\

Given that a car's wheels are 28 in. in diameter. Let $x$ be the distance the car
travels when its wheels revolve 10,000 times without slipping as shown in the below
figure.\\
\\
Now, in one revolution, the wheels of car will cover distance equal its
circumference, i.e. $$\text{Circumference} = \pi \cdot d = 28 \pi\ \text{in.}$$
Therefore, in 10,000 revolutions, the car will cover distance equal to
\begin{align*}
x & = 10,000 \times 28 \pi\\
& \approx 879646\ \text{in}\\
& \approx 879646\ \times 1.5783\ \times 10^{-5}\ \ \ \ \ \ \ \ \ \ [\ \text{in
miles}\ ]\\
& \approx 13.88\ \text{mi.}
\end{align*}
Given that Memphis has a latitude of $35^o$N and New Orleans has a latitude of
$30^o$N. Let $s$ be the distance between these two cities. Now, we know that when
the length $s$ of an arc subtends a central angle of $\theta$ radians, then $
$\theta = \dfrac{s}{r}$$
$r$ is the radius of the circle and equal to 3960 miles. The difference in
latitudes will give the central angle which is $$\theta = 35^o - 30^o = 5^o$$\\
\\
Let's multiply it by $\dfrac{\pi}{180^o}$ to convert it into radians. $$\theta =
5^o \times \dfrac{\pi}{180^o} = \dfrac{\pi}{36}\ \text{radians}$$\\
Therefore, the distance $s$ between the two cities will be \begin{align*}
s & = r \times \theta\\
& = 3960 \times \dfrac{\pi}{36}\\
& \approx 345.6\ \text{miles}
\end{align*}

From the figure, we see that the sun rays are parallel, so by alternate angles
theorem, the central angle $\theta$ will also be equal to $$\theta =7.2^o$$ or
$$7.2^o \times \dfrac{\pi}{180^o} = 0.1256637\ \text{radians}$$\\
\\
Now, we know that when the length $s$ of an arc subtends a central angle of
$\theta$ radians, then $$\theta = \dfrac{s}{r}$$\\
\\
Given that the distance $s$ between Syene and Alexandria is $500$ miles, then the
radius $r$ of the earth is
\begin{align*}
r & = \dfrac{s}{\theta}\\
& = \dfrac{500}{0.1256637}\\
& \approx 3979\ \text{miles}\\
\end{align*}
And, the circumference of the earth is
\begin{align*}
\text{Circumference} & = 2 \pi r\\
& = 2 \pi (3979)\\
& \approx 25000\ \text{miles}
\end{align*}

Given that the sprinkler pipe is $300$ ft long and pivots around a central point
through $280^o$ only or $$280^o \times \dfrac{\pi}{180^o} \approx 4.887\
\text{radians}$$\\
Area of a sector with central angle of $\theta$ radians is
$$\text{Area} = \dfrac{1}{2} r^2 \theta$$\\
Therefore, area irrigated by the given system is equal to \begin{align*}
\text{Area} & = \dfrac{1}{2} (300)^2 (4.887)\\
& \approx 219,915\ \text{square feet}
\end{align*}

Given that the top and bottom ends of a wind-shield wiper blade are 34 in. and 14
in., respectively.\\
\\
Since the wiper sweeps through $135^o$ while in operation, then the central angle
of the sector in radians is equal to $$\theta = 135^o \times \dfrac{\pi}{180^o} =
2.3562\ \text{radians}$$\\
\\
Area of a sector with central angle of $\theta$ radians is
$$\text{Area} = \dfrac{1}{2} r^2 \theta$$\\
From the figure, the area swept by the blade will be the difference between the
sectors of radii 34 in. and 14 in., i.e. $$\text{Area} = \dfrac{1}{2} (r'^2 -
r^2) \theta$$ where $r'$ and $r$ are the larger and smaller radii respectively.\\
\\
\\
Therefore, the area swept by the blade is equal to \begin{align*}
\text{Area} & = \dfrac{1}{2} (r'^2 - r^2) \theta\\
& = \dfrac{1}{2}(34^2 - 14^2) (2.3562)\\
& = (34^2 - 14^2) (1.1781)\\
& = 960 (1.1781)\\
& \approx 1131\ \text{square inches}
\end{align*}

Area that the cow can graze is divide into four different sectors as shown in the
figure below.\\
\\
All the circle have central angle equal to $\dfrac{\pi}{2}$ with radii as
\begin{align*}
r_1 & = 100−50-20 = 30\ \text{ft}\\
r_2 & = 100-50= 50\ \text{ft}\\
r_3 & = 100\ \text{ft}\\
r_4 & = 100−60 = 40\ \text{ft}
\end{align*}\\
Area of a sector with central angle of $\theta$ radians is
$$\text{Area} = \dfrac{1}{2} r^2 \theta$$\\
Therefore, the total area that the cow can graze is the sum of the areas of four
sectors, i.e. \begin{align*}
\text{Area}\ & = \dfrac{1}{2} \bigg(\dfrac{\pi}{2}\bigg) (30^2+50^2+100^2+40^2)\\
& = \bigg(\dfrac{\pi}{4}\bigg) (15000)\\
& \approx 11,781\ \text{square feet}
\end{align*}

Given that a winch of radius 2 ft which is used to lift heavy loads makes 8
revolutions every 5 seconds. \\
\\
Let $s$ be the distance any point on the rope travels in time $t$. Let $\theta$ be
the angle the ray from the center of the winch makes with the point on the rope at
time $t$. Then, $$s = r \theta$$
Then, one revolution of winch is equal to 2$\pi$ radians and 8 revolutions of winch
will be equal to $$8 \times 2 \pi = 16 \pi\ \text{radians}$$\\
Then, the angular speed of the point on the rope after 15 second will be
\begin{align*}
\omega & = \dfrac{\theta}{t}\\
& = \dfrac{16 \pi}{15}\\
& \approx 3.351\ \text{rad/s}\\
\end{align*}
The linear speed of the point on the rope, which will be equal to the speed at
which the load is rising is $$v = \dfrac{s}{t} = \dfrac{r \theta}{t} = rw$$\\
Therefore, the speed at which the load is rising is equal to
\begin{align*}
v & = r \omega\\
& = 2 (3.351)\\
& \approx 6.7\ \text{ft/s}
\end{align*}

Given radius of wheels of a car is 11 in and rotating at 600 rpm.\\


\\
The angular speed of the wheels is $$2 \pi \cdot 600 = 1200 \pi\ \text{rad/min}$$\\
Since the wheels have radius 11 in, the linear speed is \begin{align*}
v & = r \omega\\
& = (11)(1200 \pi)\\
& = 13,200 \pi\\
& \approx 41,469\ \text{in./min}
\end{align*}\\
\\
Since there are 12 inches per foot, 5280 feet per mile, and 60 minutes per hour,
the speed $v$ of car in miles per hour is \begin{align*}
v & = \dfrac{41,469\ \text{in./min} \times 60\ \text{min/h}}{12\ \text{in./ft}
\times 5280\ \text{ft/mi}}\\
& = \dfrac{2,488,140\ \text{in./h}}{63,360\ \text{in./mi}}\\
& \approx 39.27\ \text{mi/hr}
\end{align*}

Given that the radius of earth is 3960 miles and rotates about its axis once every
23 h 56 min 4 s.\\
\\
Since there are 60 min per hour and 3600 seconds per hour, the time taken for earth
to rotate about its axis once is $$23 + \dfrac{56}{60} + \dfrac{4}{3600} \approx
23.9344\ \text{h}$$
Therefore, the earth rotates at a speed of $1/23.9344$ rph.\\
\\
\\
The angular speed of a point on the equator is $$\omega = 2 \pi \bigg(\dfrac{1}
{23.9344}\bigg) = \dfrac{2\pi}{23.9344}\ \text{rad/h}$$\\
\\
Since, radius of earth is 3960 miles, linear speed of a point on the equator is
\begin{align*}
v & = r \omega\\
& = (3960)\bigg(\dfrac{2\pi}{23.9344}\bigg)\\
& = 330.9 \pi\\
& \approx 1039.55\ \text{mi/min}
\end{align*}\\

The given triangle from the exercise is labelled and shown in the figure below.
Upon using a ruler, the sides of the triangle measure \begin{align*}
AB & = 3.3\ \text{cm}\\
BC & = 1.9\ \text{cm}\\
CA & = 3.8\ \text{cm}\\
\end{align*}
Then, the trigonometric ratios of $\theta$ are \begin{align*}
\sin \theta & = \dfrac{BC}{CA} = \dfrac{1.9}{3.8} = 0.5\\
\cos \theta & = \dfrac{AB}{CA} = \dfrac{3.3}{3.8} = 0.87\\
\tan \theta & = \dfrac{BC}{AB} = \dfrac{1.9}{3.3} = 0.58\\
\csc \theta & = \dfrac{CA}{BC} = \dfrac{3.8}{1.9} = 2\\
\sec \theta & = \dfrac{CA}{AB} = \dfrac{3.8}{3.3} = 1.15\\
\cot \theta & = \dfrac{AB}{BC} = \dfrac{3.3}{1.9} = 1.7\\
\end{align*}

A right triangle PQR is sketched using a protractor that has the acute angle $40^o$
as shown below.\\
\\
Upon measuring the sides, we got \begin{align*}
PQ & = 2.9\ \text{cm}\\
QR & = 2.5\ \text{cm}\\
PR & = 3.8\ \text{cm}\\
\end{align*}
Then, the trigonometric ratios of $\theta$ are \begin{align*}
\sin 40^o & = \dfrac{QR}{PR} = \dfrac{2.5}{3.8} = 0.66\\
\cos 40^o & = \dfrac{PQ}{PR} = \dfrac{2.9}{3.8} = 0.76\\
\tan 40^o & = \dfrac{QR}{PQ} = \dfrac{2.5}{2.9} = 0.86\\
\csc 40^o & = \dfrac{PR}{QR} = \dfrac{3.8}{2.5} = 1.52\\
\sec 40^o & = \dfrac{PR}{PQ} = \dfrac{3.8}{2.9} = 1.31\\
\cot 40^o & = \dfrac{PQ}{QR} = \dfrac{2.9}{2.5} = 1.16\\
\end{align*}

In the given exercise, the circle has radius of one unit.\\


\\
Now, for the inner triangle, sides $a$ and $d$ are opposite and adjacent to the
angle $\theta$ respectively. So, we have $$\sin \theta = \dfrac{a}{1}\
\Rightarrow \boxed{a = \sin \theta}$$
$$\cos \theta = \dfrac{d}{1}\ \Rightarrow \boxed{d = \cos \theta}$$\\
\\
Now, for the outer triangle, sides $b$ and $c$ are opposite and hypotenuse to the
angle $\theta$ respectively. So, we have $$\tan \theta = \dfrac{b}{1}\
\Rightarrow \boxed{b = \tan \theta}$$
$$\sec \theta = \dfrac{c}{1}\ \Rightarrow \boxed{c = \sec \theta}$$

(a)\ Given that a laser beam is to directed toward the center of the moon, but the
beam strays $0.5^o$ from the intended path as shown in the below figure. Let $x$ be
the distance the beam has diverged from its assigned target when it reaches the
moon. The distance from the earth to the moon is $240,000$ mi.\\
\\
In the triangle, \begin{align*}
\tan 0.5^o & = \dfrac{x}{240,000}\\
x & = 240,000 \times 0.00872686779\\
& \approx 2094.45\ \text{miles}
\end{align*}\\
Therefore, the beam has diverged 2094.45 miles approximately from its assigned
target when it reaches the moon.\\
\\
\\
(b)\ Since $x$ is greater than the radius of the moon which is about 1000 miles,
the beam will not strike the moon.
Given that from the top of 200-ft lighthouse, the angle of depression to a ship in
the ocean is $23^o$ as shown in the below figure. Let $s$ be the distance from the
ship to the base of the lighthouse.\\
\\
From alternate interior angles theorem, angle of elevation from the ship to the top
of lighthouse will also be equal to $23^o$. Then, using trigonometric ratios, we
have \begin{align*}
\tan 23^o & = \dfrac{200}{s}\\
x & = \dfrac{200}{\tan 23^o}\\
& = \dfrac{200}{0.42447481621}\\
& \approx 471.17\ \text{ft}
\end{align*}\\
Therefore, the ship is nearly 471 feet away from the base of the lighthouse.

Given that a 20-ft ladder leans against a building so that the angle between the
ground and the ladder is $72^o$. Let $h$ be the height the ladder reach on the
building as shown in the figure below.\\
\\
Then, from the trigonometric ratios, \begin{align*}
\sin 72^o & = \dfrac{h}{20}\\
h & = 20 (\sin 72^o)\\
& = 20 (0.951)\\
& \approx 19\ \text{ft}
\end{align*}\\
Therefore, the ladder reaches to a height of 19 feet on the building.

Given that a 600-ft guy wire is attached to the top of a communication tower so
that the wire makes an angle of $65^o$ with the ground. Let $h$ be the height of
the communication tower as shown in the figure below.\\
\\
Then, from the trigonometric ratios, \begin{align*}
\sin 65^o & = \dfrac{h}{600}\\
h & = 600 (\sin 65^o)\\
& = 600 (0.9063)\\
& \approx 544\ \text{ft}
\end{align*}\\
Therefore, height of the communication tower is 544 feet approximately.

Given that a man, flying a kite holds the end of a kite string which is 450 ft long
at ground level and the angle of elevation of the kite is $50^o$. Let $h$ be the
height where the kite is above the ground as shown in the figure below.\\
\\
Then, from the trigonometric ratios, \begin{align*}
\sin 50^o & = \dfrac{h}{450}\\
h & = 450 (\sin 50^o)\\
& = 450 (0.766)\\
& \approx 345\ \text{ft}
\end{align*}\\
Therefore, the kite is approximately 345 feet above the ground.
Given that the pole is 60 ft tall. The angle of depression to the bottom of the
pole from the woman standing on the hill is $14^o$ and the angle of elevation to
the top of the pole is $18^o$. Let $x$ be the distance between her and the pole as
shown in the below figure.\\
\\
Let $h$ and $h'$ be the parts of the flagpole above and below her line of eyesight
as shown in the figure. Then, $$ h + h' = 60\ .\ .\ .\ .\ .\ .\ .\ .\ (1)$$\\
Now, for the upper triangle, $$\tan 18^o = \dfrac{h}{x}\ \Rightarrow h = x \tan
18^o$$
Now, for the lower triangle, $$\tan 14^o = \dfrac{h'}{x}\ \Rightarrow h' = x \tan
14^o$$\\
Using information from above and (1), we get \begin{align*}
h + h' & = 60\\
x \tan 18^o + x \tan 14^o & = 60\\
x (\tan 18^o + \tan 14^o) & = 60\\
x & = \dfrac{60}{\tan 18^o + \tan 14^o}\\
& = \dfrac{60}{0.5742477}\\
& = 104.5\ \text{ft}\\
\end{align*}
Therefore, her distance from the pole is $104.5$ ft.

Given that the water tower is located 325 ft from a building. The angle of
depression to the bottom of the tower from the observer watching from a window is
$25^o$ and the angle of elevation to the top of the tower is $39^o$ as shown in the
below figure.\\
\\
Let $h$ and $h'$ be the parts of the water tower above and below the line of
eyesight of the observer as shown in the figure. Then, $h'$ will be the height of
the window.\\
\\
Now, for the upper triangle, $$\tan 39^o = \dfrac{h}{325}\ \Rightarrow h = 325 \tan
39^o$$
Now, for the lower triangle, $$\tan 25^o = \dfrac{h'}{325}\ \Rightarrow h' = 325
\tan 25^o$$\\
Since height of the tower $H$ is $h + h'$, using information from above, we get
\begin{align*}
H & = h + h'\\
& = 325 \tan 39^o + 325 \tan 25^o\\
& = 325 (\tan 39^o + \tan 25^o)\\
& = 325 (1.276)\\
& = 414.7\ \text{ft}\\
\end{align*}
Therefore, height of the tower is $414.7$ ft.\\
\\
And the window is at a height of \begin{align*}
h' & = 325 \tan 25^o\\
& = 325 (0.46630765815)\\
& \approx 151.55\ \text{ft}
\end{align*}

Given that an airplane is at an elevation of 5150 ft directly above a straight


highway. The angle of depressions to two motorists driving cars on the highway on
opposite sides of the plane are $35^o$ and $52^o$ respectively as shown in the
figure below. From alternate interior angles theorem, angle of elevation from the
motorists to the airplane will also be equal to $35^o$ and $52^o$ respectively. \\
\\
Let $a$ and $b$ be the horizontal distance from the airplane to the motorists as
shown below. Then, for the left triangle, we have \begin{align*}
\tan 35^o & = \dfrac{5150}{a}\\
a & = \dfrac{5150}{\tan 35^o}\\
& = \dfrac{5150}{0.7}\\
& \approx 7355\ \text{ft}
\end{align*}\\
Similarly, for the right triangle, we have \begin{align*}
\tan 52^o & = \dfrac{5150}{b}\\
b & = \dfrac{5150}{\tan 52^o}\\
& = \dfrac{5150}{1.28}\\
& \approx 4024\ \text{ft}
\end{align*}
Therefore, the distance between the cars is \begin{align*}
a + b & = 7355 + 4024\\
& = 11,379\ \text{ft}
\end{align*}

Given that an airplane is at an elevation of 5150 ft directly above a straight


highway. The angle of depressions to two motorists driving cars on the highway on
the same side of the plane are $38^o$ and $52^o$ respectively as shown in the
figure below. From alternate interior angles theorem, angle of elevation from the
motorists to the airplane will also be equal to $38^o$ and $52^o$ respectively. \\
\\
Let $a$ and $b$ be the horizontal distance from the airplane to the motorists as
shown below. Then, for the larger triangle, we have \begin{align*}
\tan 38^o & = \dfrac{5150}{b}\\
b & = \dfrac{5150}{\tan 38^o}\\
& = \dfrac{5150}{0.7812856265}\\
& \approx 6591.7\ \text{ft}
\end{align*}\\
Similarly, for the smaller triangle, we have \begin{align*}
\tan 52^o & = \dfrac{5150}{a}\\
a & = \dfrac{5150}{\tan 52^o}\\
& = \dfrac{5150}{1.28}\\
& \approx 4023.6\ \text{ft}
\end{align*}
Therefore, the distance between the cars is \begin{align*}
b - a & = 6591.7 - 4023.6\\
& \approx 2568\ \text{ft}
\end{align*}

Given that a hot-air balloon is floating above a straight road. The angle of
depressions to two consecutive mileposts on the road on the same side of the
balloon are $20^o$ and $22^o$ respectively as shown in the figure below. From
alternate interior angles theorem, angle of elevation from the mileposts to the
hot-air balloon will also be equal to $20^o$ and $22^o$ respectively. \\
\\
Let $a$ be the horizontal distance from the balloon to the nearer milepost as shown
below. Then, for the smaller triangle, we have \begin{align*}
\tan 22^o & = \dfrac{h}{a}\\
a & = \dfrac{h}{\tan 22^o}\\
\end{align*}
Similarly, for the larger triangle, we have \begin{align*}
\tan 20^o & = \dfrac{h}{1 + a}\\
1 + a & = \dfrac{h}{\tan 20^o}\\
1 + \dfrac{h}{\tan 22^o} & = \dfrac{h}{\tan 20^o}\\
h\bigg( \dfrac{1}{\tan 20^o} - \dfrac{1}{\tan 22^o}\bigg) & = 1\\
h(\cot 20^o - \cot 22^o) & = 1\\
h & = \dfrac{1}{0.2724}\\
& \approx 3.67\ \text{mi.}
\end{align*}\\
Therefore, the balloon is at a height of 3.67 miles approximately.\\

Given that the angle of elevations to the top of a mountain from the plain level is
$32^o$ and after one thousand feet closer to the mountain along the plain is $35^o$
as shown in the figure below. \\
\\
Let $a$ be the distance from the mountain to point where elevation is $35^o$ as
shown below and $h$ be the height of the mountain. Then, for the smaller triangle,
we have \begin{align*}
\tan 35^o & = \dfrac{h}{a}\\
a & = \dfrac{h}{\tan 35^o}\\
\end{align*}
Similarly, for the larger triangle, we have \begin{align*}
\tan 32^o & = \dfrac{h}{1000 + a}\\
1000 + a & = \dfrac{h}{\tan 32^o}\\
1000 + \dfrac{h}{\tan 35^o} & = \dfrac{h}{\tan 32^o}\\
h\bigg( \dfrac{1}{\tan 32^o} - \dfrac{1}{\tan 35^o}\bigg) & = 1000\\
h(\cot 32^o - \cot 35^o) & = 1000\\
h & = \dfrac{1000}{0.1722}\\
& \approx 5808\ \text{ft}
\end{align*}\\
Therefore, the height of the mountain is 5808 feet approximately.\\

Given that a worker shines a spotlight towards clouds upward at an angle $75^o$
from the horizontal and an observer 600 m away measures the angle of elevation to
the spot of light to be $45^o$ as shown in the figure below. \\
\\
Let $a$ be the horizontal distance from the cloud to the spotlight. Since, the
distance between spotlight and the observer is 600 m, the horizontal distance from
the cloud to the observer will be $600 - a$ as shown below and let $h$ be the
height of the cloud cover. Then, for the right triangle, we have \begin{align*}
\tan 75^o & = \dfrac{h}{a}\\
a & = \dfrac{h}{\tan 75^o}\\
\end{align*}
Similarly, for the left triangle, we have \begin{align*}
\tan 45^o & = \dfrac{h}{600 - a}\\
h & = \bigg(600 - \dfrac{h}{\tan 75^o}\bigg) \tan 45^o\\
h & = 600 - \dfrac{h}{\tan 75^o}\\
h \bigg(1 + \dfrac{1}{\tan 75^o}\bigg) & = 600\\
h (1 + \cot 75^o) & = 600\\
h & = \dfrac{600}{1.268}\\
& \approx 473\ \text{m}
\end{align*}\\
Therefore, the height of the cloud cover is 473 m approximately.

Given situation from exercise is shown in the figure below. Let $d_{em}$ and
$d_{es}$ be the distances from earth to moon and earth to sun respectively. Then,
from the figure, we have \begin{align*}
\cos 89.85^o & = \dfrac{d_{em}}{d_{es}}\\
d_{es} & = \dfrac{d_{em}}{\cos 89.85^o}\\
& = \dfrac{240,000}{0.00261799088}\\
& = 91,673,352.2\ \text{mi.}
\end{align*}\\
Therefore, the distance from earth to the sun is 91,673,352.2 miles.

Points $A$ and $B$ are 6155 miles apart and radius of the earth is 3960 miles as
shown in the figure below.\\
\\
(a)\ Now, we know that when the length $s$ of an arc subtends a central angle of
$\theta$ radians, then $$\theta = \dfrac{s}{r}$$
Therefore, the central angle $\theta$ subtended by the arc $AB$ is $$\theta =
\dfrac{AB}{OA} = \dfrac{6155}{3960}$$
Let's multiply it by $\dfrac{180^o}{\pi}$ to convert it into degrees.
\begin{align*}
\theta & = \dfrac{6155}{3960} \times \dfrac{180^o}{\pi}\\
& = 89^o\\
\end{align*}\\
\\
(b)\ In the given right triangle, \begin{align*}
\cos \theta & = \dfrac{OB}{OM}\\
\cos 89^o & = \dfrac{3960}{OA + AM}\\
3960 + AM & = \dfrac{3960}{0.01745240643}\\
AM & = 226,902.8 - 3960\\
& = 222,942.8\ \text{mi.}
\end{align*}\\
Therefore, the distance from point $A$ to the moon is $222,942.8$ miles.

Given situation in exercise is shown in the below figure. From a satellite, $TS =
600$ miles above the earth, it is observed that the angle formed by the vertical
and the line of sight to the horizon is $60.276^o$. Let $R$ be the radius of the
earth.\\
\\
From the figure, in the right triangle HES, \begin{align*}
\sin 60.276^o & = \dfrac{EH}{ES}\\
EH & = ES (\sin 60.276^o)\\
R & = (R+600) (0.8684239)\\
R(1 - 0.8684239) & = 521.05434\\
& = \dfrac{521.05434}{0.1315761}\\
& \approx 3960\ \text{mi.}
\end{align*}\\
Therefore, the radius of the earth is approximately $3960$ miles.\\
Given situation in exercise is shown in the below figure. The angle $E_1SO$ is
called the parallex of the star. Distance from the earth to the sun $E_1O = E_2O =
9.3 \times 10^7$ mi. \\
\\
From the figure, in the right triangle $E_1OS$, \begin{align*}
\sin 0.000211^o & = \dfrac{E_1O}{E_1S}\\
E_1S & = \dfrac{E_1O}{\sin 0.000211^o}\\
& = \dfrac{9.3 \times 10^7}{3.68264 \times 10^{-6}}\\
& \approx 2.52536 \times 10^{13}\ \text{mi.}
\end{align*}\\
Therefore, the distance to this star is approximately $2.52536 \times 10^{13}$
miles.

Given that when $\alpha = 46.3^o$, the earth, Venus, and the sun form a right
triangle with a right angle at Venus as shown in the figure below. Also, distance
between the earth and the sun is 1 AU.\\
\\
Now, from the right triangle SVE, we have \begin{align*}
\sin \alpha & = \dfrac{VS}{ES}\\
VS & = ES \sin \alpha\\
& = 1 (\sin 46.3^o)\\
& \approx 0.723\ \text{AU}
\end{align*}
Therefore, distance between Venus and the sun is 0.723 astronomical units.

If two triangles are similar, then \\


(i)\ Their corresponding angles are congruent.\\
(ii)\ Their corresponding sides are all in the same proportion.\\
\\
For example, in the below figure, $\Delta$PQR is similar to $\Delta$STU. This can
be represented as $$\Delta PQR \sim \Delta STU$$
The corresponding angle are congruent, i.e. $$\angle P = \angle S,\ \angle Q =
\angle T,\ \angle R = \angle U$$
And the corresponding sides are all in the same proportion, i.e. $$\dfrac{PQ}{ST} =
\dfrac{QR}{TU} = \dfrac{PR}{SU}$$\\

Since corresponding sides are all in same proportions, we can see that $$\dfrac{PQ}
{PR} = \dfrac{ST}{SU}\ \Rightarrow \sin R = \sin U$$
Similarly, we can see that $$\dfrac{QR}{PR} = \dfrac{TU}{SU}\ \Rightarrow \cos R
= \cos U$$
Similarly, we can see that $$\dfrac{PQ}{QR} = \dfrac{ST}{TU}\ \Rightarrow \tan R
= \tan U$$\\
Therefore, it is possible to define the trigonometric ratios without the regard to
the size of the triangle.

Angle $\theta$ is in standard position and $P(x,y)$ is a point on the terminal side
of $\theta$, and $r$ is the distance from the origin to $P$ as shown in the figure
below.\\
\\
Then \begin{align*}
\sin \theta & = \dfrac{PN}{OP} = \dfrac{y}{r}\\
\\
\cos \theta & = \dfrac{ON}{OP} = \dfrac{x}{r}\\
\\
\tan \theta & = \dfrac{PN}{ON} = \dfrac{y}{x}\\
\end{align*}

Let $P(x,y)$ be the terminal point determined by an arc of length $t$ on unit
circle. Then $t$ subtends an angle $\theta$ at the center of the circle. If we drop
a perpendicular from $P$ onto the point $N$ on the $x$-axis, then triangle
$\Delta$OPN is a right triangle with legs of length $x$ and $y$ as shown in the
below figure.\\
\\
By the definition of the trigonometric functions of angle $\theta$, we have $
$\sin \theta = \dfrac{\text{opp}}{\text{hyp}} = \dfrac{y}{1} = y$$
$$\cos \theta = \dfrac{\text{adj}}{\text{hyp}} = \dfrac{x}{1} = x$$\\
The sign of a trigonometric function of $\theta$ depends on the $\mathbf{quadrant}$
in which the terminal side of angle $\theta$ lies.\\
\\
In quadrant II, $y$ is positive and therefore, $\sin \theta = y$ is
$\mathbf{positive}$\\
\\
In quadrant III, $x$ is negative and therefore, $\cos \theta = x$ is
$\mathbf{negative}$\\
\\
In quadrant IV, $y$ is negative and therefore, $\sin \theta = y$ is
$\mathbf{negative}$

The $\mathbf{reference\ angle}$ $\overline{\theta}$ associated with


$\mathbf{\theta}$ is the acute angle formed by the terminal side of
$\mathbf{\theta}$ and the $x$-axis. \\
\\
$$\dfrac{7\pi}{10}$$
The reference angle is the acute angle formed by the terminal side of the angle
$\dfrac{7\pi}{10}$ and the $x$-axis.\\
\\
\\
Let's multiply it by $\dfrac{180^o}{\pi}$ to convert it into degrees. $
$\dfrac{7\pi}{10} = \dfrac{7\pi}{10} \times \dfrac{180^o}{\pi} = 126^o $$\\
Since the terminal side of this angle is in Quadrant II, the reference angle is $
$\overline{\theta} = \pi - \dfrac{7\pi}{10} = \dfrac{3\pi}{10}$$\\

The $\mathbf{reference\ angle}$ $\overline{\theta}$ associated with


$\mathbf{\theta}$ is the acute angle formed by the terminal side of
$\mathbf{\theta}$ and the $x$-axis. \\
\\
$$\dfrac{10\pi}{3}$$
The above angle can be written as $$\dfrac{10\pi}{3} = 2\pi + \dfrac{4\pi}{3}$$
The reference angle is the acute angle formed by the terminal side of the angle
$\dfrac{4\pi}{3}$ and the $x$-axis.\\
\\
\\
Let's multiply it by $\dfrac{180^o}{\pi}$ to convert it into degrees. $
$\dfrac{4\pi}{3} = \dfrac{4\pi}{3} \times \dfrac{180^o}{\pi} = 240^o $$\\
\\
Since the terminal side of this angle is in Quadrant III, the reference angle is $
$\overline{\theta} = \dfrac{4\pi}{3} - \pi = \dfrac{\pi}{3}$$\\

Given that a rocket is fired straight up and is tracked by an observer on the


ground 1 mi. away as shown in the below figure. Let $h$ be the height of the rocket
above the ground.\\
\\
(a)\ When the angle of elevation is $\theta$, then \begin{align*}
\tan \theta & = \dfrac{\text{opp}}{\text{adj}}\\
& = \dfrac{h}{1\ \text{mi.}}\\
\tan \theta & = \dfrac{h}{5280\ \text{ft}}\\
h & = 5280\ \tan \theta
\end{align*}

(b)\ From above, the height of rocket at an angle of elevation $\theta$ is $$h =
5280\ \tan \theta$$
So, height of rocket at different elevations given is \begin{align*}
h_{20^o} & = 5280\ \tan 20^o \approx 1922\ \text{ft}\\
\\
h_{60^o} & = 5280\ \tan 60^o \approx 9145\ \text{ft}\\
\\
h_{80^o} & = 5280\ \tan 80^o \approx 29944\ \text{ft}\\
\\
h_{85^o} & = 5280\ \tan 85^o \approx 60351\ \text{ft}\\
\end{align*}\\
Therefore, the complete table of heights at different angles of elevation is shown
below.
$$\setlength{\arrayrulewidth}{0.5mm}
\setlength{\tabcolsep}{12pt}
\renewcommand{\arraystretch}{2}
\begin{tabular}{|p{0.5cm}| p{1cm}| p{1cm} | p{1cm}| p{1cm}|}\hline
$\theta$ & $\ 20^o$ & $\ 60^o$ & $\ 80^o$ & $\ 85^o$ \\
\hline
$h$ & $1922$ & $9145$ & $29944$ & $60351$\\
\hline
\end{tabular}$$\\

Given that a rain gutter is constructed from a metal sheet of width 30 cm by


bending up one-third of the sheet on each side through an angle $\theta$ as shown
in the below figure. \\
\\
(a)\ The cross-sectional area $A$ of the gutter is \begin{align*}
A & = \text{Area of}\ CEFD + \text{Area of}\ \Delta ACE + \text{Area of}\ \Delta
BDF \\
& = (EF)(DF) + \dfrac{1}{2}(AC)(EC) + \dfrac{1}{2}(DB)(DF)\\
& = (EF)(DF) + \dfrac{1}{2}[(AC)(EC) + (DB)(DF)]
\end{align*}\\
In the $\Delta$ACE, we have \begin{align*}
AC & = 10\ \cos \theta \\
EC & = 10\ \sin \theta
\end{align*}\\
Similarly, in the $\Delta$DBF, $$DB = 10\ \cos \theta\ \text{and}\ DF = 10\ \sin
\theta$$
Therefore, area of the cross-section of the gutter is \begin{align*}
A(\theta) & = (10)(10 \sin \theta) + \dfrac{1}{2}[100 \sin \theta \cos \theta + 100
\sin \theta \cos \theta]\\
& = 100 \sin \theta + \dfrac{1}{2}(200 \sin \theta \cos \theta)\\
& = 100 \sin \theta + 100 \sin \theta \cos \theta
\end{align*}

(b)\ To find the larges cross-sectional area, we differentiate $A(\theta)$ and find
the critical points. \begin{align*}
A(\theta) & = 100 \sin \theta + 100 \sin \theta \cos \theta\\
A'(\theta) & = 100 \cos \theta + 100(\cos^2 \theta - \sin^2 \theta)\\
& = 100(\cos^2 \theta + \cos \theta - (1 - \cos^2 \theta))\\
& = 100(2\cos^2 \theta + \cos \theta - 1)\\
& = 100(2\cos \theta - 1)(\cos \theta + 1)
\end{align*}\\
For critical points, find $\theta$ such that $A'(\theta) = 0$, i.e. \begin{align*}
A'(\theta) & = 0\\
100(2\cos \theta - 1)(\cos \theta + 1) & = 0\\
\cos \theta = \dfrac{1}{2}\ \text{or}\ & \cos \theta = -1\\
\theta = \dfrac{\pi}{3}\ \text{or}\ & \theta = \pi
\end{align*}\\
Upon substituting critical points, we get $$A(\pi) = 100 \sin \pi + 100 \sin \pi
\cos \pi = 0$$ and $$A(\pi/3) = 100 \sin \pi/3 + 100 \sin \pi/3 \cos \pi/3 \approx
130\ \text{cm}^2$$
Therefore, largest cross-sectional area is achieved for $\theta = \dfrac{\pi}{3}$

Given that a rectangular beam is cut from a cylindrical log of diameter 20 cm in


one of the ways as shown in the figure below.\\
\\
(a)\ The cross-sectional area $A$ of the beam is \begin{align*}
A & = \text{Area of the rectangle}\ ABCD \\
& = (\text{width})(\text{depth})
\end{align*}\\
In the $\Delta$ACD, we have \begin{align*}
\text{width} & = 20 \cos \theta \\
\text{depth} & = 20 \sin \theta
\end{align*}\\
Therefore, area of the cross-section of the beam is \begin{align*}
A(\theta) & = (20 \cos \theta)(20 \sin \theta)\\
& = 400 \cos \theta \sin \theta
\end{align*}\\

(c)\ From the above graph, we see that the largest cross sectional area is equal to
200 square cm which is achieved at an angle of $\theta = \dfrac{\pi}{4}$.\\
\\
Therefore, dimensions of the beam with largest cross-sectional area are
\begin{align*}
\text{width} & = 20 \cos \pi/4\\
& = 20\bigg(\dfrac{1}{\sqrt{2}}\bigg)\\
& \approx 14.14\ \text{cm}\\
\\
\text{depth} & = 20 \sin \pi/4\\
& = 20\bigg(\dfrac{1}{\sqrt{2}}\bigg)\\
& \approx 14.14\ \text{cm}\\
\end{align*}

It is given that the strength of the beam is proportional to the width and the
square of the depth, i.e. $$\text{Strength} \propto (\text{width})(\text{depth})^2$
$\\
In the $\Delta$ACD, we have \begin{align*}
\text{width} & = 20 \cos \theta \\
\text{depth} & = 20 \sin \theta
\end{align*}\\
Then, strength of the beam as a function of the angle $\theta$ will be
\begin{align*}
\text{Strength} & \propto (\text{width})(\text{depth})^2\\
& \propto (20 \cos \theta)(20 \sin \theta)^2\\
& \propto (20 \cos \theta)(400 \sin^2 \theta)\\
\text{Strength} & \propto 8000 \cos \theta \sin^2 \theta\\
\end{align*}

The range $R$ and height $H$ of a shot put thrown with an initial velocity of $v_0$
ft/s at an angle $\theta$ are given by $$R = \dfrac{v_0^2 \sin (2\theta)}{g}\
\text{and}\ H = \dfrac{v_0^2 \sin^2 \theta}{2g}$$\\
\\
(a)\ A shot put is thrown on the earth with $v_0 = 12$ ft/s and $\theta = \pi/6$
where $g = 32$ ft/s$^2$ \begin{align*}
\text{Range}\ R & = \dfrac{(12)^2 \sin (2 (\pi/6))}{32}\\
& = \dfrac{144 \sin (\pi/3)}{32}\\
& = (4.5)\bigg(\dfrac{\sqrt{3}}{2}\bigg)\\
& \approx 3.897\ \text{ft}\\
\\
\text{Height}\ H & = \dfrac{(12)^2 \sin^2 (\pi/6)}{2(32)}\\
& = \dfrac{144 (\sin (\pi/6))^2}{64}\\
& = (2.25)\bigg(\dfrac{1}{4}\bigg)\\
& = 0.5625\ \text{ft}
\end{align*}

(b)\ A shot put is thrown on the moon with $v_0 = 12$ ft/s and $\theta = \pi/6$
where $g = 5.2$ ft/s$^2$ \begin{align*}
\text{Range}\ R & = \dfrac{(12)^2 \sin (2 (\pi/6))}{5.2}\\
& = \dfrac{144 \sin (\pi/3)}{5.2}\\
& = \bigg(\dfrac{360}{13}\bigg) \bigg(\dfrac{\sqrt{3}}{2}\bigg)\\
& \approx 23.98\ \text{ft}\\
\\
\text{Height}\ H & = \dfrac{(12)^2 \sin^2 (\pi/6)}{2(5.2)}\\
& = \dfrac{144 (\sin (\pi/6))^2}{10.4}\\
& = \bigg(\dfrac{180}{13}\bigg)\bigg(\dfrac{1}{4}\bigg)\\
& \approx 3.46\ \text{ft}
\end{align*}

It is given that the time in seconds it takes for a sled to slide down a hillside
inclined at an angle $\theta$ is $$t = \sqrt{\dfrac{d}{16 \sin \theta}}$$ where $d$
is the length of the slope in feet.\\
\\
Then, the time it takes to slide down a 2000-ft slope inclined at $30^o$ is
\begin{align*}
t & = \sqrt{\dfrac{2000}{16 \sin 30^o}}\\
& = \sqrt{\dfrac{125}{1/2}}\\
& = \sqrt{250}\\
& \approx 15.8\ \text{seconds}
\end{align*}

Given that the amount of wax $W$ in the cell depends on the apex angle $\theta$ and
is given by $$W = 3.02 - 0.38 \cot \theta + 0.65 \csc \theta$$\\
(a)\ The graph of $W$ as a function of $\theta$ for $0 < \theta < \pi$, i.e. for $0
< \theta < 180^o$ is shown below.\\
\\
\\
(b)\ From the below graph, we see that $W$ has its minimum value equal to $3.547$
at an angle of $\theta = 54.224^o$ or $0.946$ in radians.

Given situation is shown in the figure below. From alternate angles theorem, the
angle between narrower hallway and the steel pipe is also equal to the angle
$\theta$ as shown below.\\
\\
(a)\ From the figure, we see that the length of the pipe $L$ is $$L = AB +
BC\ .\ .\ .\ .\ .\ .\ .\ (1)$$\\
Now, in the triangle made by the wide hallway and the pipe, \begin{align*}
\csc \theta & = \dfrac{\text{hyp}}{\text{opp}}\\
& = \dfrac{AB}{9}\\
AB & = 9 \csc \theta\ .\ .\ .\ .\ .\ .\ .\ (2)
\end{align*}\\
\\
Now, in the triangle made by the narrow hallway and the pipe, \begin{align*}
\sec \theta & = \dfrac{\text{hyp}}{\text{adj}}\\
& = \dfrac{BC}{6}\\
BC & = 6 \sec \theta\ .\ .\ .\ .\ .\ .\ .\ (3)
\end{align*}\\
Therefore, from (1), (2) and (3), the length of the pipe in the figure can be
modeled by the function $$L(\theta) = 9 \csc \theta + 6 \sec \theta$$

The angle of elevation $\theta$ of a rainbow is always the same with $$\theta =
4 \beta - 2\alpha,$$ where $\sin \alpha = k \sin \beta$ and $\alpha = 59.4^o$ and
$k = 1.33$ is the index of refraction of water. \\
\\
From above information, we get \begin{align*}
\sin \beta & = \dfrac{\sin \alpha}{k}\\
& = \dfrac{\sin 59.4^o}{1.33}\\
& = \dfrac{0.860742027}{1.33}\\
& = 0.64717445639\\
\beta & = \sin^{-1} (0.64717445639)\\
& \approx 40.33^o
\end{align*}\\
Therefore, the angle of elevation $\theta$ of a rainbow is \begin{align*}
\theta & = 4\beta - 2 \alpha\\
& = 4(40.33^o) - 2(59.4^o)\\
& = 161.32^o - 118.8^o\\
& = 42.52^o
\end{align*}

Calculation of my partner, i.e. $$\sin 4 = 0.0697564737$$ is wrong since we were


asked to find the sine of 4 radians but my partner calculated sine of 4 degrees. He
did not set his calculator to radians before calculating and so calculated sine of
4 degrees instead of sine of 4 radians.

For the given figure, the radius $OR$ of the circle is 1, the center is $O$,
segment $QS$ is tangent to the circle at $R$, and $\angle$SOQ is a right angle. \\
\\
Since $\angle$SOQ $ = 90^o$ and $\angle$ROQ $\theta$, then $\angle$SOR $=
(90-\theta)^o$.\\
\\
Since QS is tangent to the circle at $R$, $\angle$SRO $= \angle$QRO $ = 90^o$.
Therefore, $$\angle OSR = \theta$$\\
Now, in $\Delta$OSR, we have \begin{align*}
\csc \theta & = \dfrac{\text{hyp}}{\text{opp}} = \dfrac{|OS|}{|OR|} = \dfrac{|OS|}
{1} = |OS|\\
\\
\cot \theta & = \dfrac{\text{adj}}{\text{opp}} = \dfrac{|SR|}{|OR|} = \dfrac{|SR|}
{1} = |SR|\\
\end{align*}\\
\\
Since $\angle$ROQ $ = \theta$, then $\angle$RQO $= (90-\theta)^o$. Since
$\angle$RPQ $= 90^o$. Therefore, $$\angle PRQ = \theta$$\\
\\
Now, in $\Delta$OPR, we have \begin{align*}
\cos \theta & = \dfrac{\text{adj}}{\text{hyp}} = \dfrac{|OP|}{|OR|} = \dfrac{|OP|}
{1} = |OP|\\
\end{align*}\\
Now, in $\Delta$OQR, we have \begin{align*}
\tan \theta & = \dfrac{\text{opp}}{\text{adj}} = \dfrac{|QR|}{|OR|} = \dfrac{|QR|}
{1} = |QR|\\
\\
\sec \theta & = \dfrac{\text{hyp}}{\text{adj}} = \dfrac{|OQ|}{|OR|} = \dfrac{|OQ|}
{1} = |OQ|
\end{align*}

(a)\ First Pythagorean identity is $$\sin^2 \theta + \cos^2 \theta = 1$$\\ Dividing
both the sides by $\cos^2 \theta$, we get \begin{align*}
\dfrac{\sin^2 \theta + \cos^2 \theta}{\cos^2 \theta} & = \dfrac{1}{\cos^2 \theta}\\
\dfrac{\sin^2 \theta}{\cos^2 \theta} + \dfrac{\cos^2 \theta}{\cos^2 \theta} & =
\bigg(\dfrac{1}{\cos \theta}\bigg)^2\\
\\
\bigg(\dfrac{\sin \theta}{\cos \theta}\bigg)^2 + 1 & = \sec^2 \theta\ \ \ \ \ \ \ \
\ \bigg[\ \text{Since}\ \dfrac{1}{\cos \theta} = \sec \theta\ \bigg]\\
\\
\tan^2 \theta + 1 & = \sec^2 \theta\ \ \ \ \ \ \ \ \ \bigg[\ \text{Since}\
\dfrac{\sin \theta}{\cos \theta} = \tan \theta\ \bigg]
\end{align*}

(b)\ First Pythagorean identity is $$\sin^2 \theta + \cos^2 \theta = 1$$\\ Dividing
both the sides by $\sin^2 \theta$, we get \begin{align*}
\dfrac{\sin^2 \theta + \cos^2 \theta}{\sin^2 \theta} & = \dfrac{1}{\sin^2 \theta}\\
\dfrac{\sin^2 \theta}{\sin^2 \theta} + \dfrac{\cos^2 \theta}{\sin^2 \theta} & =
\bigg(\dfrac{1}{\sin \theta}\bigg)^2\\
\\
1 + \bigg(\dfrac{\cos \theta}{\sin \theta}\bigg)^2 & = \csc^2
\theta\ \ \ \ \ \ \ \ \ \bigg[\ \text{Since}\ \dfrac{1}{\sin \theta} = \csc \theta\
\bigg]\\
\\
1 + \cot^2 \theta & = \csc^2 \theta\ \ \ \ \ \ \ \ \ \bigg[\ \text{Since}\
\dfrac{\cos \theta}{\sin \theta} = \cot \theta\ \bigg]
\end{align*}

The earth is a sphere of radius $r = 3960$ mi and let $\theta,\ s$ and $h$ be as
shown in the figure.\\
\\
(a)\ For the right triangle formed, we have \begin{align*}
\cos \theta & = \dfrac{\text{adj}}{\text{hyp}}\\
& = \dfrac{r}{r+h}\\
& = \dfrac{3960}{3960 + h}\\
\theta & = \cos^{-1}\bigg(\dfrac{3960}{3960 + h}\bigg)
\end{align*}\\
\\
\\
(b)\ Now, we know that when the length $s$ of an arc subtends a central angle of
$\theta$ radians, then $$\theta = \dfrac{s}{r}$$
For the below figure, central angle is $2\theta$, then, we have \begin{align*}
2\theta & = \dfrac{s}{r}\\
s & = 2r \theta\\
& = 2(3960) \theta\\
& = 7920\theta
\end{align*}

(c) From (b), we got that $$s = 7920 \theta$$ and from (a), we got that $$\theta
= \cos^{-1}\bigg(\dfrac{3960}{3960 + h}\bigg)$$
Therefore, using the above two equations, we can say $$s = 7920 \cos^{-
1}\bigg(\dfrac{3960}{3960 + h}\bigg)$$\\
\\
(d)\ If the satellite is $h = 100$ mi above the earth, the distance $s$ that it can
see is \begin{align*}
s & = 7920 \cos^{-1}\bigg(\dfrac{3960}{3960 + h}\bigg)\\
& = 7920 \cos^{-1}\bigg(\dfrac{3960}{3960 + 100}\bigg)\\
& = 7920 \cos^{-1}\bigg(\dfrac{3960}{4060}\bigg)\\
& = 7920 \cos^{-1}(0.97536945812)\\
& = 7920 (0.222406483)\\
& \approx 1761.46\ \text{mi}
\end{align*}\\
\\
(e)\ If the satellite has to see both Los Angeles and New York, which are $s =
2450$ mi apart, then \begin{align*}
2450 & = 7920 \cos^{-1}\bigg(\dfrac{3960}{3960 + h}\bigg)\\
\dfrac{2450}{7920} & = \cos^{-1}\bigg(\dfrac{3960}{3960 + h}\bigg)\\
\cos (0.30934343434) & = \dfrac{3960}{3960 + h}\\
0.95253365563 & = \dfrac{3960}{3960 + h}\\
3960 + h & = 4157.33341976\\
h & \approx 197.33\ \text{mi}
\end{align*}\\
Therefore, the satellite has to be at a height of approximately 197 miles so as to
see both Los Angeles and New York.

Let $y = \sec^{-1} x$, then \begin{align*}


y & = \sec^{-1} x\\
\sec y & = x\\
\dfrac{1}{\cos y} & = x\\
\cos y & = \dfrac{1}{x}\\
y & = \cos^{-1}\bigg(\dfrac{1}{x}\bigg)\\
\sec^{-1} x & = \cos^{-1}\bigg(\dfrac{1}{x}\bigg)
\end{align*}\\
\\
Then $\sec^{-1}2 = \cos^{-1}\bigg(\dfrac{1}{2}\bigg) \approx 1.05$ radians

Let $y = \csc^{-1} x$, then \begin{align*}


y & = \csc^{-1} x\\
\csc y & = x\\
\dfrac{1}{\sin y} & = x\\
\sin y & = \dfrac{1}{x}\\
y & = \sin^{-1}\bigg(\dfrac{1}{x}\bigg)\\
\csc^{-1} x & = \sin^{-1}\bigg(\dfrac{1}{x}\bigg)
\end{align*}\\
\\
Then $\csc^{-1}3 = \sin^{-1}\bigg(\dfrac{1}{3}\bigg) \approx 0.34$ radians

Let $y = \cot^{-1} x$, then \begin{align*}


y & = \cot^{-1} x\\
\cot y & = x\\
\dfrac{1}{\tan y} & = x\\
\tan y & = \dfrac{1}{x}\\
y & = \tan^{-1}\bigg(\dfrac{1}{x}\bigg)\\
\cot^{-1} x & = \tan^{-1}\bigg(\dfrac{1}{x}\bigg)
\end{align*}\\
\\
Then $\cot^{-1}4 = \tan^{-1}\bigg(\dfrac{1}{4}\bigg) \approx 0.25$ radians

Given that a satellite orbiting the earth pass directly over two tracking stations
$A$ and $B$ which are 50 mi apart. When the satellite is on one side of the two
stations, the angles of elevation at $A$ and $B$ are measured to be $87.0^o$ and
$84.2^o$, respectively as shown in the below figure. \\
\\
\\
(a)\ In the triangle $\Delta$ABC, we have $$\angle CAB = 180^o - 87.0^o = 93^o$$
and using triangle sum property, we have \begin{align*}
\angle A + \angle B + \angle C & = 180^o\\
93^o + 84.2^o + \angle C & = 180^o\\
\angle C & = 180^o - 177.2^o\\
& = 2.8^o\\
\end{align*}
Now, using the law of sines for the triangle $\Delta$ABC, we have \begin{align*}
\dfrac{\sin C}{c} & = \dfrac{\sin B}{b}\\
b & = c\bigg(\dfrac{\sin B}{\sin C}\bigg)\\
& = 50 \bigg(\dfrac{\sin 84.2^o}{\sin 2.8^o} \bigg)\\
& \approx 50(20.366)\\
& \approx 1018.3\ \text{mi}
\end{align*}\\
Therefore, satellite is approximately 1018 miles away from station $A$.

(b)\ Let $h$ be the height of the satellite above the ground as shown in the below
figure. Then, using the sine law for the $\Delta$CDA, we get \begin{align*}
\dfrac{\sin D}{b} & = \dfrac{\sin A}{h}\\
h & = b\bigg(\dfrac{\sin A}{\sin D}\bigg)\\
& = 1018.3 \bigg(\dfrac{\sin 87^o}{\sin 90^o} \bigg)\\
& \approx 1018.3(0.99862953475)\\
& \approx 1017\ \text{mi}
\end{align*}\\
Therefore, satellite is at a height of $1017$ miles above the ground.

Given that a pilot is flying over a straight highway and determines the angles of
depression to two mileposts, 5 mi apart, to be $32^o$ and $48^o$. Let $b$ be the
distance of plane from point $A$ as shown in the figure below. \\
\\
(a)\ In the $\Delta$ABC, using alternate angles theorem, we get $$\angle A =
32^o\ \text{and}\ \angle B = 48^o$$
and using triangle sum property, we have \begin{align*}
\angle A + \angle B + \angle C & = 180^o\\
32^o + 48^o + \angle C & = 180^o\\
\angle C & = 180^o - 80^o\\
& = 100^o\\
\end{align*}
Now, using the law of sines for the triangle, we get \begin{align*}
\dfrac{\sin C}{c} & = \dfrac{\sin B}{b}\\
b & = c\bigg(\dfrac{\sin B}{\sin C}\bigg)\\
& = 5 \bigg(\dfrac{\sin 48^o}{\sin 100^o}\bigg)\\
& = 5 (0.7546)\\
& = 3.773\ \text{mi}
\end{align*}\\
Therefore, the plane is at a distance of 3.773 miles from point $A$.

(b)\ Let $h$ be the elevation of plane as shown in the below figure. Then, using
the sine law for the $\Delta$CDA, we get \begin{align*}
\dfrac{\sin D}{b} & = \dfrac{\sin A}{h}\\
h & = b\bigg(\dfrac{\sin A}{\sin D}\bigg)\\
& = 3.773 \bigg(\dfrac{\sin 32^o}{\sin 90^o} \bigg)\\
& \approx 3.773(0.52991926423)\\
& \approx 2\ \text{mi}
\end{align*}
Therefore, elevation of the plane is $2$ miles.

Given that a tree on a hillside casts a shadow 215 ft down the hill. The angle of
inclination of the hillside is $22^o$ to the horizontal and the angle of elevation
of the sun is $52^o$.\\
\\
Using triangle sum property for $\Delta$ABC, we have \begin{align*}
\angle A + \angle B + \angle C & = 180^o\\
52^o + 90^o + \angle C & = 180^o\\
\angle C & = 180^o - 142^o\\
& = 38^o\\
\end{align*}
From the below figure, we have \begin{align*}
\angle CAD & = \angle CAB - \angle DAB \\
& = 52^o - 22^o\\
& = 30^o
\end{align*}\\
Now, using law of sines for the $\Delta$CAD, we have \begin{align*}
\dfrac{\sin C}{AD} & = \dfrac{\sin A}{CD}\\
CD & = AD\bigg(\dfrac{\sin A}{\sin C}\bigg)\\
& = 215 \bigg(\dfrac{\sin 30^o}{\sin 38^o}\bigg)\\
& = 215 (0.81213462274)\\
& = 174.6\ \text{ft}
\end{align*}

Given that a tower is located at the top of a hill whose angle of inclination is
$58^o$. A guywire is attached to the top of the tower and to the ground, 100 m
downhill from the base of the tower as shown in the figure below.\\
\\
Using triangle sum property for $\Delta$ABC, we have \begin{align*}
\angle DAB + \angle CAD + \angle B + \angle C & = 180^o\\
58^o + 12^o + 90^o + \angle C & = 180^o\\
\angle C & = 180^o - 160^o\\
& = 20^o\\
\end{align*}
Using triangle sum property for $\Delta$ADC, we have \begin{align*}
\angle CAD + \angle ADC + \angle C & = 180^o\\
12^o + \angle ADC + 20^o & = 180^o\\
\angle ADC & = 180^o - 32^o\\
& = 148^o\\
\end{align*}
Now, using law of sines for the $\Delta$CAD, we have \begin{align*}
\dfrac{\sin C}{AD} & = \dfrac{\sin ADC}{AC}\\
AC & = AD\bigg(\dfrac{\sin ADC}{\sin C}\bigg)\\
& = 100 \bigg(\dfrac{\sin 148^o}{\sin 20^o}\bigg)\\
& = 100 (1.5493802765)\\
& \approx 155\ \text{m}
\end{align*}\\
Therefore, length of the cable for the guy wire is approximately $155$ m.

Given that observer $P$ is located 60 m from $Q$ down the hill that is inclined at
an angel $32^o$ to the horizontal. The angle of elevations to a hot-air balloon
from $P$ and $Q$ are $62^o$ and $71^o$ respectively as shown in the figure below.\\
\\
Now, in $\Delta$TQU, from angle sum property, we have , \begin{align*}
\angle TQU + \angle TUQ + \angle QTU & = 180^o\\
71^o + 90^o + \angle QTU & = 180^o\\
\angle QTU & = 180^o - 161^o\\
& = 19^o
\end{align*}\\
Now, in $\Delta$TPR, from angle sum property, we have , \begin{align*}
\angle TPR + \angle PRT + \angle RTP & = 180^o\\
62^o + 90^o + \angle RTP & = 180^o\\
\angle RTP & = 180^o - 152^o\\
& = 28^o
\end{align*}\\
Now, from the below figure, \begin{align*}
\angle TPQ & = \angle TPR - \angle QPR\\
& = 62^o - 32^o\\
& = 30^o\\
\\
\text{Also}\ \angle QTP & = \angle RTP - \angle QTU\\
& = 28^o - 19^o\\
& = 9^o
\end{align*}

Now, using the law of sines, we can set up the following equation \begin{align*}
\dfrac{\sin \angle QTP}{PQ} & = \dfrac{\sin \angle TPQ}{QT}\\
QT & = PQ\bigg(\dfrac{\sin \angle TPQ}{\sin \angle QTP}\bigg)\\
& = 60 \bigg(\dfrac{\sin 30^o}{\sin 9^o}\bigg)\\
& = 60 (3.19622661075)\\
& \approx 191.77\ \text{m}
\end{align*}\\
Therefore, distance from point $Q$ to the balloon is approximately 191.77 meters.

Given that the distance from the sun to Venus is $BD = BC = 0.723$ AU and at a
certian time the elongation $\alpha$ of Venus is found to $39.4^o$ in the below
figure. $AC$ and $AD$ are the two possible distances from earth to Venus.\\
\\
Now, using law of sines in $\Delta$ABD, we have \begin{align*}
\dfrac{\sin \angle ADB}{AB} & = \dfrac{\sin \alpha}{BD}\\
\sin \angle ADB & = 1 \bigg(\dfrac{\sin 39.4^o}{0.723}\bigg)\\
& = 0.87791218976\\
\angle ADB & = \sin^{-1}(0.87791218976)\\
& = 61.4^o\ \text{or}\ 118.6^o
\end{align*}\\
Since $\angle$ ADB is obtuse, we will have $$\angle ADB = 118.6^o\ \text{and}\
\angle ACB = 61.4^o$$
\\
Now, in $\Delta$ADB, from angle sum property, we have , \begin{align*}
\angle ADB + \angle ABD + \alpha & = 180^o\\
118.6^o + \angle ABD + 39.4^o& = 180^o\\
\angle ABD & = 180^o - 158^o\\
& = 22^o
\end{align*}\\
Now, in $\Delta$ABC, from angle sum property, we have , \begin{align*}
\angle ABC + \angle ACB + \alpha & = 180^o\\
\angle ABC + 61.4^o + 39.4^o & = 180^o\\
\angle ABC & = 180^o - 100.8^o\\
& = 79.2^o
\end{align*}
Now, using law of sines for $\Delta$ADB, we have \begin{align*}
\dfrac{\sin \angle ADB}{AB} & = \dfrac{\sin \angle ABD}{AD}\\
AD & = AB\bigg(\dfrac{\sin \angle ABD}{\sin \angle ADB}\bigg)\\
& = 1\bigg(\dfrac{\sin 22^o}{\sin 118.6^o}\bigg)\\
& \approx 0.4266\ \text{AU}
\end{align*}\\
Now, using law of sines for $\Delta$ABC, we have \begin{align*}
\dfrac{\sin \angle ACB}{AB} & = \dfrac{\sin \angle ABC}{AC}\\
AC & = AB\bigg(\dfrac{\sin \angle ABC}{\sin \angle ACB}\bigg)\\
& = 1\bigg(\dfrac{\sin 79.2^o}{\sin 61.4^o}\bigg)\\
& \approx 1.1188\ \text{AU}
\end{align*}
Therefore, possible distances from earth to Venus are $$AD = 0.4266\ \text{AU}\
\text{and}\ AC = 1.1188\ \text{AU}$$

Let the two cars diverge at point $A$ with speeds 50 mi/h and 30 mi/h at 2:00 PM as
shown in the figure below. Let $C$ and $B$ be the points where the two cars are at
$2:30$ PM. Then $a$ is the distance between them at that instance.\\
\\
Using distance formula, the distance traveled by the car with speed 30 mi/h will be
\begin{align*}
c & = 30 \times \dfrac{1}{2}\\
& = 15\ \text{mi}
\end{align*}\\
Similarly, the distance traveled by the car with speed 50 mi/h will be
\begin{align*}
b & = 50 \times \dfrac{1}{2}\\
& = 25\ \text{mi}
\end{align*}\\
Now, using the law of cosines, we have \begin{align*}
a^2 & = b^2 + c^2 - 2bc \cos A\\
& = (25)^2 + (15)^2 - 2(25)(15) \cos 65^o\\
& = 625 + 225 - 750(0.42261826174)\\
& = 850 - 317\\
& \approx 533\\
a & \approx \sqrt{533}\\
& \approx 23.09\ \text{mi}
\end{align*}\\
Therefore, the cars are 23.09 miles apart at 2:30 PM.

Let the pilot flying from $A$ for 1 h 30 min changes its course at point $B$,
heading $10^o$ to the right and flies 2 h in the new direction as shown in the
figure below. Let $C$ be her position flying after 2 h in the new direction. Then
$b$ is her distance from the starting position.\\
\\
From the below figure, we have
$$\angle ABC = 180^o - 10^o = 170^o$$
The distance traveled by the pilot for 1 h 30 min will be \begin{align*}
c & = 625 \times (1.5)\\
& = 937.5\ \text{mi}
\end{align*}\\
Similarly, distance traveled by the pilot for 2 h in the new direction will be
\begin{align*}
a & = 625 \times 2\\
& = 1250\ \text{mi}
\end{align*}\\
Now, using the law of cosines, we have \begin{align*}
b^2 & = a^2 + c^2 - 2ac \cos B\\
& = (1250)^2 + (937.5)^2 - 2(1250)(937.5) \cos 170^o\\
& = 1,562,500 + 878,906.25 - 2,343,750(-0.98480775301)\\
& = 2,441,406.25 + 2,308,143.17112\\
& \approx 4,749,549.42\\
b & \approx \sqrt{4,749,549.42112}\\
& \approx 2179.346\ \text{mi}
\end{align*}\\
Therefore, the pilot is 2179.346 miles from her starting position.

Let $a$ be the distance between the fisherman's home port an Forrest Island. \\
\\
(a)\ From the figure, we have $$\angle CAD = 90^o - 10^o = 80^o$$\\
Using angle sum property for $\Delta$BAD, we have \begin{align*}
\angle DAB + \angle ABD + \angle ADB & = 180^o\\
\angle DAB + 70^o + 90^o & = 180^o\\
\angle DAB & = 180^o - 160^o\\
& = 20^o\\
\end{align*}
So, for the $\Delta$ABC, we have $$\angle CAB = \angle CAD + \angle DAB = 80^o +
20^o = 100^o$$\\
Now, using the law of cosines for $\Delta$ABC, we have \begin{align*}
a^2 & = b^2 + c^2 - 2bc \cos A\\
& = (50)^2 + (30)^2 - 2(50)(30) \cos 100^o\\
& = 2500 + 900 - 3000(-0.17364817766)\\
& = 3400 + 520.94453298\\
& \approx 3921\\
a & \approx \sqrt{3921}\\
& \approx 62.62\ \text{mi}
\end{align*}
Therefore, the distance between the fisherman's home port and Forrest Island is
approximately 62.62 miles.

(b)\ Now using the law of sines for $\Delta$ABC, we have \begin{align*}
\dfrac{\sin \angle ACB}{AB} & = \dfrac{\sin \angle CAB}{BC}\\
\dfrac{\sin \angle ACB}{30} & = \dfrac{\sin 100^o}{62.62}\\
\sin \angle ACB & = 30\bigg(\dfrac{0.98480775301}{62.62}\bigg)\\
\sin \angle ACB & = 0.4718018616\\
\angle ACB & = \sin^{-1} (0.4718018616)\\
& \approx 28.15^o
\end{align*}\\
Now, using angle sum property in the $\Delta$ACE, we have \begin{align*}
\angle ACE + \angle AEC + \angle CAE & = 180^o\\
\angle ACE + 90^o + 80^o & = 180^o\\
\angle ACE & = 180^o - 170^o\\
& = 10^o\\
\end{align*}
So, in the figure, we have \begin{align*}
\angle ECD & = \angle ACB - \angle ACE\\
& = 28.15^o - 10^o\\
& = 18.15^o
\end{align*}\\
Therefore, the fisherman is in the direction S $18.15^o$ E from Forrest Island back
to his home port.

Let airports A and B be at the points $A$ and $B$ respectively. It is given that a
pilot wishing to fly from $A$ to $B$ mistakenly flies due east at 200 mi/h for 30
min (half an hour), when he notices his error.\\
\\
(a)\ Let $C$ be the point at which he notices his error. Then $$AC = 200 \times
\bigg(\dfrac{1}{2}\bigg) = 100\ \text{mi}$$
From the below figure, we have $$\angle BAC = 90^o - 50^o = 40^o$$
Now, using the law of cosines, we have \begin{align*}
a^2 & = b^2 + c^2 - 2bc \cos \angle BAC\\
& = (100)^2 + (300)^2 - 2(100)(300) \cos 40^o\\
& = 10,000 + 90,000 - 60,000(0.76604444311)\\
& = 100,000 - 45,962.6665866\\
& \approx 54,037.3334\\
a & \approx \sqrt{54,037.3334}\\
& \approx 232.46\ \text{mi}
\end{align*}
Therefore, the pilot is approximately 232 46 miles far from his destination at the
time he notices his error.

(b)\ Now using the law of sines, we have \begin{align*}


\dfrac{\sin \angle ACB}{AB} & = \dfrac{\sin \angle BAC}{BC}
\dfrac{\sin \angle ACB}{300} & = \dfrac{\sin 40^o}{232.46}
\sin \angle ACB & = 300 \bigg(\dfrac{0.64278760968}{232.46}\bigg)
\sin \angle ACB & =.0.82954608493\\
\angle ACB & = \sin^{-1} (0.82954608493)
& \approx 56\ \text{or}\ 124^o
\end{align*}\\
Since $\angle$ACB is obtuse, $\angle$ACB = $124^o$ and so from the figure, we
have \begin{align*}
\angle BCD & = \angle ACB - \angle ACD\\
& = 124^o - 90^o\\
& = 34^o
\end{align*}
Therefore, in order to arrive at airport $B$, the new heading should be N $34^o$
E\\

We know that area of a triangle $\Delta$ABC with sides $a$ and $b$ and the included
angle $C$ is $$\text{Area} = \dfrac{1}{2}ab \sin C\ .\ .\ .\ .\ .\ .\ .\ .\ (1)$$\\
Also from law of sines we have $$\dfrac{\sin A}{a} = \dfrac{\sin B}{b} =
\dfrac{\sin C}{c}\ .\ .\ .\ .\ .\ .\ (2)$$
From (2), we have $$b \sin A = a \sin B\ \Rightarrow b = a \dfrac{\sin B}{\sin A}$$
Substituting the value of $b$ from the above equation in (1), we get \begin{align*}
\text{Area} & = \dfrac{1}{2}a\bigg(a \dfrac{\sin B}{\sin A}\bigg)\sin C\\
& = \dfrac{a^2 \sin B \sin C}{2 \sin A}
\end{align*}

From the law of cosines, in any triangle $ABC$, we have \begin{align*}


a^2 & = b^2 + c^2 - 2bc\ \cos A\ .\ .\ .\ .\ .\ .\ .\ .\ .\ .\ (1)\\
b^2 & = a^2 + c^2 - 2ac\ \cos B\ .\ .\ .\ .\ .\ .\ .\ .\ .\ .\ (2)\\
c^2 & = a^2 + b^2 - 2ab\ \cos C\ .\ .\ .\ .\ .\ .\ .\ .\ .\ .\ (3)\\
\end{align*}\\
Upon adding (2) and (3), we get \begin{align*}
b^2 + c^2 & = a^2 + c^2 - 2ac\ \cos B + a^2 + b^2 - 2ab\ \cos C\\
2a^2 & = 2a(b \cos C + c \cos B)\\
a & = b \cos C + c \cos B
\end{align*}\\
\\
Upon adding (1) and (3), we get \begin{align*}
a^2 + c^2 & = b^2 + c^2 - 2bc\ \cos A + a^2 + b^2 - 2ab\ \cos C\\
2b^2 & = 2b(c \cos A + a \cos C)\\
b & = c \cos A + a \cos C
\end{align*}\\
\\
Upon adding (1) and (2), we get \begin{align*}
a^2 + b^2 & = b^2 + c^2 - 2bc\ \cos A + a^2 + c^2 - 2ac\ \cos B\\
2c^2 & = 2c(b \cos A + a \cos B)\\
c & = b \cos A + a \cos B
\end{align*}

Given that a woman on the observation deck, 1150 ft above the ground observes that
the angle formed by the lines of sight to two landmarks is $43^o$. She also
observes that the angles between the vertical and the line of sight to the
landmarks are $62^o$ and $54^o$ as shown in the below figure.\\
\\
In $\Delta$ABC, we have \begin{align*}
\cos 54^o & = \dfrac{\text{adj}}{\text{hyp}}\\
& = \dfrac{1150}{d}\\
d & = \dfrac{1150}{\cos 54^o}\\
& \approx 1956.5\ \text{ft}
\end{align*}\\
\\
Similarly, in $\Delta$ABD, we have \begin{align*}
\cos 62^o & = \dfrac{\text{adj}}{\text{hyp}}\\
& = \dfrac{1150}{c}\\
c & = \dfrac{1150}{\cos 62^o}\\
& \approx 2449.56\ \text{ft}
\end{align*}\\
\\
Now, using the law of cosines for the $\Delta$BCD, we have \begin{align*}
b^2 & = c^2 + d^2 - 2cd \cos \angle CBD\\
& = (2449.56)^2 + (1956.5)^2 - 2(2449.56)(1956.5) \cos 43^o\\
& = 6000344.1936 + 3827892.25 - 9585128.28(0.73135370161)\\
& = 9828236.4436 - 7010119.04798\\
& \approx 2818117.4\\
b & \approx \sqrt{2818117.4}\\
& \approx 1678.725\ \text{ft}
\end{align*}
Therefore, distance between the two landmarks is approximately 1679 ft.

In the given figure, since segment $AB$ is parallel to the $x$-axis, using
alternate angle theorem, we get $\angle$ ABO $= \theta$ too. We also have $OA = 1$
since radius of circle is one unit.\\
\\
Now, for the $\Delta$AOB, we have \begin{align*}
\cot ABO = \cot \theta & = \dfrac{\text{adj}}{\text{opp}}\\
& = \dfrac{AB}{OA}\\
\cot \theta & = \dfrac{a}{1}\\
\text{Thus}\ a & = \cot \theta\\
\\
\text{Also},\ \csc ABO = \csc \theta & = \dfrac{\text{hyp}}{\text{opp}}\\
& = \dfrac{OB}{OA}\\
\csc \theta & = \dfrac{b}{1}\\
\text{Thus}\ b & = \csc \theta\\
\end{align*}

Given that the angle of elevation from a distance of 1 km from the base of the CN
Tower in Toronto, Canada is $28.81^o$. Let $h$ be the height of the tower as shown
in the below figure.\\
\\
Then, from the figure, \begin{align*}
\tan 28.81^o & = \dfrac{\text{opp}}{\text{adj}}\\
0.54998195604 & = \dfrac{h}{1}\\
\text{Thus,}\ h & \approx 0.55\ \text{km}\ \text{or}\\
& \approx 550\ \text{m}
\end{align*}\\
Therefore, height of the tower is approximately 550 meters.

Given below is the figure of a regular hexagon that is inscribed in a circle of


radius $8$ m. Arc $AB$ subtends central angle $60^o$ as shown below.\\
\\
Now, using the law of cosines for the $\Delta$OAB, we have \begin{align*}
AB^2 & = OA^2 + OB^2 - 2(OA)(OB)\cos O\\
& = (8)^2 + (8)^2 - 2(8)(8)\cos 60^o\\
& = 128 - 128\bigg(\dfrac{1}{2}\bigg)\\
& = 128 - 64\\
AB^2 & = 64\\
AB & = \sqrt{64}\\
& = 8
\end{align*}\\
Now, in the figure, $AB$ is one of the side of the regular hexagon and so, the
perimeter of the regular hexagon will be equal to $$\text{Perimeter} = 6 \times AB
= 6 \times 8 = 48\ \text{m}$$

The given situation is redrawn below. From the figure, since $\angle$POR = $90^o$
and $\angle$QOR $ = \theta$, we have $\angle$POQ = $90^o - \theta$.\\
\\
Since radius of the circle is 2 units, we have $OQ = 2$ too.\\
\\
\\
Now using the law of cosines for the $\Delta$OQP, we have \begin{align*}
\cos (90-\theta) & = \dfrac{OQ^2 + OP^2 - QP^2}{2(OQ)(OP)}\\
\sin \theta & = \dfrac{2^2 + OP^2 - 8^2}{2(2)(OP)}\\
& = \dfrac{4 + OP^2 - 64}{4OP}\\
4OP \sin \theta & = OP^2 - 60\\
OP^2 - 4OP \sin \theta - 60 & = 0\\
\end{align*}
Using quadratic formula, we get \begin{align*}
OP & = \dfrac{4\sin \theta \pm \sqrt{(-4\sin \theta)^2 - 4(1)(-60)}}{2(1)}\\
& = \dfrac{4\sin \theta \pm \sqrt{16\sin^2 \theta + 240}}{2}\\
& = \dfrac{4\sin \theta \pm \sqrt{16(\sin^2 \theta + 15)}}{2}\\
& = \dfrac{4\sin \theta \pm 4 \sqrt{\sin^2 \theta + 15}}{2}\\
& = 2 \sin \theta \pm 2\sqrt{\sin^2 \theta + 15}
\end{align*}\\
Since $OP$ can not be negative, we get $$OP = 2 (\sin \theta + \sqrt{\sin^2 \theta
+ 15})$$

Given that the angle subtended by the full moon as viewed from the earth is
$0.518^o$ and the distance $AB$ from the earth to the moon is $236,900$ mi.\\
\\
Let $r$ be the radius of the moon as shown below. The angle $OAC$ will be half of
the angle subtended by moon, i.e. $$\angle OAC = \dfrac{0.518^o}{2} = 0.259^o$$\\
Now, for the $\Delta$OAC, we have \begin{align*}
\csc OAC = \dfrac{\text{hyp}}{\text{opp}} & = \dfrac{OA}{OC}\\
& = \dfrac{r + AB}{r}\\
\csc 0.259^o & = 1 + \dfrac{236,900}{r}\\
\dfrac{236,900}{r} & = \csc 0.259^o - 1\\
& = 221.22 - 1\\
& = 220.22\\
r & = \dfrac{236,900}{220.22}\\
& \approx 1075.74\ \text{mi}
\end{align*}\\
Therefore, radius of the moon is $1075.74$ miles approximately.

Given that a pilot measures the angles of depressions of two ships to be $40^o$ and
$52^o$ as shown in the below figure. The pilot is flying at an elevation of
$35,000$ ft.\\
\\
Now for the $\Delta$PDB, we have \begin{align*}
\cot 40^o & = \dfrac{\text{adj}}{\text{opp}}\\
1.19175359259 & = \dfrac{PD}{BD}\\
& = \dfrac{PD}{35,000}\\
PD & = 35,000 \times 1.19175359259\\
& \approx 41,711\ \text{ft}
\end{align*}\\
Now for the $\Delta$PCA, we have \begin{align*}
\cot 52^o & = \dfrac{\text{adj}}{\text{opp}}\\
0.7812856265 & = \dfrac{PC}{AC}\\
& = \dfrac{PC}{35,000}\\
PC & = 35,000 \times 0.7812856265\\
& \approx 27,345\ \text{ft}
\end{align*}\\
Therefore, the distance between the two ships is \begin{align*}
AB & = CD \\
& = PD - PC\\
& = 41,711 - 27,345\\
& = 14,366\ \text{ft}
\end{align*}
$$\sin 315^o$$\\
To find the value of any trigonometric function for any angle $\theta$, we first
find the reference angle $\overline{\theta}$ associated with the angle $\theta$.
The reference angle is the acute angle formed by the terminal side of the angle
$315^o$ and the $x$-axis.\\
\\
Since, the terminal side of this angle is in Quadrant IV, the reference angle will
be $$\overline{\theta} = 360^o - 315^o = 45^o$$
Since, the sign of sine function is negative in the Quadrant IV, the value of $\sin
315^o$ is $$\sin 315^o = -\sin 45^o = -\dfrac{1}{\sqrt{2}}$$

$$\csc \dfrac{9\pi}{4}$$\\
To find the value of any trigonometric function for any angle $\theta$, we first
find the reference angle $\overline{\theta}$ associated with the angle $\theta$.
The reference angle is the acute angle formed by the terminal side of the angle
$\dfrac{9\pi}{4}$ and the $x$-axis.\\
\\
\\
The above angle can be written as $$\dfrac{9\pi}{4} = 2\pi + \dfrac{\pi}{4}$$\\
\\
Since, the terminal side of this angle is in Quadrant I, the reference angle will
be the angle itself. Since, the sign of cosecant function is positive in the
Quadrant I, the value of $\csc \dfrac{9\pi}{4}$ is $$\csc \dfrac{9\pi}{4} = \csc
\dfrac{\pi}{4} = \sqrt{2}$$

$$\tan(-135^o)$$\\
To find the value of any trigonometric function for any angle $\theta$, we first
find the reference angle $\overline{\theta}$ associated with the angle $\theta$.
The reference angle is the acute angle formed by the terminal side of the angle $-
135^o$ and the $x$-axis.\\
\\
Since, the terminal side of this angle is in Quadrant III, the reference angle will
be $$\overline{\theta} = 180^o - 135^o = 45^o$$
Since, the sign of tangent function is positive in the Quadrant III, the value of
$\tan(-135^o)$ is $$\tan(-135^o) = \tan 45^o = 1$$

$$\cos \dfrac{5\pi}{6}$$\\
To find the value of any trigonometric function for any angle $\theta$, we first
find the reference angle $\overline{\theta}$ associated with the angle $\theta$.
The reference angle is the acute angle formed by the terminal side of the angle
$\dfrac{5\pi}{6}$ and the $x$-axis.\\
\\
Since, the terminal side of this angle is in Quadrant II, the reference angle will
be $$\overline{\theta} = \pi - \dfrac{5\pi}{6} = \dfrac{\pi}{6}$$ Since, the sign
of cosine function is negative in the Quadrant II, the value of $\cos \dfrac{5\pi}
{6}$ is $$\cos \dfrac{5\pi}{6} = -\cos \dfrac{\pi}{6} = -\dfrac{\sqrt{3}}{2}$$

$$\cot \bigg(-\dfrac{22\pi}{3}\bigg)$$\\
The angle $-\dfrac{22\pi}{3}$ is coterminal with $-\dfrac{4\pi}{3}$ and these
angles are in Quadrant II, and its reference angle is $$\overline{\theta} =
\dfrac{4\pi}{3} - \pi = \dfrac{\pi}{3}$$
Since the value of cotangent is negative in Quadrant II, we have $$\cot
\bigg(-\dfrac{22\pi}{3}\bigg) = -\cot \dfrac{\pi}{3} = -\dfrac{1}{\sqrt{3}}$$

$$\sin 405^o$$\\
The angle $405^o$ is coterminal with $45^o$ and the terminal side of this angle is
in Quadrant I, the reference angle will be $$\overline{\theta} = 45^o$$
Since, the sign of sine function is positive in the Quadrant I, the value of $\sin
405^o$ is $$\sin 405^o = \sin 45^o = \dfrac{1}{\sqrt{2}}$$

$$\cos 585^o$$\\
The angle $585^o$ is coterminal with $225^o$ and the terminal side of this angle is
in Quadrant III, the reference angle will be $$\overline{\theta} = 225 - 180^o =
45^o$$
Since, the sign of cosine function is negative in the Quadrant III, the value of
$\cos 585^o$ is $$\cos 585^o = -\cos 45^o = -\dfrac{1}{\sqrt{2}}$$

$$\sec \dfrac{22\pi}{3}$$\\
The angle $\dfrac{22\pi}{3}$ is coterminal with $\dfrac{4\pi}{3}$ and the terminal
side of this angle is in Quadrant III, the reference angle will be $
$\overline{\theta} = \dfrac{4\pi}{3} - \pi = \dfrac{\pi}{3}$$
Since, the sign of secant function is negative in the Quadrant III, the value of
$\sec \dfrac{22\pi}{3}$ is $$\sec \dfrac{22\pi}{3} = -\sec \dfrac{\pi}{3} = -2$$

$$\csc \dfrac{8\pi}{3}$$\\
The angle $\dfrac{8\pi}{3}$ is coterminal with $\dfrac{2\pi}{3}$ and the terminal
side of this angle is in Quadrant II, the reference angle will be $
$\overline{\theta} = \pi - \dfrac{2\pi}{3} = \dfrac{\pi}{3}$$
Since, the sign of cosecant function is positive in the Quadrant II, the value of
$\csc \dfrac{8\pi}{3}$ is $$\csc \dfrac{8\pi}{3} = \csc \dfrac{\pi}{3} = \dfrac{2}
{\sqrt{3}}$$

$$\sec \dfrac{13\pi}{6}$$\\
The angle $\dfrac{13\pi}{6}$ is coterminal with $\dfrac{\pi}{6}$ and the terminal
side of this angle is in Quadrant I, the reference angle will be $
$\overline{\theta} = \dfrac{\pi}{6}$$
Since, the sign of secant function is positive in the Quadrant I, the value of
$\sec \dfrac{13\pi}{6}$ is $$\sec \dfrac{13\pi}{6} = \sec \dfrac{\pi}{6} =
\dfrac{2}{\sqrt{3}}$$

$$\cot (-390^o)$$\\
The angle $-390^o$ is coterminal with $-30^o$ and the terminal side of this angle
is in Quadrant IV, the reference angle will be $$\overline{\theta} = 30^o$$
Since, the sign of cotangent function is negative in the Quadrant IV, the value of
$\cot (-390^o)$ is $$\cot (-390^o) = -\cot 30^o = -\sqrt{3}$$
$$\tan \dfrac{23\pi}{4}$$\\
The angle $\dfrac{23\pi}{4}$ is coterminal with $\dfrac{7\pi}{4}$ and the terminal
side of this angle is in Quadrant IV, the reference angle will be $
$\overline{\theta} = 2\pi - \dfrac{7\pi}{4} = \dfrac{\pi}{4}$$
Since, the sign of tangent function is negative in the Quadrant IV, the value of
$\tan \dfrac{23\pi}{4}$ is $$\tan \dfrac{23\pi}{4} = -\tan \dfrac{\pi}{4} = -1$$

Let's find the point of intersection of the line $y - \sqrt{3}x + 1 = 0$ and the
$x$-axis. On $x$-axis, when $y = 0$, we get $$x = \dfrac{1}{\sqrt{3}}$$ So, the
point on $x$-axis is $\bigg(\dfrac{1}{\sqrt{3}},0\bigg)$.\\
\\
\\
Let us find any other point on the line in the first quadrant. When $x =
\sqrt{3}$, $$y - \sqrt{3}(\sqrt{3}) + 1 = 0\ \Rightarrow y = 2$$ Thus, we can
consider the other point as $(\sqrt{3},2)$ on the line as shown in the below
figure. $\theta$ is the acute angle formed by the line and the $x$-axis. \\
\\
For the below figure, we have $$AB = 2,\ \text{and}\ BC = \dfrac{2}{\sqrt{3}}$$
Now, from the figure, we have \begin{align*}
\tan \theta = \dfrac{\text{opp}}{\text{adj}} & = \dfrac{AB}{BC} = \dfrac{2}
{\dfrac{2}{\sqrt{3}}}\\
\tan \theta & = \sqrt{3}\\
\theta & = \tan^{-1} (\sqrt{3})\\
& = 60^o\\
\end{align*}
Therefore, the acute angle made by the line with the $x$-axis is equal to $60^o$

We know that the slope of the parallel lines are equal, i.e. a terminal side in
Quadrant III parallel to the line $4y - 2x -1 = 0$ will have the same trigonometric
ratios of the angle $\theta$ in standard position as any other point on the line
$4y - 2x - 1 = 0$ in Quadrant III.\\
\\
A straight line can be written in its slope-intercept form $$y = mx + c$$ where $m$
is the slope of the line and $m = \tan \theta$.\\
\\
The given line can be written in its slope-intercept form as $$4y = 2x + 1\
\Rightarrow \boxed{y = \dfrac{1}{2}x + \dfrac{1}{4}}$$ Thus, we get $m = \tan
\theta = \dfrac{1}{2}$.\\
\\
We sketch a triangle so that $\tan \theta = \dfrac{1}{2}$. Taking into account the
fact that $\theta$ is in Quadrant III, we get $$\sin \theta = -\dfrac{1}{\sqrt{5}}\
\ \ \ \ \ \cos \theta = -\dfrac{2}{\sqrt{5}}\ \ \ \ \ \ \ \tan \theta = \dfrac{1}
{2}$$
$$\csc \theta = -\dfrac{\sqrt{5}}{1}\ \ \ \ \ \ \sec \theta = -\dfrac{\sqrt{5}}{2}\
\ \ \ \ \ \ \cot \theta = 2$$

Given $\tan \theta = -\dfrac{1}{2}$ for $\theta$ in Quadrant II.\\


\\
We sketch a triangle so that $\tan \theta = -\dfrac{1}{2}$. Taking into account the
fact that $\theta$ is in Quadrant II, we can say that $\sin \theta$ must be
positive and $\cos \theta$ must be negative. So, using the sketch of below triangle
$$\sin \theta = \dfrac{1}{\sqrt{5}}\ \text{and}\ \cos \theta = -\dfrac{2}{\sqrt{5}}
$$
Therefore, \begin{align*}
\sin \theta + \cos \theta & = \dfrac{1}{\sqrt{5}} + \bigg(-\dfrac{2}
{\sqrt{5}}\bigg)\\
& = -\dfrac{1}{\sqrt{5}}
\end{align*}

Given $\sin \theta = \dfrac{1}{2}$ and $\theta$ lies in the Quadrant I. \\


\\
Using the identity $\sin^2 \theta + cos^2 \theta = 1$, we get $\cos \theta = \pm
\sqrt{1 - \sin^2 \theta}$. Now, $\cos \theta$ is positive in the Quadrant I. So, $
$\cos \theta = \sqrt{1 - \sin^2 \theta} = \sqrt{1 - \dfrac{1}{4}} =
\dfrac{\sqrt{3}}{2}$$\\
Therefore, \begin{align*}
\tan \theta + \sec \theta & = \dfrac{\sin \theta}{\cos \theta} + \dfrac{1}{\cos
\theta} = \dfrac{1 + \sin \theta}{\cos \theta}\\
\\
& = \dfrac{1 + \dfrac{1}{2}}{\dfrac{\sqrt{3}}{2}} = \dfrac{\dfrac{3}{2}}
{\dfrac{\sqrt{3}}{2}}\\
\\
& = \sqrt{3}
\end{align*}

Given $\tan \theta = -1$ and we need to find out the value of the trigonometric
expression $\sin^2 \theta + \cos^2 \theta$.\\
\\
But we already know the identity that $$\sin^2 \theta + \cos^2 \theta = 1$$
Therefore, no matter what the value of $\tan \theta$ is, we always get $$\sin^2
\theta + \cos^2 \theta = 1$$

Given $\cos \theta = -\dfrac{\sqrt{3}}{2}$ and $\pi/2 < \theta < \pi$, i.e.
$\theta$ lies in the Quadrant III. \\
\\
The angle in the interval $[\pi/2,\pi]$ whose cosine is $-\dfrac{\sqrt{3}}{2}$ is
$\dfrac{5\pi}{6}$. Thus, $$\theta = \dfrac{5\pi}{6}\ \text{and}\ 2\theta =
\dfrac{5\pi}{3}$$\\
Now, we need to find the value of $\sin 2\theta$, i.e. $\sin \dfrac{5\pi}{3}$.
Since, the terminal side of this angle is in Quadrant IV, the reference angle will
be $$\overline{\theta} = 2 \pi - \dfrac{5\pi}{3} = \dfrac{\pi}{3}$$ Since, the sign
of sine function is negative in the Quadrant IV, the value of $\sin 2\theta$ is $
$\sin 2\theta = \sin \dfrac{5\pi}{3} = -\sin \dfrac{\pi}{3} = -\dfrac{\sqrt{3}}{2}$
$

Let one ship that leaves port $C$ and travels at $20$ mi/h in a direction N $32^o$
E reach point $A$ after 2 hours. Let another ship that leaves port $C$ and travels
at $28$ mi/h in a direction S $42^o$ E reach point $B$ after 2 hours as shown in
the below figure.\\
\\
Then the distance $b$ that one ship travels will be equal to $$b = 20 \times 2 =
40\ \text{mi}$$
And the distance $a$ that the other ship travels will be equal to $$a = 28 \times 2
= 56\ \text{mi}$$\\
Since, $y$-axis is a straight line, $\angle$ACB will be equal to $$\angle ACB =
180^o - (32^o + 42^o) = 180^o - 74^o = 106^o$$\\
Now, using the law of cosines, we get \begin{align*}
c^2 & = a^2 + b^2 - 2ab \cos C\\
& = (56)^2 + (40)^2 - 2(56)(40)\cos 106^o\\
& = 3136 + 1600 - 4480(-0.27563735581)\\
& = 4736 + 1234.855\\
c^2 & = 5970.855\\
c & = \sqrt{5970.855}\\
& = 77.27
\end{align*}\\
Therefore, the two ships are 77.27 miles apart after 2 hours.

Given that the angle of elevation from a point $A$ to the top of a building from
the ground is $24.1^o$ and from a point $B$, which is 600 ft closer to the
building, the angle of elevation is $30.2^o$ as shown in the figure below. \\
\\
Let $BD$ be the distance from the building to point where elevation is $30.2^o$ as
shown below and $CD$ be the height of the building. Then, for the smaller triangle,
we have \begin{align*}
\tan 30.2^o & = \dfrac{CD}{BD}\\
BD & = \dfrac{CD}{\tan 30.2^o}\\
\end{align*}
Similarly, for the larger triangle, we have \begin{align*}
\tan 24.1^o & = \dfrac{CD}{600 + BD}\\
600 + BD & = \dfrac{CD}{\tan 24.1^o}\\
600 + \dfrac{CD}{\tan 30.2^o} & = \dfrac{CD}{\tan 24.1^o}\\
CD\bigg( \dfrac{1}{\tan 24.1^o} - \dfrac{1}{\tan 30.2^o}\bigg) & = 600\\
CD(\cot 24.1^o - \cot 30.2^o) & = 600\\
CD & = \dfrac{600}{0.517356}\\
& \approx 1159.74\ \text{ft}
\end{align*}\\
Therefore, the height of the building is 1160 feet approximately.\\

Given that a boat is cruising the ocean off a straight shoreline. Points $A$ and
$B$ are 120 mi apart on the shore, as shown below. It is found that $\angle$A =
$42.3^o$ and $\angle$B = $68.9^o$. Let $CD$ is the shortest distance from the boat
to the shore. \\
\\
Now, in the $\Delta$BCD, we have \begin{align*}
\tan 68.9^o & = \dfrac{CD}{BD}\\
BD & = \dfrac{CD}{\tan 68.9^o}\ .\ .\ .\ .\ .\ .\ (1)
\end{align*}\\
\\
Now, in the $\Delta$ACD, we have \begin{align*}
\tan 42.3^o & = \dfrac{CD}{AD}\\
AD & = \dfrac{CD}{\tan 42.3^o}\ .\ .\ .\ .\ .\ .\ .\ (2)
\end{align*}\\
Upon adding (1) and (2), we get \begin{align*}
BD + AD & = \dfrac{CD}{\tan 68.9^o} + \dfrac{CD}{\tan 42.3^o}\\
120 & = CD\bigg(\dfrac{1}{\tan 68.9^o} + \dfrac{1}{\tan 42.3^o}\bigg)\\
& = CD(\cot 68.9^o + \cot 42.3^o)\\
& = CD(1.48485353643)\\
CD & = \dfrac{120}{1.48485353643}\\
& \approx 80.8\ \text{mi}
\end{align*}\\
Therefore, the shortest distance from the boat to the shore is approximately 80.8
miles.

The given figure labelled and redrawn again as shown below.\\


\\
In $\Delta$ABD, we have \begin{align*}
\tan 20^o & = \dfrac{BD}{AB}\\
0.36397023426 & = \dfrac{BD}{50}\\
\text{Thus}\ BD & = 50 \times 0.36397023426\\
& \approx 18.2
\end{align*}\\
\\
In $\Delta$ABC, we have \begin{align*}
\tan 28^o & = \dfrac{BC}{AB}\\
0.53170943166 & = \dfrac{x + BD}{50}\\
\text{Thus}\ x + BD & = 50 \times 0.53170943166\\
x + 18.2 & \approx 26.6\\
x & \approx 8.4
\end{align*}

We know that the length $s$ of an arc when subtends a central angle of $\theta$
radians is equal to $$s = r \theta$$\\
Let's multiply the angle $\theta$ by $\dfrac{\pi}{180^o}$ to convert it into
radians. $$\theta = 72^o \times \dfrac{\pi}{180^o} = \dfrac{2\pi}{5}\
\text{radians}$$\\
Then, the length of the arc $BC$ in the given figure will be \begin{align*}
BC & = 10 \times \dfrac{2\pi}{5}\\
& = 4\pi\\
& \approx 12.56637\ \text{m}
\end{align*}\\
Now, using law of cosines for the $\Delta$ABC, we get \begin{align*}
BC^2 & = AB^2 + AC^2 - 2(AB)(AC)\cos A\\
& = (10)^2 + (10)^2 - 2(10)(10)\cos 72^o\\
& = 100 + 100 - 200(0.309)\\
& = 200 - 61.8\\
BC^2 & = 138.2\\
BC & = \sqrt{138.2}\\
& \approx 11.756\ \text{m}
\end{align*}\\
Therefore, the perimeter of the shaded region is \begin{align*}
\text{Perimeter}\ & = BC + \widehat{BC}\\
& = 11.756 + 12.56637\\
& \approx 24.32\ \text{m}
\end{align*}

Given that the angle of elevation from two points $A$ and $B$ where two wires
tether a balloon to the ground to the balloon are $75^o$ and $85^o$ respectively as
shown in the figure below. The distance between the points $A$ and $B$ is 100 feet.
\\
\\
Let $CD = h$ be the height of the balloon above the ground. Then, for the smaller
triangle, we have \begin{align*}
\tan 85^o & = \dfrac{h}{BC}\\
BC & = \dfrac{h}{\tan 85^o}\\
\end{align*}
Similarly, for the larger triangle, we have \begin{align*}
\tan 75^o & = \dfrac{h}{100 + BC}\\
100 + BC & = \dfrac{h}{\tan 75^o}\\
100 + \dfrac{h}{\tan 85^o} & = \dfrac{h}{\tan 75^o}\\
h\bigg( \dfrac{1}{\tan 75^o} - \dfrac{1}{\tan 85^o}\bigg) & = 100\\
h(\cot 75^o - \cot 85^o) & = 100\\
h & = \dfrac{100}{0.1804605289}\\
& \approx 554.14\ \text{ft}
\end{align*}\\
Therefore, height of the balloon is 554 feet approximately above the ground.\\

The distance between City Hall and the first bridge is given to be $0.86$ miles. \\
\\
Now, in the triangle formed by the City Hall, Church and the first bridge, the
angle at the first bridge can be found out using the angle sum property, i.e. if
$\theta$ is the angle at the vertex of the triangle at the first bridge, then $
$\theta = 180^o - (25^o + 30^o) = 180^o - 55^o = 125^o$$\\
The distance $x$ between the Church and City Hall can be calculated by applying the
Law of Sines to the triangle with the vertices at City Hall, Church and the first
bridge. \begin{align*}
\dfrac{x}{\sin 125^o} & = \dfrac{0.86}{\sin 30^o}\\
x & = 0.86 \times \dfrac{\sin 125^o}{\sin 30^o}\\
& = 0.86 \times 1.63830408858\\
& \approx 1.4\ \text{mi}
\end{align*}
Therefore, the distance between the Church and the city hall is 1.4 miles
approximately.

We already know that the distance between the bank and the cliff is $1.55$
miles. \\
\\
Now, in the triangle formed by the bank, cliff and the second bridge, the angle at
the second bridge can be found out using the angle sum property, i.e. if $\theta$
is the angle at the vertex of the triangle at the second bridge, then $$\theta =
180^o - (50^o + 60^o) = 180^o - 110^o = 70^o$$\\
The distance $x$ between the cliff and the second bridge can be calculated by
applying the Law of Sines to the triangle with the vertices at the bank, cliff and
the second bridge. \begin{align*}
\dfrac{x}{\sin 50^o} & = \dfrac{1.55}{\sin 70^o}\\
x & = 1.55 \times \dfrac{\sin 50^o}{\sin 70^o}\\
& = 1.55 \times 0.81520746909\\
& \approx 1.26\ \text{mi}
\end{align*}
So, the distance between the cliff and the second bridge is 1.26 miles
approximately.\\
\\
Now, in the triangle formed by the school, cliff and the second bridge, the angle
at the second bridge can be found out using the angle sum property, i.e. if
$\theta$ is the angle at the vertex of the triangle at the second bridge, then $
$\theta = 180^o - (80^o + 55^o) = 180^o - 135^o = 45^o$$\\
The distance $y$ between the cliff and the school can be calculated by applying the
Law of Sines to the triangle with the vertices at the school, cliff and the second
bridge. \begin{align*}
\dfrac{y}{\sin 45^o} & = \dfrac{1.26}{\sin 55^o}\\
y & = 1.26 \times \dfrac{\sin 45^o}{\sin 55^o}\\
& = 1.26 \times 0.86321799001\\
& \approx 1.09\ \text{mi}
\end{align*}
So, the distance between the cliff and the school is 1.09 miles approximately.

Now, in the triangle formed by the school, cliff and the fire hall, the angle at
the fire hall can be found out using the angle sum property, i.e. if $\theta$ is
the angle at the vertex of the triangle at the fire hall, then $$\theta = 180^o -
(80^o + 45^o) = 180^o - 125^o = 55^o$$\\
The distance $z$ between the fire hall and the school can be calculated by applying
the Law of Sines to the triangle with the vertices at the school, cliff and the
fire hall. \begin{align*}
\dfrac{z}{\sin 80^o} & = \dfrac{1.09}{\sin 55^o}\\
z & = 1.09 \times \dfrac{\sin 80^o}{\sin 55^o}\\
& = 1.09 \times 1.20222827969\\
& \approx 1.31\ \text{mi}
\end{align*}
Therefore, the distance between the fire hall and the school is 1.31 miles
approximately.

In the given figure, the measure of $\angle$A in the triangle ACD can be found out
using the angle sum property, i.e. $$\angle A = 180^o - (45^o + 20^o + 40^o) =
180^o - 105^o = 75^o$$\\
The distance $AC$ can be calculated by applying the Law of Sines to the triangle
$ACD$. \begin{align*}
\dfrac{AC}{\sin 45^o} & = \dfrac{20}{\sin 75^o}\\
AC & = 20 \times \dfrac{\sin 45^o}{\sin 75^o}\\
& = 20 \times 0.73205080756\\
& \approx 14.64\ \text{m}
\end{align*}
So, the distance $AC$ is 14.64 m approximately.\\
\\
\\
In the given figure, the measure of $\angle$B in the triangle BCD can be found out
using the angle sum property, i.e. $$\angle B = 180^o - (20^o + 45^o + 50^o) =
180^o - 115^o = 65^o$$\\
The distance $BC$ can be calculated by applying the Law of Sines to the triangle
$BCD$. \begin{align*}
\dfrac{BC}{\sin 95^o} & = \dfrac{20}{\sin 65^o}\\
BC & = 20 \times \dfrac{\sin 95^o}{\sin 65^o}\\
& = 20 \times 1.09917923286\\
& \approx 21.98\ \text{m}
\end{align*}
So, the distance $BC$ is 21.98 m approximately.
Now, the distance between $A$ and $B$ can be calculated using the Law of Cosines
for the triangle $ABC$.\begin{align*}
AB^2 & = AC^2 + BC^2 - 2(AC)(BC) \cos C\\
& = (14.64)^2 + (21.98)^2 - 2(14.64)(21.98) \cos 40^o\\
& = 214.3296 + 483.1204 - 643.5744(0.76604444311)\\
& = 697.45 - 493 \\
AB^2 & = 204.45\\
AB & = \sqrt{204.45}\\
& \approx 14.3\ \text{m}
\end{align*}
Therefore, the distance between $A$ and $B$ is 14.3 m approximately.

In the given below figure, using the angle sum property for the $\Delta$ACD, we get
$$\angle C = 180^o - (69.4^o + 51.6^o) = 180^o - 121^o = 59^o$$\\
Now, using the Law of Sines for the $\Delta$ACD, we get \begin{align*}
\dfrac{AC}{\sin 51.6^o} & = \dfrac{200}{\sin 59^o}\\
AC & = 200 \times \dfrac{\sin 51.6^o}{\sin 59^o}\\
& = 200 \times 0.91428296049\\
& \approx 182.86\ \text{m}
\end{align*}\\
\\
Now, in the $\Delta$ABC, we have \begin{align*}
\tan 33.1^o & = \dfrac{BC}{AC}\\
BC & = AC \times \tan 33.1^o\\
& = 182.86 \times 0.65189179873\\
& \approx 119.2\ \text{m}
\end{align*}
Therefore, the height of the cliff is $119.2$ m approximately.

The angles $\alpha$ and $\beta$ are the angles of elevation from points $A$ and $B$
to the top of a mountain of height $h$ as shown in the figure below. $d$ is the
distance between the points $A$ and $B$. \\
\\
Let $a$ be the distance between the mountain from the point $B$. Then, for the
smaller triangle, we have \begin{align*}
\tan \beta & = \dfrac{h}{a}\\
a & = \dfrac{h}{\tan \beta}\\
\end{align*}
Similarly, for the larger triangle, we have \begin{align*}
\tan \alpha & = \dfrac{h}{d + a}\\
d + a & = \dfrac{h}{\tan \alpha}\\
d + \dfrac{h}{\tan \beta} & = \dfrac{h}{\tan \alpha}\\
h\bigg( \dfrac{1}{\tan \alpha} - \dfrac{1}{\tan \beta}\bigg) & = d\\
h(\cot \alpha - \cot \beta) & = d\\
\text{Thus,}\ h & = \dfrac{d}{\cot \alpha - \cot \beta}\\
\end{align*}

Using the angle sum property for $\Delta$BCD, we have $$\angle BCD = 180^o - (\beta
+ 90^o) = 90^o - \beta$$\\
Similarly, using the angle sum property for $\Delta$ACD, we have $$\angle ACD =
180^o - (\alpha + 90^o) = 90^o - \alpha$$\\
Now, for the $\Delta$ABC, we have \begin{align*}
\angle ACB & = \angle ACD - \angle BCD\\
& = (90^o - \alpha) - (90^o - \beta)\\
& = \beta - \alpha
\end{align*}\\
\\
Now, using the Law of Sines for the $\Delta$ABC, we get \begin{align*}
\dfrac{AB}{\sin ACB} & = \dfrac{BC}{\sin BAC}\\
\dfrac{d}{\sin (\beta - \alpha)} & = \dfrac{BC}{\sin \alpha}\\
BC & = d\dfrac{\sin \alpha}{\sin (\beta - \alpha)}
\end{align*}\\
\\
Now, for the $\Delta$BCD, we have \begin{align*}
\sin \beta & = \dfrac{\text{opp}}{\text{hyp}}\\
& = \dfrac{h}{BC}\\
h & = BC \sin \beta\\
& = d\dfrac{\sin \alpha \sin \beta}{\sin (\beta - \alpha)}
\end{align*}

If $\alpha = 25^o$, $\beta = 29^o$ and $d = 800$ ft, then height of the mountain
using the first formula is \begin{align*}
h & = \dfrac{d}{\cot \alpha - \cot \beta}\\
& = \dfrac{800}{\cot 25^o - \cot 29^o}\\
& = \dfrac{800}{0.34045916523}\\
& \approx 2349.77\ \text{ft}
\end{align*}\\
\\
If $\alpha = 25^o$, $\beta = 29^o$ and $d = 800$ ft, then height of the mountain
using the second formula is \begin{align*}
h & = d\dfrac{\sin \alpha \sin \beta}{\sin (\beta - \alpha)}\\
& = 800 \dfrac{\sin 25^o \sin 29^o}{\sin (29^o - 25^o)}\\
& = 800 \dfrac{(0.42261826174) (0.48480962024)}{\sin (4^o)}\\
& = 800 \bigg(\dfrac{0.20488939898}{0.06975647374}\bigg)\\
& = 800(2.93720981)\\
& \approx 2349.77\ \text{ft}
\end{align*}
Therefore, we see that we get the same answer from each formula.

Let $A$ and $B$ be the two landmarks at the base of the mountain where angle of
depression from the top of the mountain of height $2340$ ft are $42^o$ and $39^o$
respectively as shown in the below figure. \\
\\
Now, for the $\Delta$BCD, we get \begin{align*}
\sin 39^o & = \dfrac{CD}{BC}\\
BC & = \dfrac{CD}{\sin 39^o}\\
& = \dfrac{2340}{0.62932039105}\\
& \approx 3718.3\ \text{ft}
\end{align*}\\
Similarly, for the $\Delta$ACD, we get \begin{align*}
\sin 42^o & = \dfrac{CD}{AC}\\
AC & = \dfrac{CD}{\sin 42^o}\\
& = \dfrac{2340}{0.66913060635}\\
& \approx 3497\ \text{ft}
\end{align*}\\
Now, using the Law of Cosines for the $\Delta$ABC, we get \begin{align*}
AB^2 & = AC^2 + BC^2 - 2(AC)(BC)\cos C\\
& = (3497)^2 + (3718.3)^2 - 2(3497)(3718.3) \cos 68^o\\
& = 12,229,009 + 13,825,754.89 - 26,005,790.2(0.37460659341)\\
& = 26,054,763.89 - 9,741,940.47576\\
AB^2 & = 16,312,823.4142\\
AB & = \sqrt{16,312,823.4142}\\
& \approx 4039\ \text{ft}
\end{align*}
Therefore, distance between the two landmarks is 4039 ft approximately.

The measurements of the lots are labelled and redrawn as shown in the figure below.
In the below figure, the missing angles measure \begin{align*}
\angle ACB & = 180^o - (60^o + 91^o) = 180^o - 151^o = 29^o\\
\angle BCD & = 180^o - (32^o + 87^o) = 180^o - 119^o = 61^o\\
\angle BED & = 180^o - (41^o + 88^o) = 180^o - 129^o = 51^o\\
\angle BEF & = 180^o - (50^o + 92^o) = 180^o - 142^o = 38^o\\
\end{align*}
All the measurements can be calculated using the Law of Sines as follows :
\begin{align*}
\dfrac{b}{\sin 91^o} & = \dfrac{150}{\sin 60^o}\\
b & = 150\bigg(\dfrac{\sin 91^o}{\sin 60^o}\bigg)\\
& = 150(1.15452467189)\\
& \approx 173.2\ \text{ft}\\
\\
\dfrac{a}{\sin 29^o} & = \dfrac{150}{\sin 60^o}\\
a & = 150\bigg(\dfrac{\sin 29^o}{\sin 60^o}\bigg)\\
& = 150(0.55980992951)\\
& \approx 84\ \text{ft}
\end{align*}\\

\begin{align*}
\dfrac{c}{\sin 32^o} & = \dfrac{b}{\sin 87^o}\\
c & = 173.2\bigg(\dfrac{\sin 32^o}{\sin 87^o}\bigg)\\
& = 173.2(0.53064649681)\\
& \approx 91.9\ \text{ft}\\
\\
\dfrac{d}{\sin 61^o} & = \dfrac{b}{\sin 87^o}\\
d & = 173.2\bigg(\dfrac{\sin 61^o}{\sin 87^o}\bigg)\\
& = 173.2(0.87581998799)\\
& \approx 151.7\ \text{ft}\\
\\
\dfrac{e}{\sin 41^o} & = \dfrac{d}{\sin 51^o}\\
e & = 151.7\bigg(\dfrac{\sin 41^o}{\sin 51^o}\bigg)\\
& = 151.7(0.84419023134)\\
& \approx 128\ \text{ft}
\end{align*}

\begin{align*}
\dfrac{f}{\sin 88^o} & = \dfrac{e}{\sin 41^o}\\
f & = 128\bigg(\dfrac{\sin 88^o}{\sin 41^o}\bigg)\\
& = 128(1.52332455291)\\
& \approx 195\ \text{ft}\\
\\
\dfrac{g}{\sin 50^o} & = \dfrac{f}{\sin 92^o}\\
g & = 195\bigg(\dfrac{\sin 50^o}{\sin 92^o}\bigg)\\
& = 195(0.76651138114)\\
& \approx 149.5\ \text{ft}\\
\\
\dfrac{h}{\sin 38^o} & = \dfrac{f}{\sin 92^o}\\
h & = 195\bigg(\dfrac{\sin 38^o}{\sin 92^o}\bigg)\\
& = 195(0.61603674826)\\
& \approx 120.2\ \text{ft}
\end{align*}\\
The accurate map of the two lots is shown below.

$$\dfrac{\sin t}{1 - \cos t} - \csc t$$\\


\begin{align*}
\dfrac{\sin t}{1 - \cos t} - \csc t & = \dfrac{\sin t}{1 - \cos t} - \dfrac{1}{\sin
t}\ \ \ \ \ \ \ \ \ \ \ [\ \text{Reciprocal identity}\ ]\\
\\
& = \dfrac{\sin^2 t - (1 - \cos t)}{\sin t (1 - \cos t)}\ \ \ \ \ \ \ \ [\
\text{Common denominator}\ ]\\
\\
& = \dfrac{\cos t - (1 - \sin^2 t)}{\sin t (1 - \cos t)}\\
\\
& = \dfrac{\cos t - \cos^2 t}{\sin t (1 - \cos t)}\ \ \ \ \ \ \ \ \ \ \ \ \ [\
\text{Pythagorean identity}\ ]\\
\\
& = \dfrac{\cos t(1 - \cos t)}{\sin t (1 - \cos t)}\ \ \ \ \ \ \ \ \ \ \ \ [\
\text{Taking common factor}\ ]\\
\\
& = \dfrac{\cos t}{\sin t }\ \ \ \ \ \ \ \ \ \ \ \ \ \ \ \ \ \ \ \ \ \ \ \ \ [\
\text{Cancel common factor}\ ]\\
\\
& = \cot t\ \ \ \ \ \ \ \ \ \ \ \ \ \ \ \ \ \ \ \ \ \ \ \ \ \ [\ \text{Reciprocal
identity}\ ]
\end{align*}

$$\dfrac{\cot A - 1}{1 + \tan(-A)}$$\\


\begin{align*}
\dfrac{\cot A - 1}{1 + \tan(-A)} & = \dfrac{\cot A - 1}{1 - \tan
A}\ \ \ \ \ \ \ \ \ \ \ \ \ \ \ \ \ \ \ \ \ \ [\ \text{Odd identity}\ ]\\
\\
& = \dfrac{\dfrac{\cos A}{\sin A} - 1}{1 - \dfrac{\sin A}{\cos A}}\ \ \ \ \ \ \ \ \
\ \ \ \ \ \ \ \ \ \ \ \ \ [\ \text{Reciprocal identity}\ ]\\
\\
& = \dfrac{\dfrac{\cos A - \sin A}{\sin A}}{\dfrac{\cos A - \sin A}{\cos A}}\ \ \ \
\ \ \ \ \ \ \ \ \ \ \ \ \ [\ \text{Common denominator}\ ]\\
\\
& = \dfrac{(\cos A - \sin A)\cos A}{(\cos A - \sin A)\sin A}\ \ \ \ \ \ \ \ [\
\text{Simplify}\ ]\\
\\
& = \dfrac{\cos A}{\sin A}\ \ \ \ \ \ \ \ \ \ \ \ \ \ \ \ \ \ \ \ \ \ \ \ \ \ \
[\ \text{Cancel common factor}\ ]\\
\\
& = \cot A\ \ \ \ \ \ \ \ \ \ \ \ \ \ \ \ \ \ \ \ \ \ \ \ \ \ \ \ [\
\text{Reciprocal identity}\ ]\\
\end{align*}

$$\dfrac{\tan y}{\csc y} = \dfrac{1}{\cos y} - \dfrac{1}{\sec y}$$\\


Finding a common denominator and combining the fractions on the right-hand side of
this equation, we get \begin{align*}
\text{RHS} & = \dfrac{1}{\cos y} - \dfrac{1}{\sec y}\\
\\
& = \dfrac{1}{\cos y} - \cos y\ \ \ \ \ \ \ \ \ \ \ \ \ \ \ \ \ [\ \text{Reciprocal
identity}\ ]\\
\\
& = \dfrac{1 - \cos^2 y}{\cos y}\ \ \ \ \ \ \ \ \ \ \ \ \ \ \ \ \ \ \ \ \ [\
\text{Common denominator}\ ]\\
\\
& = \dfrac{\sin^2 y}{\cos y}\ \ \ \ \ \ \ \ \ \ \ \ \ \ \ \ \ \ \ \ \ \ \ \ \ \
[\ \text{Pythagorean identity}\ ]\\
\\
& = \bigg(\dfrac{\sin y}{\cos y}\bigg)\sin y\ \ \ \ \ \ \ \ \ \ \ \ \ \ \ \ \ [\
\text{Factor}\ ]\\
\\
& = \dfrac{\tan y}{\csc y} = \text{LHS} \ \ \ \ \ \ \ \ \ \ \ \ \ \ \ \ [\
\text{Reciprocal identity}\ ]\\
\end{align*}

$$\dfrac{1- \cos \alpha}{\sin \alpha} = \dfrac{\sin \alpha}{1 + \cos \alpha}$$\\


We start with the left-hand side and multiply the numerator and denominator by $1 +
\cos \alpha$ \begin{align*}
\text{LHS} & = \dfrac{1- \cos \alpha}{\sin \alpha}\\
\\
& = \dfrac{1- \cos \alpha}{\sin \alpha} \cdot \dfrac{1 + \cos \alpha}{1+ \cos
\alpha}\\
\\
& = \dfrac{1 - \cos^2 \alpha}{\sin \alpha(1 + \cos \alpha)}\ \ \ \ \ \ \ \ \ \ \ [\
\text{Expand Numerator}\ ]\\
\\
& = \dfrac{\sin^2 \alpha}{\sin \alpha(1 + \cos \alpha)}\ \ \ \ \ \ \ \ \ \ \ [\
\text{Pythagorean identity}\ ]\\
\\
& = \dfrac{\sin \alpha}{1 + \cos \alpha} = \text{RHS}\\
\end{align*}

$$\dfrac{\sec x}{\sec x - \tan x} = \sec x(\sec x + \tan x)$$\\


We start with the left-hand side and multiply the numerator and denominator by
$\sec x + \tan x$ \begin{align*}
\text{LHS} & = \dfrac{\sec x}{\sec x - \tan x}\\
\\
& = \dfrac{\sec x}{\sec x - \tan x} \cdot \dfrac{\sec x + \tan x}{\sec x + \tan
x}\\
\\
& = \dfrac{\sec x(\sec x + \tan x)}{\sec^2 x - \tan^2 x}\ \ \ \ \ \ \ \ \ \ \ [\
\text{Expand Denominator}\ ]\\
\\
& = \dfrac{\sec x(\sec x + \tan x)}{1}\ \ \ \ \ \ \ \ \ \ \ [\ \text{Pythagorean
identity}\ ]\\
\\
& = \sec x(\sec x + \tan x) = \text{RHS}\\
\end{align*}

$$\dfrac{1 - \sin x}{1 + \sin x} = (\sec x - \tan x)^2$$\\


We prove the identity by changing each side separately into the same expression.\\
\begin{align*}
\text{LHS} & = \dfrac{1 - \sin x}{1 + \sin x}\\
& = \dfrac{1 - \sin x}{1 + \sin x} \cdot \dfrac{1 - \sin x}{1 - \sin x}\\
& = \dfrac{(1 - \sin x)^2}{1 - \sin^2 x}\\
& = \dfrac{(1 - \sin x)^2}{\cos^2 x}\\
\\
\text{RHS} & = (\sec x - \tan x)^2\\
& = \bigg(\dfrac{1}{\cos x} - \dfrac{\sin x}{\cos x}\bigg)^2\\
& = \bigg(\dfrac{1 - \sin x}{\cos x}\bigg)^2\\
& = \dfrac{(1 - \sin x)^2}{\cos^2 x}\\
\end{align*}
It follows that LHS = RHS, so the equation is an identity.

$$\dfrac{\sec u - 1}{\sec u + 1} = \dfrac{\tan u - \sin u}{\tan u + \sin u}$$\\


We prove the identity by changing each side separately into the same expression.\\
\begin{align*}
\text{LHS} & = \dfrac{\sec u - 1}{\sec u + 1} = \dfrac{\dfrac{1}{\cos u} - 1}
{\dfrac{1}{\cos u} + 1}\\
& = \dfrac{\dfrac{1 - \cos u}{\cos u}}{\dfrac{1 + \cos u}{\cos u}} = \dfrac{(1
- \cos u) \cos u}{(1 + \cos u) \cos u}\\
& = \dfrac{1 - \cos u}{1 + \cos u}\\
\\
\text{RHS} & = \dfrac{\tan u - \sin u}{\tan u + \sin u} = \dfrac{\dfrac{\sin u}
{\cos u} - \sin u}{\dfrac{\sin u}{\cos u} + \sin u}\\
& = \dfrac{\sin u\bigg(\dfrac{1}{\cos u} - 1\bigg)}{\sin u\bigg(\dfrac{1}{\cos u} +
1\bigg)} = \dfrac{\dfrac{1 - \cos u}{\cos u}}{\dfrac{1 + \cos u}{\cos u}}\\
& = \dfrac{(1 - \cos u) \cos u}{(1 + \cos u) \cos u} \\
& = \dfrac{1 - \cos u}{1 + \cos u}\\
\end{align*}
It follows that LHS = RHS, so the equation is an identity.\\

The graph of $$f(x) = \tan x(1 + \sin x)$$ and $$g(x) = \dfrac{\sin x \cos x}{1
+ \sin x}$$ are shown in red and blue color lines respectively.\\
\\
From the below graph we see that $$f(x) = g(x)\ \text{for}\ n\pi$$ where $n$ can be
any integer.\\
\\
Apart from these points, there are no values of $x$ for which $f(x) = g(x)$ holds
true. Therefore, $f(x) = g(x)$ is not an identity.

The graph of $$f(x) = (\sin x + \cos x)^2$$ and $$g(x) = 1$$ are shown in red and
blue color lines respectively.\\
\\
From the below graph we see that $$f(x) = g(x)\ \text{for}\ \dfrac{n\pi}{2}$$ where
$n$ can be any integer.\\
\\
Apart from these points, there are no values of $x$ for which $f(x) = g(x)$ holds
true. Therefore, $f(x) = g(x)$ is not an identity.

$$(\sin x \sin y - \cos x \cos y)(\sin x \sin y + \cos x \cos y) = \sin^2 y -


\cos^2 x$$\\
Let's expand the LHS using the identity $$(a- b)(a + b) = a^2 - b^2$$
\begin{align*}
\text{LHS}\ & = (\sin x \sin y - \cos x \cos y)(\sin x \sin y + \cos x \cos y)\\
& = (\sin x \sin y)^2 - (\cos x \cos y)^2\\
& = \sin^2 x \sin^2 y - \cos^2 x \cos^2 y\\
& = (1 - \cos^2 x)\sin^2 y - \cos^2 x (1 - \sin^2 y)\\
& = \sin^2 y - \sin^2 y \cos^2 x - \cos^2 x + \sin^2 y \cos^2 x\\
& = \sin^2 y - \cos^2 x = \text{RHS}
\end{align*}

$$\dfrac{1 + \cos x + \sin x}{1 + \cos x - \sin x} = \dfrac{1 + \sin x}{\cos x}$$\\
We start with the left-hand side and multiply the numerator and denominator by $1 +
\cos x + \sin x$ \begin{align*}
\text{LHS} & = \dfrac{(1 + \cos x) + \sin x}{(1 + \cos x) - \sin x}\\
& = \dfrac{(1 + \cos x) + \sin x}{(1 + \cos x) - \sin x} \cdot \dfrac{(1 + \cos x)
+ \sin x}{(1 + \cos x) + \sin x}\\
& = \dfrac{[(1 + \cos x) + \sin x]^2}{(1 + \cos x)^2 - \sin^2 x}\\
& = \dfrac{(1 + \cos x)^2 + \sin^2 x + 2(1+ \cos x)\sin x}{1 + \cos^2 x + 2 \cos x
- \sin^2 x}\\
& = \dfrac{1 + \cos^2 x + 2\cos x + \sin^2 x + 2 \cos x \sin x + 2 \sin x}{\cos^2 x
+ \cos^2 x + 2 \cos x}\\
& = \dfrac{1 + 1 + 2(\cos x + \cos x \sin x + \sin x)}{2\cos^2 x + 2 \cos x}\\
& = \dfrac{2 + 2(\cos x + \cos x \sin x + \sin x)}{2\cos x(\cos x + 1)}\\
& = \dfrac{2(1 + \cos x + \cos x \sin x + \sin x)}{2\cos x(\cos x + 1)}\\
& = \dfrac{1 + \cos x + \sin x( 1 + \cos x)}{ \cos x(\cos x + 1)}\\
& = \dfrac{(1 + \cos x)(1 + \sin x)}{ \cos x(\cos x + 1)}\\
& = \dfrac{1 + \sin x}{ \cos x} = \text{RHS}\\
\end{align*}

$$(\tan x + \cot x)^4 = \sec^4 x \csc^4 x$$


\begin{align*}
\text{LHS}\ & = (\tan x + \cot x)^4\\
& = \bigg(\dfrac{\sin x}{\cos x} + \dfrac{\cos x}{\sin x} \bigg)^4\\
& = \bigg(\dfrac{\sin^2 x + \cos^2 x}{\cos x \sin x} \bigg)^4\\
& = \bigg(\dfrac{1}{\cos x \sin x} \bigg)^4\\
& = (\sec x \csc x )^4\\
& = \sec^4 x \csc^4 x = \text{RHS}
\end{align*}

$$(\sin \alpha - \tan \alpha)(\cos \alpha - \cot \alpha) = (\cos \alpha - 1)


(\sin \alpha - 1)$$
\begin{align*}
\text{LHS}\ & = (\sin \alpha - \tan \alpha)(\cos \alpha - \cot \alpha)\\
& = \bigg(\sin \alpha - \dfrac{\sin \alpha}{\cos \alpha}\bigg) \bigg(\cos \alpha
- \dfrac{\cos \alpha}{\sin \alpha}\bigg)\\
& = \sin \alpha \bigg(1 - \dfrac{1}{\cos \alpha}\bigg) \cos \alpha \bigg(1 -
\dfrac{1}{\sin \alpha}\bigg)\\
& = \sin \alpha \bigg(\dfrac{\cos \alpha - 1}{\cos \alpha}\bigg) \cos \alpha
\bigg(\dfrac{\sin \alpha - 1}{\sin \alpha}\bigg)\\
& = \dfrac{\sin \alpha}{\cos \alpha} (\cos \alpha - 1) \dfrac{\cos \alpha}{\sin
\alpha}(\sin \alpha - 1)\\
& = \dfrac{\sin \alpha}{\cos \alpha} \cdot \dfrac{\cos \alpha}{\sin \alpha}
(\cos \alpha - 1) (\sin \alpha - 1)\\
& = (\cos \alpha - 1) (\sin \alpha - 1) = \text{RHS}
\end{align*}

$$\dfrac{\sin^3 y - \csc^3 y}{\sin y - \csc y} = \sin^2 y + \csc^2 y + 1$$


Using the identity $$a^3 - b^3 = (a-b)(a^2 + b^2 + ab)$$ we expand the LHS into
\begin{align*}
\text{LHS}\ & = \dfrac{\sin^3 y - \csc^3 y}{\sin y - \csc y}\\
& = \dfrac{(\sin y - \csc y)(\sin^2 y + \csc^2 y + \sin y \csc y)}{\sin y - \csc
y}\\
& = \sin^2 y + \csc^2 y + \sin y \cdot \dfrac{1}{\sin y}\\
& = \sin^2 y + \csc^2 y + 1 = \text{RHS}\\
\end{align*}

$$\sin^6 \beta + \cos^6 \beta = 1 - 3 \sin^2 \beta \cos^2 \beta$$


Using identity $$a^6 + b^6 = (a^2 + b^2)(a^4 + b^4 - a^2b^2)$$ we expand the LHS
into \begin{align*}
\text{LHS}\ & = \sin^6 \beta + \cos^6 \beta \\
& = (\sin^2 \beta + \cos^2 \beta)(\sin^4 \beta + \cos^4 \beta - \sin^2 \beta \cos^2
\beta)\\
& = (1)(\sin^4 \beta + \cos^4 \beta - \sin^2 \beta \cos^2 \beta)\\
\end{align*}\\
Using identity $$a^4 + b^4 = (a^2 + b^2)^2 - 2a^2b^2$$ we further expand the
equation into \begin{align*}
& = \sin^4 \beta + \cos^4 \beta - \sin^2 \beta \cos^2 \beta\\
& = (\sin^2 \beta + \cos^2 \beta)^2 - 2\sin^2 \beta \cos^2 \beta - \sin^2 \beta
\cos^2 \beta\\
& = (1)^2 - 3\sin^2 \beta \cos^2 \beta\\
& = 1 - 3\sin^2 \beta \cos^2 \beta = \text{RHS}\\
\end{align*}

$$\ln | \tan x \sin x | = 2 \ln | \sin x | + \ln | \sec x |$$


\begin{align*}
\text{LHS}\ & = \ln | \tan x \sin x |\\
& = \ln \bigg| \dfrac{\sin x}{\cos x} \sin x
\bigg|\ \ \ \ \ \ \ \ \ \ \ \ \ \ \ \ \ \ \ [\ \text{Reciprocal identity}\ ]\\
& = \ln \bigg| \dfrac{\sin^2 x}{\cos x} \bigg|\\
& = \ln | (\sin x)^2(\sec x) |\\
& = \ln | (\sin x)^2| + \ln |\sec x | \ \ \ \ \ \ \ \ [\ \text{Using}\ \ln ab =
\ln a + \ln b\ ]\\
& = 2 \ln | \sin x | + \ln |\sec x | \ \ \ \ \ \ \ \ \ [\ \text{Using}\ \ln a^b =
b \ln a \ ]\\
& = \text{RHS}
\end{align*}
$$\ln | \tan x | + \ln | \cot x | = 0$$
\begin{align*}
\text{LHS}\ & = \ln | \tan x | + \ln | \cot x |\\
& = \ln | (\tan x)(\cot x)| \ \ \ \ \ \ \ \ \ \ \ \ \ \ \ \ [\ \text{Using}\ \ln a
+ \ln b = \ln ab\ ]\\
& = \ln \bigg| \tan x \cdot \dfrac{1}{\tan x} \bigg| \ \ \ \ \ \ \ \ \ \ \ \ \ \ \
\ \ [\ \text{Reciprocal identity}\ ]\\
& = \ln | 1 |\\
& = 0 = \text{RHS}
\end{align*}

$$e^{\sin^2 x}e^{\tan^2 x} = e^{\sec^2 x}e^{-\cos^2 x}$$


\begin{align*}
\text{LHS}\ & = e^{\sin^2 x}e^{\tan^2 x}\\
& = e^{\sin^2 x + \tan^2 x} \ \ \ \ \ \ \ \ \ \ \ \ \ \ \ \ \ [\ \text{Using}\ a^x
a^y = a^{x+y}\ ]\\
& = e^{1-\cos^2 x + \sec^2 x - 1}\ \ \ \ \ \ \ \ \ \ \ [\ \text{Pythagorean
identity}\ ]\\
& = e^{\sec^2 x - \cos^2 x}\\
& = e^{\sec^2 x + (-\cos^2 x)}\\
& = e^{\sec^2 x}e^{-\cos^2 x} \ \ \ \ \ \ \ \ \ \ \ \ \ \ \ [\ \text{Using}\
a^{x+y} = a^x a^y\ ]\\
& = \text{RHS}
\end{align*}

$$e^{x + 2 \ln | \sin x| } = e^{x}\sin^2 x$$


\begin{align*}
\text{LHS}\ & = e^{x + 2 \ln | \sin x| }\\
& = e^x \cdot e^{2 \ln | \sin x | } \ \ \ \ \ \ \ \ \ \ \ \ \ [\ \text{Using}\ a^{x
+y} = a^x a^y\ ]\\
& = e^x \cdot e^{\ln | (\sin x)^2 | }\ \ \ \ \ \ \ \ \ \ \ \ [\ \text{Using}\ b
\ln a = \ln a^b\ ]\\
& = e^x \cdot e^{\ln | \sin^2 x |}\\
& = e^x \cdot \sin^2 x\ \ \ \ \ \ \ \ \ \ \ \ \ \ \ \ \ \ [\ \text{Using}\ e^{\ln
a} = a\ ]\\
& = \text{RHS}
\end{align*}

$$e^{\sin^2 x}e^{\cos^2 x} = e$$


\begin{align*}
\text{LHS}\ & = e^{\sin^2 x}e^{\cos^2 x}\\
& = e^{\sin^2 x + \cos^2 x} \ \ \ \ \ \ \ \ [\ \text{Using}\ a^x a^y = a^{x+y}\ ]\\
& = e^{1}\ \ \ \ \ \ \ \ \ \ \ \ \ \ \ \ \ \ \ \ [\ \text{Pythagorean
identity}\ ]\\
& = e\\
& = \text{RHS}
\end{align*}
Since LHS = RHS, the equation is an identity.
$$\dfrac{x}{x+1} = 1 + x$$
The given equation is not an identity. Let's equate to solve the solutions.
\begin{align*}
\dfrac{x}{x+1} & = 1 + x\\
x & = (1+x)^2\\
x & = 1 + x^2 + 2x\\
x^2 + x + 1 & = 0\\
\end{align*}
Using quadratic formula, we get \begin{align*}
x & = \dfrac{-1 \pm \sqrt{(1)^2 - 4(1)(1)}}{2(1)}\\
& = \dfrac{-1 \pm \sqrt{1 - 4}}{2}\\
& = \dfrac{-1 \pm \sqrt{-3}}{2}\\
& = \dfrac{-1 \pm i\sqrt{3}}{2}\\
\end{align*}

$$\sqrt{\sin^2 x + 1} = \sqrt{\sin^2 x} + 1$$\\


The given equation is not an identity. Let's equate to solve the solutions.
\begin{align*}
\sqrt{\sin^2 x + 1} & = \sqrt{\sin^2 x} + 1\\
\sin^2 x + 1 & = (\sqrt{\sin^2 x} + 1)^2\ \ \ \ \ \ \ \ \ \ [\ \text{Squaring on
both sides}\ ]\\
\sin^2 x + 1 & = \sin^2 x + 1 + 2\sin^2 x\\
2\sin^2 x & = 0\\
\sin x & = 0\\
x & = \sin^{-1} (0)\\
& = n\pi \ \ \ \ \ \ \ \ \ \ \ \ \ \ \ \ \ \ \ \ \ \ \ \ \ \ [\ \text{where}\ n =
\text{any integer}\ ]\\
\end{align*}

$$x e^{\ln x^2} = x^3$$


\begin{align*}
\text{LHS} & = x e^{\ln x^2}\\
& = x \cdot x^2 \ \ \ \ \ \ \ \ [\ \text{Since}\ e^{\ln a} = a\ ]\\
& = x^3\\
& = \text{RHS}
\end{align*}
Since LHS = RHS, the equation is an identity.

Given that $$x = R \cos \theta \sin \phi,\ y = R \sin \theta \sin \phi,\
\text{and}\ z = R \cos \phi$$ we need to verify the identity $$x^2 + y^2 + z^2 =
R^2$$
\begin{align*}
\text{LHS}\ & = x^2 + y^2 + z^2\\
& = (R \cos \theta \sin \phi)^2 + (R \sin \theta \sin \phi)^2 + (R \cos \phi)^2\\
& = R^2 \cos^2 \theta \sin^2 \phi + R^2 \sin^2 \theta \sin^2 \phi + R^2 \cos^2
\phi\\
& = R^2 \sin^2 \phi (\cos^2 \theta + \sin^2 \theta) + R^2 \cos^2
\phi\ \ \ \ \ \ \ \ [\ \text{Taking common}\ ]\\
& = R^2 \sin^2 \phi (1) + R^2 \cos^2 \phi \ \ \ \ \ \ \ \ \ \ \ \ \ \ \ \ \ \ \ \ \
\ \ \ [\ \text{Pythagorean identity}\ ]\\
& = R^2 (\sin^2 \phi + \cos^2
\phi)\ \ \ \ \ \ \ \ \ \ \ \ \ \ \ \ \ \ \ \ \ \ \ \ \ \ \ \ \ \ [\ \text{Taking
common}\ ]\\
& =
R^2(1) \ \ \ \ \ \ \ \ \ \ \ \ \ \ \ \ \ \ \ \ \ \ \ \ \ \ \ \ \ \ \ \ \ \ \ \ \ \
\ \ \ \ \ \ \ \ \ [\ \text{Pythagorean identity}\ ]\\
& = R^2 = \text{RHS}
\end{align*}

$$(x+y)^2 = x^2 + 2xy + y^2$$\\


The given equation is an identity. Let's take different values of $x$ and $y$ to
show that. \begin{align*}
\text{Let}\ x = y = 1,\ \ \ \ \ \ \ \text{then}\ (1+1)^2 & \overset{?} = (1)^2 +
2(1)(1) + (1)^2\\
(2)^2 & \overset{?} = 1 + 2 + 1\\
4 & \overset{?} = 4\\
\text{LHS}\ & = \text{RHS}\\
\\
\text{Let}\ x = 1,\ y = 2,\ \ \ \ \ \ \text{then}\ (1+2)^2 & \overset{?} = (1)^2 +
2(1)(2) + (2)^2\\
(3)^2 & \overset{?} = 1 + 4 + 4\\
9 & \overset{?} = 9\\
\text{LHS}\ & = \text{RHS}\\
\\
\text{Let}\ x = 2,\ y = 3, \ \ \ \ \ \text{then}\ (2+3)^2 & \overset{?} = (2)^2 +
2(2)(3) + (3)^2\\
(5)^2 & \overset{?} = 4 + 12 + 9\\
25 & \overset{?} = 25\\
\text{LHS}\ & = \text{RHS}
\end{align*}
Therefore, we see that the given equation is an identity.

$$x^2+y^2 = 1$$\\
The given equation is not an identity. Let's take some values of $x$ and $y$ to
show that. \begin{align*}
\text{Let}\ x = y = 1,\ \ \ \text{then}\ (1)^2 +(1)^2 & \overset{?} = 1\\
1 + 1 & \overset{?} = 1\\
2 & \overset{?} = 1\\
\text{LHS}\ & \ne \text{RHS}
\end{align*}
Therefore, we see that the given equation is not an identity.

$$x(y + z) = xy + xz$$\\
The given equation is an identity. Let's take different values of $x,\ y$ and $z$
to show that. \begin{align*}
\text{Let}\ x = y = z = 1,\ \ \ \ \ \ \ \text{then}\ 1(1+1) & \overset{?} = (1)(1)
+ (1)(1)\\
1(2) & \overset{?} = 1 + 1\\
2 & \overset{?} = 2\\
\text{LHS}\ & = \text{RHS}\\
\\
\text{Let}\ x = 1,\ y = 2,\ z = 3,\ \ \ \ \ \ \ \text{then}\ 1(2+3) & \overset{?} =
(1)(2) + (1)(3)\\
1(5) & \overset{?} = 2 + 3\\
5 & \overset{?} = 5\\
\text{LHS}\ & = \text{RHS}\\
\\
\text{Let}\ x = 2,\ y = 3,\ z = 4,\ \ \ \ \ \ \ \text{then}\ 2(3+4) & \overset{?} =
(2)(3) + (2)(4)\\
2(7) & \overset{?} = 6 + 8\\
14 & \overset{?} = 14\\
\text{LHS}\ & = \text{RHS}
\end{align*}
Therefore, we see that the given equation is an identity.

$$t^2 - \cos^2 t = (t - \cos t)(t + \cos t)$$\\


Given equation is an identity. Let's take different values of $t$ to show that.
\begin{align*}
\text{Let}\ t = 0,\ \ \ \ \ \ \ \text{then}\ (0)^2 - \cos^2 (0) & \overset{?} = (0
- \cos 0)(0 + \cos 0)\\
0 - 1 & \overset{?} = (-1)(1)\\
-1 & \overset{?} = -1\\
\text{LHS}\ & = \text{RHS}\\
\\
\text{Let}\ t = \pi/2,\ \ \ \ \ \ \ \text{then}\ (\pi/2)^2 - \cos^2 (\pi/2) &
\overset{?} = (\pi/2 - \cos (\pi/2))(\pi/2 + \cos (\pi/2))\\
\pi^2/4 - 0 & \overset{?} = (\pi/2 - 0)(\pi/2 + 0)\\
\pi^2/4 & \overset{?} = \pi^2/4\\
\text{LHS}\ & = \text{RHS}\\
\\
\text{Let}\ t = \pi,\ \ \ \ \ \ \ \text{then}\ (\pi)^2 - \cos^2 (\pi) & \overset{?}
= (\pi - \cos (\pi))(\pi + \cos (\pi))\\
\pi^2 - 1 & \overset{?} = (\pi - (-1))(\pi + (-1))\\
\pi^2 -1 & \overset{?} = \pi^2 - \pi + \pi - 1\\
\pi^2 -1 & \overset{?} = \pi^2 - 1\\
\text{LHS}\ & = \text{RHS}
\end{align*}
Therefore, we see that the given equation is an identity.

$$\sin t + \cos t = 1$$\\


Given equation is not an identity. Let's take some value of $t$ to show that.
\begin{align*}
\text{Let}\ t = \dfrac{\pi}{4},\ \ \ \text{then}\ \sin \dfrac{\pi}{4} + \cos
\dfrac{\pi}{4} & \overset{?} = 1\\
\dfrac{1}{\sqrt{2}} + \dfrac{1}{\sqrt{2}} & \overset{?} = 1\\
\dfrac{2}{\sqrt{2}} & \overset{?} = 1\\
\sqrt{2} & \overset{?} = 1\\
\text{LHS}\ & \ne \text{RHS}
\end{align*}
Therefore, we see that the given equation is not an identity.

$$x^2 + \tan^2 x = 0$$\\


Given equation is not an identity. Let's take some value of $x$ to show that.
\begin{align*}
\text{Let}\ x = \dfrac{\pi}{3},\ \ \ \text{then}\ \bigg(\dfrac{\pi}{3}\bigg)^2 +
\tan^2 \bigg(\dfrac{\pi}{3}\bigg) & \overset{?} = 0\\
\dfrac{\pi^2}{9} + 3 & \overset{?} = 0\\
\text{LHS}\ & \ne \text{RHS}
\end{align*}
Therefore, we see that the given equation is not an identity.

An equation of the form $$f(x) = g(x)$$ is not an identity if for any value of $x$,
LHS $\ne$ RHS.\\
\\
\\
Given equation is $$\sin 2x = 2 \sin x$$ Let $x = \dfrac{\pi}{3}$. Then
\begin{align*}
\sin 2\bigg(\dfrac{\pi}{3}\bigg) & \overset{?} = 2 \sin \bigg(\dfrac{\pi}
{3}\bigg)\\
\sin \bigg(\dfrac{2\pi}{3}\bigg) & \overset{?} = 2 \bigg(\dfrac{\sqrt{3}}
{2}\bigg)\\
\dfrac{\sqrt{3}}{2} & \overset{?} = \sqrt{3}\\
\text{LHS}\ & \ne \text{RHS}
\end{align*}
Since, LHS $\ne$ RHS, the given equation is not an identity.

An equation of the form $$f(x) = g(x)$$ is not an identity if for any value of $x$,
LHS $\ne$ RHS.\\
\\
\\
Given equation is $$\sin (x+y) = \sin x + \sin y$$ Let $x = y = \dfrac{\pi}{2}$.
Then \begin{align*}
\sin \bigg(\dfrac{\pi}{2} + \dfrac{\pi}{2} \bigg) & \overset{?} = \sin
\bigg(\dfrac{\pi}{2}\bigg) + \sin \bigg(\dfrac{\pi}{2}\bigg)\\
\sin (\pi) & \overset{?} = 1 + 1\\
0 & \overset{?} = 1\\
\text{LHS}\ & \ne \text{RHS}
\end{align*}
Since, LHS $\ne$ RHS, the given equation is not an identity.

An equation of the form $$f(x) = g(x)$$ is not an identity if for any value of $x$,
LHS $\ne$ RHS.\\
\\
\\
Given equation is $$\sec^2 x + \csc^2 x = 1$$ Let $x = \dfrac{\pi}{3}$. Then
\begin{align*}
\sec^2 \bigg(\dfrac{\pi}{3}\bigg) + \csc^2 \bigg(\dfrac{\pi}{3}\bigg) & \overset{?}
= 1\\
4 + \dfrac{4}{3} & \overset{?} = 1\\
\dfrac{16}{3} & \overset{?} = 1\\
\text{LHS}\ & \ne \text{RHS}
\end{align*}
Since, LHS $\ne$ RHS, the given equation is not an identity.
An equation of the form $$f(x) = g(x)$$ is not an identity if for any value of $x$,
LHS $\ne$ RHS.\\
\\
\\
Given equation is $$\dfrac{1}{\sin x + \cos x} = \csc x + \sec x$$ Let $x =
\dfrac{\pi}{3}$. Then \begin{align*}
\dfrac{1}{\sin \bigg(\dfrac{\pi}{3}\bigg) + \cos \bigg(\dfrac{\pi}{3}\bigg)} &
\overset{?} = \csc \bigg(\dfrac{\pi}{3}\bigg) + \sec \bigg(\dfrac{\pi}{3}\bigg)\\
\dfrac{1}{\dfrac{\sqrt{3}}{2} + \dfrac{1}{2}} & \overset{?} = \dfrac{2}{\sqrt{3}} +
2\\
\dfrac{1}{\dfrac{\sqrt{3} + 1}{2}} & \overset{?} = \dfrac{2 + 2\sqrt{3}}
{\sqrt{3}}\\
\dfrac{2}{\sqrt{3} + 1} & \overset{?} = \dfrac{2(\sqrt{3} +1)}{\sqrt{3}}\\
\dfrac{2(\sqrt{3} - 1)}{3 - 1} & \overset{?} = \dfrac{2(\sqrt{3} + 1)}{\sqrt{3}}\\
\sqrt{3} - 1 & \overset{?} = \dfrac{2(\sqrt{3} + 1)}{\sqrt{3}}\\
\text{LHS}\ & \ne \text{RHS}
\end{align*}
Since, LHS $\ne$ RHS, the given equation is not an identity.

$$\tan \bigg(x - \dfrac{\pi}{2}\bigg) = -\cot x$$\\


Using $\tan x = \dfrac{\sin x}{\cos x}$, we change the LHS to \begin{align*}
\text{LHS}\ & = \tan \bigg(x - \dfrac{\pi}{2}\bigg)\\
& = \dfrac{\sin \bigg(x - \dfrac{\pi}{2}\bigg)}{\cos \bigg(x - \dfrac{\pi}
{2}\bigg)}\\
& = \dfrac{\sin x \cos \dfrac{\pi}{2} - \cos x \sin \dfrac{\pi}{2}}{\cos x \cos
\dfrac{\pi}{2} + \sin x \sin \dfrac{\pi}{2}} \ \ \ \ \ \ \ \ \ \ [\
\text{Subtraction formula for sine and cosine}\ ]\\
& = \dfrac{\sin x (0) - \cos x (1)}{\cos x (0) + \sin x (1)}\\
& = \dfrac{- \cos x}{\sin x}\\
& = -\cot x = \text{RHS}
\end{align*}

$$\cos \bigg(x + \dfrac{\pi}{3}\bigg) + \sin \bigg(x - \dfrac{\pi}{6}\bigg) = 0$$\\


\\
Using addition formula for cosine and subtraction formula for sine, we get
\begin{align*}
\text{LHS}\ & = \cos \bigg(x + \dfrac{\pi}{3}\bigg) + \sin \bigg(x - \dfrac{\pi}
{6}\bigg)\\
& = \cos x \cos \dfrac{\pi}{3} - \sin x \sin \dfrac{\pi}{3} + \sin x \cos
\dfrac{\pi}{6} - \cos x \sin \dfrac{\pi}{6} \\
& = \cos x \cdot \dfrac{1}{2} - \sin x \cdot \dfrac{\sqrt{3}}{2} + \sin x \cdot
\dfrac{\sqrt{3}}{2} - \cos x \cdot \dfrac{1}{2} \\
& = \dfrac{\cos x}{2} - \dfrac{\sqrt{3}\sin x}{2} + \dfrac{\sqrt{3}\sin x}{2} -
\dfrac{\cos x}{2} \\
& = 0 = \text{RHS}
\end{align*}

$$\tan \bigg(x + \dfrac{\pi}{3}\bigg) = \dfrac{\sqrt{3} + \tan x}{1 - \sqrt{3} \tan


x}$$\\
\begin{align*}
\text{Using addition}\ & \text{formula for tangent, we get}\\
\text{LHS}\ & = \tan \bigg(x + \dfrac{\pi}{3}\bigg)\\
& = \dfrac{\tan x + \tan \dfrac{\pi}{3}}{1 - \tan x \tan \dfrac{\pi}{3}}\\
& = \dfrac{\tan x + \sqrt{3}}{1 - \tan x \cdot \sqrt{3}}\\
& = \dfrac{\sqrt{3} + \tan x}{1 - \sqrt{3} \tan x}\\
& = \text{RHS}
\end{align*}

$$\dfrac{\tan x - \tan y}{1 - \tan x \tan y} = \dfrac{\sin (x-y)}{\cos (x + y)}$$\\


Using $\tan x = \dfrac{\sin x}{\cos x}$, we change the LHS to \begin{align*}
\text{LHS}\ & = \dfrac{\tan x - \tan y}{1 - \tan x \tan y}\\
& = \dfrac{\dfrac{\sin x}{\cos x} - \dfrac{\sin y}{\cos y}}{1 - \dfrac{\sin x}{\cos
x} \dfrac{\sin y}{\cos y}}\\
& = \dfrac{\dfrac{\sin x \sin y - \cos x \sin y}{\cos x \cos y}}{\dfrac{\cos x \cos
y - \sin x \sin y}{\cos x \cos y}}\\
& = \dfrac{\sin x \sin y - \cos x \sin y}{\cos x \cos y - \sin x \sin y}\\
& = \dfrac{\sin (x -y)}{\cos (x + y)}\\
& = \text{RHS}\\
\end{align*}

$$\cos (x+y)\cos y + \sin(x+y)\sin y = \cos x$$\\


Using addition formula for cosine and subtraction formula for sine, we get
\begin{align*}
\text{LHS}\ & = \cos (x+y)\cos y + \sin(x+y)\sin y\\
& = (\cos x \cos y - \sin x \sin y)\cos y + (\sin x \cos y + \cos x \sin y)\sin y\\
& = \cos x \cos^2 y - \sin x \sin y \cos y + \sin x \sin y \cos y + \cos x \sin^2
y\\
& = \cos x ( \cos^2 y + \sin^2 y)\\
& = \cos x (1)\\
& = \cos x = \text{RHS}\\
\end{align*}

$$\sin(\sin^{-1}x + \cos^{-1} y)$$\\


Let $\theta = \sin^{-1} x$ and $\phi = \cos^{-1} y$. Using the methods of Section
6.4, we sketch triangles with angles $\theta$ and $\phi$ such that $\sin \theta =
x$ and $\cos \phi = y$. From the triangles, we have $$\cos \theta = \sqrt{1 - x^2}\
\ \text{and}\ \ \sin \phi = \sqrt{1 - y^2}$$\\
From the Addition Formula For Sine, we have \begin{align*}
\sin(\sin^{-1}x + \cos^{-1} y) & = \sin(\theta + \phi)\\
& = \sin \theta \cos \phi + \cos \theta \sin \phi\\
& = x y + \sqrt{1 - x^2} \sqrt{1 - y^2}
\end{align*}

$$\sin\bigg(\cos^{-1} \dfrac{1}{2} + \tan^{-1} 1\bigg)$$\\


Let $\theta = \cos^{-1} \dfrac{1}{2}$ and $\phi = \tan^{-1} 1$. Using the methods
of Section 6.4, we sketch triangles with angles $\theta$ and $\phi$ such that $\cos
\theta = \dfrac{1}{2}$ and $\tan \phi = 1$. From the triangles, we have $$\theta
= \dfrac{\pi}{3}\ \ \text{and}\ \ \phi = \dfrac{\pi}{4}$$\\
From the Addition Formula For Sine, we have \begin{align*}
\sin\bigg(\cos^{-1} \dfrac{1}{2} + \tan^{-1} 1\bigg) & = \sin(\theta + \phi)\\
& = \sin\bigg( \dfrac{\pi}{3} + \dfrac{\pi}{4} \bigg)\\
& = \sin \dfrac{\pi}{3} \cos \dfrac{\pi}{4} + \cos \dfrac{\pi}{3} \sin \dfrac{\pi}
{4}\\
& = \dfrac{\sqrt{3}}{2} \cdot \dfrac{1}{\sqrt{2}} + \dfrac{1}{2} \cdot \dfrac{1}
{\sqrt{2}}\\
& = \dfrac{\sqrt{3}}{2\sqrt{2}} + \dfrac{1}{2\sqrt{2}}\\
& = \dfrac{\sqrt{3} + 1}{2\sqrt{2}}\\
& = \dfrac{\sqrt{6} + \sqrt{2}}{4}\\
\end{align*}

$$\cos\bigg(\sin^{-1} \dfrac{\sqrt{3}}{2} + \cot^{-1} \sqrt{3}\bigg)$$\\


Let $\theta = \sin^{-1} \dfrac{\sqrt{3}}{2}$ and $\phi = \cot^{-1} \sqrt{3}$. Using
the methods of Section 6.4, we sketch triangles with angles $\theta$ and $\phi$
such that $\sin \theta = \dfrac{\sqrt{3}}{2}$ and $\cot \phi = \sqrt{3}$. From the
triangles, we have $$\theta = \dfrac{\pi}{3}\ \ \text{and}\ \ \phi = \dfrac{\pi}
{6}$$\\
From the Addition Formula For Coine, we have \begin{align*}
\cos\bigg(\sin^{-1} \dfrac{\sqrt{3}}{2} + \cot^{-1} \sqrt{3}\bigg) & = \cos (\theta
+ \phi)\\
& = \cos \bigg( \dfrac{\pi}{3} + \dfrac{\pi}{6} \bigg)\\
& = \cos \bigg( \dfrac{\pi}{2}\bigg)\\
& = 0
\end{align*}

$$\tan\bigg(\sin^{-1} \dfrac{3}{4} - \cos^{-1} \dfrac{1}{3}\bigg)$$\\


Let $\theta = \sin^{-1} \dfrac{3}{4}$ and $\phi = \cos^{-1} \dfrac{1}{3}$. Using
the methods of Section 6.4, we sketch triangles with angles $\theta$ and $\phi$
such that $\sin \theta = \dfrac{3}{4}$ and $\cos \phi = \dfrac{1}{3}$. From the
triangles, we have $$\tan \theta = \dfrac{3}{\sqrt{7}}\ \ \text{and}\ \ \tan \phi =
2\sqrt{2}$$\\
From the Subtraction Formula For Tangent, we have \begin{align*}
\tan\bigg(\sin^{-1} \dfrac{3}{4} - \cos^{-1} \dfrac{1}{3}\bigg) & = \tan(\theta
- \phi)\\
& = \dfrac{ \tan \theta - \tan \pi}{1 + \tan \theta \tan \phi}\\
& = \dfrac{ \dfrac{3}{\sqrt{7}} - 2\sqrt{2}}{1 + \dfrac{3}{\sqrt{7}} \cdot
2\sqrt{2}}\\
& = \dfrac{ \dfrac{3 - 2\sqrt{14}}{\sqrt{7}} }{1 + \dfrac{6\sqrt{2}}{\sqrt{7}}} \\
& = \dfrac{ \dfrac{3 - 2\sqrt{14}}{\sqrt{7}} }{\dfrac{\sqrt{7} + 6\sqrt{2}}
{\sqrt{7}}} \\
& = \dfrac{3 - 2\sqrt{14}}{\sqrt{7} + 6\sqrt{2}} \\
\end{align*}\\

$$\sin\bigg(\cos^{-1} \dfrac{2}{3} - \tan^{-1} \dfrac{1}{2}\bigg)$$\\


Let $\theta = \cos^{-1} \dfrac{2}{3}$ and $\phi = \tan^{-1} \dfrac{1}{2}$. Using
the methods of Section 6.4, we sketch triangles with angles $\theta$ and $\phi$
such that $\cos \theta = \dfrac{2}{3}$ and $\tan \phi = \dfrac{1}{2}$. From the
triangles, we have $$\sin \theta = \dfrac{\sqrt{5}}{3},\ \ \ \sin \phi = \dfrac{1}
{\sqrt{5}}\ \ \ \text{and}\ \ \cos \phi = \dfrac{2}{\sqrt{5}}$$\\
From the Subtraction Formula For Sine, we have \begin{align*}
\sin\bigg(\cos^{-1} \dfrac{2}{3} - \tan^{-1} \dfrac{1}{2}\bigg) & = \sin(\theta
- \phi)\\
& = \sin \theta \cos \phi - \cos \theta \sin \phi\\
& = \dfrac{\sqrt{5}}{3} \cdot \dfrac{2}{\sqrt{5}} - \dfrac{2}{3} \cdot \dfrac{1}
{\sqrt{5}}\\
& = \dfrac{2}{3} - \dfrac{2}{3\sqrt{5}}\\
& = \dfrac{2\sqrt{5} - 2}{3\sqrt{5}}\\
& = \dfrac{10 - 2\sqrt{5}}{15}\\
\end{align*}\\

It is given that $$\cos \theta = \dfrac{3}{5},\ \theta \in \text{Quadrant IV}$$


Since $\sin \theta$ is negative in Quadrant IV, we get $$\sin \theta = -\sqrt{1
- \cos^2 \theta} = -\sqrt{1 - \dfrac{9}{25}} = \sqrt{\dfrac{16}{25}} = -\dfrac{4}
{5}$$ It is also given that $$\tan \phi = -\sqrt{3},\ \phi \in \text{Quadrant II}$$
Then $\phi = \pi - \dfrac{\pi}{3} = \dfrac{2 \pi}{3}$.
\\
\\
\\
From the Subtraction Formula For Cosine, we have \begin{align*}
\cos (\theta - \phi) & = \cos \theta \cos \phi + \sin \theta \sin \phi\\
& = \dfrac{3}{5} \cdot \cos \dfrac{2\pi}{3} + \bigg(-\dfrac{4}{5}\bigg) \cdot
\sin \dfrac{2\pi}{3}\\
& = \dfrac{3}{5} \cdot \bigg(-\dfrac{1}{2}\bigg) - \bigg(\dfrac{4}{5}\bigg)
\cdot \dfrac{\sqrt{3}}{2}\\
& = -\dfrac{3}{10} - \dfrac{2\sqrt{3}}{5}\\
& = -\dfrac{3 + 4\sqrt{3}}{10}\\
\end{align*}

It is given that $$\tan \theta = \dfrac{4}{3},\ \theta \in \text{Quadrant III}$$


Since both $\sin \theta$ and $\cos \theta$ are negative in Quadrant III, we get $
$\sin \theta = -\dfrac{4}{\sqrt{4^2 + 3^2}} = -\dfrac{4}{5}\ \text{and}\ \cos
\theta = -\dfrac{3}{\sqrt{4^2 + 3^2}} = -\dfrac{3}{5}$$ It is also given that $
$\sin \phi = -\dfrac{\sqrt{10}}{10} = -\dfrac{1}{\sqrt{10}},\ \phi \in
\text{Quadrant IV}$$ Since, $\cos \phi$ is positive in Quadrant IV, we get $
$\cos \phi = \sqrt{1 - \dfrac{1}{10}} = \dfrac{3}{\sqrt{10}}$$
\\
\\
From the Subtraction Formula For Sine, we have \begin{align*}
\sin (\theta - \phi) & = \sin \theta \cos \phi - \cos \theta \sin \phi\\
& = -\dfrac{4}{5} \cdot \dfrac{3}{\sqrt{10}} - \bigg(-\dfrac{3}{5}\bigg) \cdot
\bigg(-\dfrac{1}{\sqrt{10}}\bigg)\\
& = -\dfrac{12}{5\sqrt{10}} - \dfrac{3}{5\sqrt{10}}\\
& = -\dfrac{15}{5\sqrt{10}}\\
& = -\dfrac{3}{\sqrt{10}}\\
& = -\dfrac{3\sqrt{10}}{10}\\
\end{align*}

It is given that $$\sin \theta = \dfrac{5}{13},\ \theta \in \text{Quadrant I}$$


Since $\cos \theta$ is positive in Quadrant I, we get $$\cos \theta = \sqrt{1 -
\sin^2 \theta} = \sqrt{1 - \dfrac{25}{169}} = \sqrt{\dfrac{144}{169}} = \dfrac{12}
{13}$$ It is also given that $$\cos \phi = -\dfrac{2\sqrt{5}}{5},\ \phi \in
\text{Quadrant II}$$ Since $\sin \phi$ is positive in Quadrant II, we get $$\sin
\phi = \sqrt{1 - \cos^2 \phi} = \sqrt{1 - \dfrac{20}{25}} = \sqrt{\dfrac{5}{25}}
= \dfrac{\sqrt{5}}{5}$$
\\
\\
\\
From the Addition Formula For Sine, we have \begin{align*}
\sin (\theta + \phi) & = \sin \theta \cos \phi + \cos \theta \sin \phi\\
& = \dfrac{5}{13} \cdot \bigg(\dfrac{-2\sqrt{5}}{5}\bigg) + \dfrac{12}{13}
\cdot \dfrac{\sqrt{5}}{5}\\
& = -\dfrac{2\sqrt{5}}{13} + \dfrac{12\sqrt{5}}{65}\\
& = -\dfrac{10\sqrt{5} - 12\sqrt{5} }{65}\\
& = -\dfrac{-2\sqrt{5} }{65}\\
& = \dfrac{2\sqrt{5}}{65}\\
\end{align*}

It is given that $$\cos \theta = -\dfrac{1}{3},\ \theta \in \text{Quadrant III}$$


Since $\tan \theta$ is positive in Quadrant III, we get $$\tan \theta =
\dfrac{\sqrt{3^2 - 1^2}}{1} = \sqrt{8} = 2\sqrt{2}$$ It is also given that $
$\sin \phi = \dfrac{1}{4},\ \phi \in \text{Quadrant II}$$ Since $\tan \phi$ is
negative in Quadrant II, we get $$\tan \phi = -\dfrac{1}{\sqrt{4^2 - 1^2}} =
-\dfrac{1}{\sqrt{15}}$$
\\
\\
\\
From the Addition Formula For Tangent, we have \begin{align*}
\tan (\theta + \phi) & = \dfrac{\tan \theta + \tan \phi}{1 - \tan \theta \tan
\phi}\\
& = \dfrac{2\sqrt{2} + \bigg(-\dfrac{1}{\sqrt{15}}\bigg)}{1 - 2\sqrt{2}
\bigg(-\dfrac{1}{\sqrt{15}}\bigg)}\\
& = \dfrac{\dfrac{2\sqrt{30} -1}{\sqrt{15}}}{1 + \dfrac{2\sqrt{2}}{\sqrt{15}}}\\
& = \dfrac{\dfrac{2\sqrt{30} -1}{\sqrt{15}}}{\dfrac{\sqrt{15} + 2\sqrt{2}}
{\sqrt{15}}}\\
& = \dfrac{2\sqrt{30} -1}{\sqrt{15} + 2\sqrt{2}}\\
\end{align*}

$$\sin x - \cos x$$\\


From Sum of Sines And Cosines, we know that if $A$ and $B$ are real numbers, then $
$A \sin x + B \cos x = k\sin (x + \phi)$$
where $k = \sqrt{A^2 + B^2}$ and $\phi$ satisfies $$\cos \phi = \dfrac{A}{\sqrt{A^2
+ B^2}}\ \ \ \ \text{and}\ \ \ \ \sin \phi = \dfrac{B}{\sqrt{A^2 + B^2}}$$\\
Comparing the given exercise with the above theorem, we have $$A = 1\ \text{and}\ \
B = -1\ \ \Rightarrow \ \ k = \sqrt{(1)^2 + (-1)^2} = \sqrt{2}$$
and \begin{align*}
\cos \phi & = \dfrac{1}{\sqrt{2}}\\
\phi & = \cos^{-1} \bigg(\dfrac{1}{\sqrt{2}}\bigg)\\
& = \dfrac{\pi}{4}\ \text{or}\ \dfrac{7\pi}{4}\\
\\
\sin \phi & = \dfrac{-1}{\sqrt{2}}\\
\phi & = \sin^{-1} \bigg(\dfrac{-1}{\sqrt{2}}\bigg)\\
& = \dfrac{5\pi}{4}\ \text{or}\ \dfrac{7\pi}{4}
\end{align*}\\
From the above two cases, we get $\phi = \dfrac{7\pi}{4}$.\\
\\
Therefore, the given expression can be written in terms of sine only as
$$\sin x - \cos x = \sqrt{2} \sin \bigg(x + \dfrac{7\pi}{4}\bigg)$$
$$3\sin \pi x + 3\sqrt{3} \cos \pi x$$\\
From Sum of Sines And Cosines, we know that if $A$ and $B$ are real numbers, then $
$A \sin x + B \cos x = k\sin (x + \phi)$$
where $k = \sqrt{A^2 + B^2}$ and $\phi$ satisfies $$\cos \phi = \dfrac{A}{\sqrt{A^2
+ B^2}}\ \ \ \ \text{and}\ \ \ \ \sin \phi = \dfrac{B}{\sqrt{A^2 + B^2}}$$\\
\\
Comparing the given exercise with the above theorem, we have $$A = 3\ \text{and}\ B
= 3\sqrt{3}, \ \ \Rightarrow \ \ k = \sqrt{(3)^2 + (3\sqrt{3})^2} = 6$$
and \begin{align*}
\cos \phi & = \dfrac{3}{6}\\
\phi & = \cos^{-1} \bigg(\dfrac{1}{2}\bigg)\\
& = \dfrac{\pi}{3}\ \text{or}\ \dfrac{5\pi}{6}\\
\\
\sin \phi & = \dfrac{3\sqrt{3}}{6}\\
\phi & = \sin^{-1} \bigg(\dfrac{\sqrt{3}}{2}\bigg)\\
& = \dfrac{\pi}{3}\ \text{or}\ \dfrac{2\pi}{3}
\end{align*}\\
From the above two cases, we get $\phi = \dfrac{\pi}{3}$.\\
\\
Therefore, the given expression can be written in terms of sine only as
$$3 \sin \pi x - 3\sqrt{3} \cos \pi x = 6 \sin \bigg(\pi x + \dfrac{\pi}{3}\bigg)$$

(a)\ $g(x) = \cos 2x + \sqrt{3} \sin 2x$\\


\\
From Sum of Sines And Cosines, we know that if $A$ and $B$ are real numbers, then $
$A \sin x + B \cos x = k\sin (x + \phi)$$
where $k = \sqrt{A^2 + B^2}$ and $\phi$ satisfies $$\cos \phi = \dfrac{A}{\sqrt{A^2
+ B^2}}\ \ \ \ \text{and}\ \ \ \ \sin \phi = \dfrac{B}{\sqrt{A^2 + B^2}}$$\\
\\
Comparing the given exercise with the above theorem, we have $$A = \sqrt{3}\
\text{and}\ B = 1, \ \ \Rightarrow \ \ k = \sqrt{(\sqrt{3})^2 + (1)^2} = 2$$
and \begin{align*}
\cos \phi & = \dfrac{\sqrt{3}}{2}\\
\phi & = \cos^{-1} \bigg(\dfrac{\sqrt{3}}{2}\bigg)\\
& = \dfrac{\pi}{6}\ \text{or}\ \dfrac{11\pi}{6}\\
\\
\sin \phi & = \dfrac{1}{2}\\
\phi & = \sin^{-1} \bigg(\dfrac{1}{2}\bigg)\\
& = \dfrac{\pi}{6}\ \text{or}\ \dfrac{5\pi}{6}
\end{align*}\\
From the above two cases, we get $\phi = \dfrac{\pi}{6}$.\\
\\
Therefore, the given expression can be written in terms of sine only as
$$g(x) = 2 \sin \bigg(2x + \dfrac{\pi}{6}\bigg)$$

(a)\ $f(x) = \sin x + \cos x$\\


\\
From Sum of Sines And Cosines, we know that if $A$ and $B$ are real numbers, then $
$A \sin x + B \cos x = k\sin (x + \phi)$$
where $k = \sqrt{A^2 + B^2}$ and $\phi$ satisfies $$\cos \phi = \dfrac{A}{\sqrt{A^2
+ B^2}}\ \ \ \ \text{and}\ \ \ \ \sin \phi = \dfrac{B}{\sqrt{A^2 + B^2}}$$\\
\\
Comparing the given exercise with the above theorem, we have $$A = 1\ \text{and}\ B
= 1, \ \ \Rightarrow \ \ k = \sqrt{(1)^2 + (1)^2} = \sqrt{2}$$
and \begin{align*}
\cos \phi & = \dfrac{1}{\sqrt{2}}\\
\phi & = \cos^{-1} \bigg(\dfrac{1}{\sqrt{2}}\bigg)\\
& = \dfrac{\pi}{4}\ \text{or}\ \dfrac{7\pi}{4}\\
\\
\sin \phi & = \dfrac{1}{\sqrt{2}}\\
\phi & = \sin^{-1} \bigg(\dfrac{1}{\sqrt{2}}\bigg)\\
& = \dfrac{\pi}{4}\ \text{or}\ \dfrac{3\pi}{4}
\end{align*}\\
From the above two cases, we get $\phi = \dfrac{\pi}{4}$.\\
\\
Therefore, the given expression can be written in terms of sine only as
$$f(x) = \sqrt{2} \sin \bigg(x + \dfrac{\pi}{4}\bigg)$$

Given $f(x) = \cos x$ and $g(x) = \sin x$. Then, \begin{align*}


\dfrac{f(x+h) - f(x)}{h} & = \dfrac{\cos(x+h) - \cos x}{h}
\end{align*}\\
Using Addition Formula for Cosines, we get \begin{align*}
\dfrac{f(x+h) - f(x)}{h} & = \dfrac{\cos(x+h) - \cos x}{h}\\
& = \dfrac{\cos x \cos h - \sin x \sin h - \cos x}{h}\\
& = \dfrac{\cos x (\cos h - 1 ) - \sin x \sin h}{h}\\
& = \dfrac{\cos x (\cos h - 1 )}{h} - \dfrac{\sin x \sin h}{h}\\
& = -\cos x \bigg(\dfrac{1 - \cos h}{h}\bigg) - \sin x \bigg( \dfrac{\sin h}
{h}\bigg)
\end{align*}
Hence Proved.

Given $f(x) = \cos x$ and $g(x) = \sin x$. Then, \begin{align*}


\dfrac{g(x+h) - g(x)}{h} & = \dfrac{\sin(x+h) - \sin x}{h}
\end{align*}\\
Using Addition Formula for Sines, we get \begin{align*}
\dfrac{g(x+h) - g(x)}{h} & = \dfrac{\sin(x+h) - \sin x}{h}\\
& = \dfrac{\sin x \cos h + \cos x \sin h - \sin x}{h}\\
& = \dfrac{\cos x \sin h + \sin x (\cos h - 1 )}{h}\\
& = \dfrac{\cos x \sin h}{h} + \dfrac{\sin x (\cos h - 1 )}{h}\\
& = \cos x \bigg(\dfrac{\sin h}{h}\bigg) - \sin x \bigg( \dfrac{1 - \cosh}{h}\bigg)
\end{align*}
Hence Proved.

(a)\ The graph of function $$y = \sin^2 \bigg( x + \dfrac{\pi}{4}\bigg) + \sin^2


\bigg( x - \dfrac{\pi}{4}\bigg)$$ is shown below. From the below graph, we can see
that the graph of function $y$ is an horizontal line $y = 1$.

(b)\ To prove the above conjecture, we need to expand the function $y$ and see if
it represents an horizontal line.
\begin{align*}
y & = \sin^2 \bigg( x + \dfrac{\pi}{4}\bigg) + \sin^2 \bigg( x - \dfrac{\pi}
{4}\bigg)\\
& = \bigg[\sin \bigg( x + \dfrac{\pi}{4}\bigg)\bigg]^2 + \bigg[\sin \bigg( x -
\dfrac{\pi}{4}\bigg)\bigg]^2\\
& = \bigg[\sin x \cos \dfrac{\pi}{4} + \cos x \sin \dfrac{\pi}{4} \bigg]^2 +
\bigg[\sin x \cos \dfrac{\pi}{4} - \cos x \sin \dfrac{\pi}{4} \bigg]^2 \\
& = \bigg[\dfrac{\sin x}{\sqrt{2}} + \dfrac{\cos}{\sqrt{2}} \bigg]^2
+\bigg[\dfrac{\sin x}{\sqrt{2}} - \dfrac{\cos}{\sqrt{2}} \bigg]^2 \\
& = \bigg[\dfrac{\sin x + \cos x}{\sqrt{2}}\bigg]^2 + \bigg[\dfrac{\sin x - \cos x}
{\sqrt{2}}\bigg]^2 \\
& = \dfrac{\sin^2 x + \cos^2 x + 2\sin x \cos x}{2} + \dfrac{\sin^2 x + \cos^2 x -
2\sin x \cos x}{2}\\
& = \dfrac{1 + 2\sin x \cos x}{2} + \dfrac{1 - 2\sin x \cos x}{2}\\
& = \dfrac{1 + 2\sin x \cos x + 1 - 2\sin x \cos x}{2}\\
& = 1
\end{align*}
Hence Proved.

If $\beta - \alpha = \pi/2$, then $$\beta = \alpha + \pi/2$$\\ Substituting the


value of $\beta$ in the given expression, we get \begin{align*}
\sin (x + \alpha ) + \cos (x + \beta ) & = \sin (x + \alpha ) + \cos (x + \alpha
+ \pi/2)\\
& = \sin (x + \alpha ) + \cos ((x + \alpha) + \pi/2)\\
& = \sin (x + \alpha ) + \cos (x + \alpha) \cos \pi/2 - \sin (x + \alpha) \sin
\pi/2\\
\\
\text{Using}\ \sin \pi/2 = 1\ & \text{and}\ \cos \pi/2 = 0,\ \text{we get}\\
\\
\sin (x + \alpha ) + \cos (x + \beta ) & = \sin (x + \alpha ) + 0 - \sin (x +
\alpha)\\
& = 0
\end{align*}
Hence Proved.

Upon substituting the value of $\beta$ from the given condition into additions of
sines and cosines, we get $$\sin (x + \alpha) + \cos (x + \beta) = 0$$

The given figure is redrawn and labelled as shown below.\\


\\
Now, in $\Delta$ABC, from angle sum property, we have $$\angle BCA = 90^o - \alpha$
$
Similarly, in $\Delta$EFG, from angle sum property, we have $$\angle GEF = 90^o
- \beta$$\\
Now, using vertical angles theorem, we get $$\angle DCE = \angle BCA = 90^o -
\alpha$$ and $$\angle DEC = \angle GEF = 90^o - \beta$$\\
Using information from above and angle sum property for $\Delta$CDE, we get
\begin{align*}
\angle DCE + \angle DEC + \angle CDE & = 180^o\\
90^o - \alpha + 90^o - \beta + \gamma & = 180^o\\
180^o - (\alpha + \beta) + \gamma & = 180^o\\
\alpha + \beta & = \gamma
\end{align*}
Hence Proved.
From the above figure, in $\Delta$ABC, we have $$\tan \alpha = \dfrac{\text{opp}}
{\text{adj}} = \dfrac{4}{6} = \dfrac{2}{3}$$
Similarly, in $\Delta$EFG, we have $$\tan \beta = \dfrac{\text{opp}}{\text{adj}}
= \dfrac{3}{4}$$
Since $\alpha + \beta = \gamma$ from above, we get \begin{align*}
\tan \gamma & = \tan (\alpha + \beta)\\
& = \dfrac{\tan \alpha + \tan \beta}{1 - \tan \alpha \tan \beta}\\
& = \dfrac{\dfrac{2}{3} + \dfrac{3}{4}}{1 - \dfrac{2}{3} \cdot \dfrac{3}{4}}\\
& = \dfrac{\dfrac{17}{12}}{1 - \dfrac{1}{2}}\\
& = \dfrac{\dfrac{17}{12}}{\dfrac{1}{2}}\\
& = \dfrac{17}{6}
\end{align*}

To prove the given identity, let $u = \tan^{-1} x$ and $v = \tan^{-1} y$. Then $$x
= \tan u\ \text{and}\ y = \tan v$$\\
Now using Addition Formula for Tangent, we get \begin{align*}
\tan ( u + v) & = \dfrac{\tan u + \tan v}{1 - \tan u \tan v}\\
\tan (\tan^{-1} x + \tan^{-1} y ) & = \dfrac{x + y}{1 - xy}\\
\tan^{-1} x + \tan^{-1} y & = \tan^{-1} \bigg(\dfrac{x + y}{1 - xy}\bigg)
\end{align*}
Hence identity is proved, i.e. $$\tan^{-1} \bigg(\dfrac{x + y}{1 - xy}\bigg) =
\tan^{-1} x + \tan^{-1} y$$

To prove the given identity, let $u = \tan^{-1} x$ and $v = \tan^{-


1}\bigg(\dfrac{1}{x}\bigg) $. Then $$x = \tan u\ \text{and}\ \dfrac{1}{x} = \tan v$
$\\
Now using Addition Formula for Tangent, we get \begin{align*}
\tan ( u + v) & = \dfrac{\tan u + \tan v}{1 - \tan u \tan v}\\
\text{So,}\ \cot (u + v) = \dfrac{1}{\tan(u + v)} & = \dfrac{1 - \tan u \tan v}
{\tan u + \tan v}\\
\cot \bigg(\tan^{-1} x + \tan^{-1} \bigg(\dfrac{1}{x}\bigg)\bigg) & = \dfrac{1 -
x \cdot \bigg(\dfrac{1}{x}\bigg)}{x + \dfrac{1}{x}}\\
& = 0\\
\tan^{-1}x + \tan^{-1} \bigg(\dfrac{1}{x}\bigg) & = \cot^{-1} 0\\
& = \dfrac{\pi}{2}
\end{align*}
Hence identity is proved, i.e. $$\tan^{-1}x + \tan^{-1} \bigg(\dfrac{1}{x}\bigg) =
\dfrac{\pi}{2}$$

Let $L$ be a line in the plane and $\theta$ be the angle formed by the line and the
$x$-axis as shown in the below figure. Let the line pass through the points $(a,0)$
and $(0,b)$ where $b<0$ on the $x$ and $y$ axis respectively.\\
\\
We know that equation of a line in its slope-intercept form is $$y = mx + c$$ where
$m$ is the slope of the line. \\
\\
Since line passes through the points, $(a,0)$ and $(0,b)$, upon substituting the
points in the line equation, we get \begin{align*}
\text{For}\ (a,0)\ \Rightarrow 0 & = ma + c\\
c & = -ma\\
\\
\text{For}\ (0,b)\ \Rightarrow b & = 0 + c\\
c & = b
\end{align*}
Therefore, from above, we get $$-ma = b\ \Rightarrow m = -\dfrac{b}{a}$$
Now, from the above figure, in the triangle, we have \begin{align*}
\tan \theta & = \dfrac{\text{opp}}{\text{adj}}\\
& = \dfrac{b}{a}
\end{align*}\\
Since $b < 0$, we get $\tan \theta = -\dfrac{b}{a}$ which is same as the slope $m$
of the line.\\
\\
Therefore, we can say that $$\boxed{m = \tan \theta}$$

In the given figure, $L_1$ and $L_2$ are two nonparallel lines in the plane with
slopes $m_1$ and $m_2$ respectively. Also, $\psi$ is the acute angle formed by the
two lines. \\
\\
From previous exercise, we showed that if $\theta$ is the angle formed by the line
of slope $m$ with the $x$-axis, then $$m = \tan \theta$$
So, for the line $L_1$ which makes angle $\theta_1$ with the $x$-axis, we have $
$m_1 = \tan \theta_1$$
Similarly, for the line $L_2$ which makes angle $\theta_2$ with the $x$-axis, we
have $$m_2 = \tan \theta_2$$\\
Now, since $\psi = \theta_2 - \theta_1$, using subtraction formula for tangent, we
get \begin{align*}
\tan \psi & = \tan (\theta_2 - \theta_1)\\
& = \dfrac{\tan \theta_2 - \tan \theta_1}{1 + \tan \theta_2 \tan \theta_1}\\
& = \dfrac{m_2 - m_1}{1 + m_2 m_1}
\end{align*}
Hence Proved.

Given two lines $$y = \dfrac{1}{3}x + 1\ \ \ \text{and}\ \ \ y = \dfrac{1}{2} x -3$


$
Since $y = mx + c$ is the equation of a line in its slope-intercept form where $m$
is the slope of the line, for the above two lines, we have slopes $$m_1 = \dfrac{1}
{3}\ \ \ \text{and}\ \ \ m_2 = \dfrac{1}{2}$$\\
From previous exercise, we saw that if $\psi$ is the acute angle formed between two
nonparallel lines of slope $m_1$ and $m_2$, then $$\tan \psi = \dfrac{m_2 - m_1}{1
+ m_2 m_1}$$
If $\psi$ is the angle formed by the two lines, then \begin{align*}
\tan \psi & = \dfrac{\dfrac{1}{2} - \dfrac{1}{3}}{1 + \dfrac{1}{2} \cdot \dfrac{1}
{3}}\\
& = \dfrac{\dfrac{1}{6}}{1 + \dfrac{1}{6}}\\
& = \dfrac{\dfrac{1}{6}}{\dfrac{7}{6}}\\
& = \dfrac{1}{7}\\
\psi & = \tan^{-1} \bigg(\dfrac{1}{7}\bigg)\\
& = 8.13^o
\end{align*}

From earlier exercise, we saw that if $\psi$ is the acute angle formed between two
nonparallel lines of slope $m_1$ and $m_2$, then $$\tan \psi = \dfrac{m_2 - m_1}{1
+ m_2 m_1}$$
So, from reciprocal identities, we have $$\cot \psi = \dfrac{1 + m_2 m_1}{m_2 -
m_1}$$\\
Now, if two lines are perpendicular, then the angle formed by the lines will be
equal to $\pi/2$. So, we get \begin{align*}
\cot \psi & = \dfrac{1 + m_2 m_1}{m_2 - m_1}\\
\cot \dfrac{\pi}{2} & = \dfrac{1 + m_2 m_1}{m_2 - m_1}\\
0 & = \dfrac{1 + m_2 m_1}{m_2 - m_1}\\
1 + m_2 m_1 & = 0\\
m_1 m_2 & = -1\\
m_1 & = -\dfrac{1}{m_2}
\end{align*}
Therefore, we see that if two lines are perpendicular, then the slope of one is
negative reciprocal of the slope of the other.

The figure is labelled and redrawn as shown below.\\


\\
In $\Delta$PTQ and $\Delta$PSQ, we have \begin{align*}
\tan A & = \dfrac{\text{opp}}{\text{adj}} = \dfrac{PQ}{TQ} = \dfrac{1}{3}\\
\\
\tan B & = \dfrac{\text{opp}}{\text{adj}} = \dfrac{PQ}{SQ} = \dfrac{1}{2}
\end{align*}\\
Using Addition formula for tangent, we get \begin{align*}
\tan (A + B) & = \dfrac{\tan A + \tan B}{1 - \tan A \tan B}\\
& = \dfrac{\dfrac{1}{3} + \dfrac{1}{2}}{1 - \dfrac{1}{3} \cdot \dfrac{1}{2}}\\
& = \dfrac{\dfrac{5}{6}}{1 - \dfrac{1}{6}}\\
& = \dfrac{\dfrac{5}{6}}{\dfrac{5}{6}}\\
& = 1\\
\text{Thus}\ A + B & = \tan^{-1} 1\\
& = 45^o
\end{align*}
Now, in $\Delta$PRQ, we have \begin{align*}
\tan C & = \dfrac{\text{opp}}{\text{adj}}\\
& = \dfrac{PQ}{RQ}\\
& = \dfrac{1}{1}\\
& = 1\\
\text{Thus}\ C & = \tan^{-1} 1\\
& = 45^o
\end{align*}\\
Therefore, we have $$\angle A + \angle B + \angle C = 45^o + 45^o = 90^o$$

(a)\ The two sound waves interfere to produce a single sound modeled as $$f(t) =
C \sin \omega t + C\sin(\omega t + \alpha)$$\\
Using Addition Formula for Sine, the above function can be expanded as
\begin{align*}
f(t) & = C \sin \omega t + C\sin(\omega t + \alpha)\\
& = C \sin \omega t + C( \sin \omega t \cos \alpha + \cos \omega t \sin \alpha)\\
& = C \sin \omega t + C \sin \omega t \cos \alpha + C \cos \omega t \sin \alpha\\
& = C \sin \omega t(1 + \cos \alpha) + C \cos \omega t \sin \alpha\\
& = (C + C \cos \alpha)\sin \omega t + (C \sin \alpha) \cos \omega t
\end{align*}\\
The RHS above is of the form of $A \sin \omega t + B \cos \omega t$ where $$A = C +
C \cos \alpha\ \ \ \text{and}\ \ \ B = C \sin \alpha$$\\
\\
(b)\ When $C = 10$ and $\alpha = \pi/3$, the constants $A$ and $B$ changes to $$A =
10 + 10 \cos \pi/3 = 10 + 10 \bigg(\dfrac{1}{2}\bigg) = 10 + 5 = 15$$
and $$B = 10 \sin \pi/3 = 10 \bigg(\dfrac{\sqrt{3}}{2}\bigg) = 5\sqrt{3}$$
Therefore the modeled function becomes $$f(t) = 15 \sin \omega t + 5\sqrt{3}
\cos \omega t$$

From Sum of Sines And Cosines, we know that if $A$ and $B$ are real numbers, then $
$A \sin x + B \cos x = k\sin (x + \phi)$$
where $k = \sqrt{A^2 + B^2}$ and $\phi$ satisfies $$\cos \phi = \dfrac{A}{\sqrt{A^2
+ B^2}}\ \ \ \ \text{and}\ \ \ \ \sin \phi = \dfrac{B}{\sqrt{A^2 + B^2}}$$\\
\\
Comparing the modeled function with the above theorem, we have $$A = 15\
\text{and}\ B = 5\sqrt{3}, \ \ \Rightarrow \ \ k = \sqrt{(15)^2 + (5\sqrt{3})^2} =
10\sqrt{3}$$
and \begin{align*}
\cos \phi & = \dfrac{15}{10\sqrt{3}} = \dfrac{\sqrt{3}}{2}\\
\phi & = \cos^{-1} \bigg(\dfrac{\sqrt{3}}{2}\bigg)\\
& = \dfrac{\pi}{6}\ \text{or}\ \dfrac{11\pi}{6}\\
\\
\sin \phi & = \dfrac{5\sqrt{3}}{10\sqrt{3}}\\
\phi & = \sin^{-1} \bigg(\dfrac{1}{2}\bigg)\\
& = \dfrac{\pi}{6}\ \text{or}\ \dfrac{5\pi}{6}
\end{align*}\\
From the above two cases, we get $\phi = \dfrac{\pi}{6}$.\\
\\
Therefore, the given expression can be written in terms of sine only as
$$f(t) = 10\sqrt{3} \sin \bigg(wt + \dfrac{\pi}{6}\bigg)$$

Let's use the first cofunction identity $$\sin x = \cos \bigg(\dfrac{\pi}{2} -


x\bigg)$$ to write $$\sin (s + t) = \cos \bigg(\dfrac{\pi}{2} - (s + t)\bigg)$$\\
Using Subtraction Formula for Cosine, we can write it as \begin{align*}
\sin (s + t) & = \cos \bigg(\dfrac{\pi}{2} - (s + t)\bigg)\\
& = \cos \bigg(\bigg(\dfrac{\pi}{2} - s\bigg) - t \bigg)\\
& = \cos \bigg(\dfrac{\pi}{2} - s\bigg) \cos t + \sin \bigg(\dfrac{\pi}{2} -
s\bigg) \sin t\\
& = \sin s \cos t + \cos s \sin t\\
\end{align*}
Therefore, Addition Formula for Sine is $$\sin (s + t) = \sin s \cos t + \cos s
\sin t$$

Using reciprocal identities, we can write $\tan (s + t)$ as $$\tan (s +t) =


\dfrac{\sin (s+ t)}{\cos (s+ t)}$$\\
Now, using the Addition Formulas for Sine and Cosine, we get \begin{align*}
\tan (s + t) & = \dfrac{\sin (s+ t)}{\cos (s+ t)}\\
& = \dfrac{\sin s \cos t + \cos s \sin t}{\cos s \cos t - \sin s \sin t}\\
& = \dfrac{\dfrac{\sin s \cos t + \cos s \sin t}{\cos s \cos t}}{\dfrac{\cos s \cos
t - \sin s \sin t}{{\cos s \cos t}}}\\
& = \dfrac{\dfrac{\sin s \cos t}{\cos s \cos t} + \dfrac{\cos s \sin t}{\cos s \cos
t}}{\dfrac{\cos s \cos t}{\cos s \cos t} - \dfrac{\sin s \sin t}{{\cos s \cos
t}}}\\
& = \dfrac{\tan s + \tan t}{1 - \tan s \tan t}\\
\end{align*}
Therefore, the Addition Formula for Tangent is $$\tan (s+ t) = \dfrac{\tan s + \tan
t}{1 - \tan s \tan t}$$
If we know the values of $\sin x$ and $\cos x$, we can find the value of $\sin 2x$
by using the $\mathbf{Double}-\mathbf{Angle}$ Formula for Sine.\\
\\
Also, the formula for $\sin 2x$ is $$\sin 2x = \bold{2 \sin x \cos x}$$

If we know the value of $\cos x$ and the quadrant in which $x/2$ lies, we can find
the value of $\sin (x/2)$ by using the $\mathbf{Half}-\mathbf{Angle}$ Formula for
Sine.\\
\\
Also, the formula for $\sin (x/2)$ is $$\sin (x/2) = \pm \sqrt{(1 - \cos x)/2}$$

$$\csc x = 4,\ \ \ \tan x < 0$$\\


Using reciprocal identities, we have $$\sin x = \dfrac{1}{\csc x} = \dfrac{1}{4}$$
Now, $\tan x$ is negative in Quadrants II and IV and $\sin x$ is positive in
Quadrants I and II. Therefore, $x$ lies in Quadrant II. \\
\\
Since $\cos x$ is negative in Quadrant II, we get $$\cos x = -\sqrt{1 - \sin^2 x} =
-\sqrt{1 - \dfrac{1}{16}} = -\dfrac{\sqrt{15}}{4}$$\\
Now, \begin{align*}
\sin 2x & = 2 \sin x \cos x = 2 \bigg(\dfrac{1}{4}\bigg) \bigg(-\dfrac{\sqrt{15}}
{4}\bigg) = -\dfrac{\sqrt{15}}{8}\\
\cos 2x & = 1 - 2 \sin^2 x = 1 - 2 \bigg(\dfrac{1}{4}\bigg)^2 = \dfrac{7}{8}\\
\tan 2x & = \dfrac{\sin 2x}{\cos 2x} = \dfrac{-\dfrac{\sqrt{15}}{8}}{\dfrac{7}{8}}
= -\dfrac{\sqrt{15}}{7}
\end{align*}

$$\cos x = \dfrac{4}{5},\ \ \ \csc x < 0$$\\


Now, $\csc x$ is negative in Quadrants III and IV and $\cos x$ is positive in
Quadrants I and IV. Therefore, $x$ lies in Quadrant IV. \\
\\
Since $\sin x$ is negative in Quadrant IV, we get $$\sin x = -\sqrt{1 - \cos^2 x} =
-\sqrt{1 - \dfrac{16}{25}} = -\dfrac{3}{5}$$\\
Now, \begin{align*}
\sin 2x & = 2 \sin x \cos x\ \ \ \ \ \ \ \ \ \ \ \ \ \ \ \ [\ \text{Double Angle
Formula}\ ]\\
& = 2 \bigg(-\dfrac{3}{5}\bigg) \bigg(\dfrac{\sqrt{4}}{5}\bigg)\\
& = -\dfrac{24}{25}\\
\cos 2x & = 2 \cos^2 x - 1\ \ \ \ \ \ \ \ \ \ \ \ \ \ \ [\ \text{Double Angle
Formula}\ ]\\
& = 2 \bigg(\dfrac{4}{5}\bigg)^2 - 1\\
& = \dfrac{7}{25}\\
\tan 2x & = \dfrac{\sin 2x}{\cos 2x}\ \ \ \ \ \ \ \ \ \ \ \ \ \ \ \ \ \ \ \ \ [\
\text{Reciprocal identities}\ ]\\
& = \dfrac{-\dfrac{24}{25}}{\dfrac{7}{25}}\\
& = -\dfrac{24}{7}
\end{align*}

$$\sin x = -\dfrac{3}{5},\ \ \ x\ \text{in Quadrant III}$$\\


Since $\cos x$ is negative in Quadrant III, we get $$\cos x = -\sqrt{1 - \sin^2 x}
= -\sqrt{1 - \dfrac{9}{25}} = -\dfrac{4}{5}$$\\
Now, \begin{align*}
\sin 2x & = 2 \sin x \cos x\ \ \ \ \ \ \ \ \ \ \ \ \ \ \ \ [\ \text{Double Angle
Formula}\ ]\\
& = 2 \bigg(-\dfrac{3}{5}\bigg) \bigg(-\dfrac{4}{5}\bigg)\\
& = \dfrac{24}{25}\\
\cos 2x & = 1 - 2 \sin^2 x\ \ \ \ \ \ \ \ \ \ \ \ \ \ \ \ \ [\ \text{Double Angle
Formula}\ ]\\
& = 1 - 2 \bigg(-\dfrac{3}{5}\bigg)^2\\
& = \dfrac{7}{25}\\
\tan 2x & = \dfrac{\sin 2x}{\cos 2x}\ \ \ \ \ \ \ \ \ \ \ \ \ \ \ \ \ \ \ \ \ \ \
[\ \text{Reciprocal identities}\ ]\\
& = \dfrac{\dfrac{24}{25}}{\dfrac{7}{25}}\\
& = \dfrac{24}{7}
\end{align*}

$$\sec x = 2,\ \ \ x\ \text{in Quadrant IV}$$\\


Using reciprocal identities, we have $$\cos x = \dfrac{1}{\sec x} = \dfrac{1}{2}$
$\\
Since $\sin x$ is negative in Quadrant IV, we get $$\sin x = -\sqrt{1 - \cos^2 x} =
-\sqrt{1 - \dfrac{1}{4}} = -\dfrac{\sqrt{3}}{2}$$\\
Now, \begin{align*}
\sin 2x & = 2 \sin x \cos x\ \ \ \ \ \ \ \ \ \ \ \ \ \ \ \ [\ \text{Double Angle
Formula}\ ]\\
& = 2 \bigg(-\dfrac{\sqrt{3}}{2}\bigg) \bigg(\dfrac{1}{2}\bigg)\\
& = -\dfrac{\sqrt{3}}{2}\\
\cos 2x & = 2 \cos^2 x - 1\ \ \ \ \ \ \ \ \ \ \ \ \ \ \ \ \ [\ \text{Double Angle
Formula}\ ]\\
& = 2 \bigg(\dfrac{1}{2}\bigg)^2 - 1\\
& = -\dfrac{1}{2}\\
\tan 2x & = \dfrac{\sin 2x}{\cos 2x}\ \ \ \ \ \ \ \ \ \ \ \ \ \ \ \ \ \ \ \ \ \ \
[\ \text{Reciprocal identities}\ ]\\
& = \dfrac{-\dfrac{\sqrt{3}}{2}}{-\dfrac{1}{2}}\\
& = \sqrt{3}
\end{align*}

We use the formulas for lowering powers repeatedly. \begin{align*}


\cos^4 x \sin^2 x & = \sin^2 x \cos^2 x \cos^2 x\\
& = (\sin x \cos x)^2 \cos^2 x\\
& = \bigg(\dfrac{1}{2} \sin 2x\bigg)^2\bigg(\dfrac{1 + \cos 2x}{2}\bigg)\\
& = \bigg(\dfrac{1}{4}\sin^2 2x\bigg)\bigg(\dfrac{1 + \cos 2x}{2}\bigg)\\
& = \dfrac{1}{8}\bigg( \dfrac{1 - \cos 4x}{2}\bigg)(1 + \cos 2x)\\
& = \dfrac{1}{16}(1 - \cos 4x)(1 + \cos 2x)\\
& = \dfrac{1}{16}(1 + \cos 2x - \cos 4x - \cos 2x \cos 4x)\\
\end{align*}

(b)\ $2 \sin 3\theta \cos 3\theta$\\


\\
Using Double-Angle Formula for Sine, i.e. $$\sin 2x = 2 \sin x \cos x$$
let $x = 3\theta$, then $$2 \sin 3\theta \cos 3\theta = \sin 2(3\theta) = \sin
6\theta$$
(a)\ $\dfrac{2 \tan 7^o}{1 - \tan^2 7^o}$\\
\\
Using Double-Angle Formula for Tangent, i.e. $$\tan 2x = \dfrac{2 \tan x}{1 -
\tan^2 x}$$
let $x = 7^o$, then $$\dfrac{2 \tan 7^o}{1 - \tan^2 7^o} = \tan 2(7^o) = \tan
14^o$$

(b)\ $\cos^2 5\theta - \sin^2 5\theta$\\


\\
Using Double-Angle Formula for Cosine, i.e. $$\cos 2x = \cos^2 x - \sin^2 x$$
let $x = 5\theta$, then $$\cos^2 5\theta - \sin^2 5\theta = \cos 2(5\theta) = \cos
10\theta$$

$\text{(a)} \cos^2 \dfrac{\theta}{2} - \sin^2 \dfrac{\theta}{2}$\\


\\
\\
Using Double-Angle Formula for Cosine, i.e. $$\cos 2x = \cos^2 x - \sin^2 x$$ and
Formula for lowering powers, i.e. $$\cos x = \cos^2 \dfrac{x}{2} - \sin^2 \dfrac{x}
{2}$$
let $x = \theta$, then $$\boxed{\cos^2 \dfrac{\theta}{2} - \sin^2 \dfrac{\theta}{2}
= \cos \theta}$$

$\text{(b)}\ 2\sin \dfrac{\theta}{2} \cos \dfrac{\theta}{2}$\\


\\
\\
Using Double-Angle Formula for Sine, i.e. $$\sin 2x = 2\sin x \cos x$$ and Formula
for lowering powers, i.e. $$\sin x = 2\sin \dfrac{x}{2} \cos \dfrac{x}{2}$$
let $x = \theta$, then $$\boxed{2\sin \dfrac{\theta}{2} \cos \dfrac{\theta}{2} =
\sin \theta}$$

(b)\ $\dfrac{1 - \cos 4\theta}{\sin 4\theta}$\\


\\
\\
Using Half-Angle Formula for Tangent, i.e. $$\tan \dfrac{u}{2} = \dfrac{1 - \cos u}
{\sin u}$$
let $u = 4\theta$, then $$\dfrac{1 - \cos 4\theta}{\sin 4\theta} = \tan
\dfrac{4\theta}{2}^o = \tan 2\theta$$

(a)\ $\dfrac{\sin 8^o}{1 + \cos 8^o}$\\


\\
\\
Using Half-Angle Formula for Tangent, i.e. $$\tan \dfrac{u}{2} = \dfrac{\sin u}{1 +
u}$$
let $u = 8^o$, then $$\dfrac{\sin 8^o}{1 + \cos 8^o} = \tan \dfrac{8}{2}^o = \tan
4^o$$
(b)\ $\sqrt{\dfrac{1 - \cos 8\theta}{2}}$\\
\\
\\
Using Half-Angle Formula for Sine, i.e. $$\sin \dfrac{u}{2} = \pm \sqrt{\dfrac{ 1 -
\cos u}{2}}$$ where the choice of $+$ or $-$ sign depends on the quadrant in which
$u/2$ lies.\\
\\
Let $u = 8\theta$, then $$\sqrt{\dfrac{1 - \cos 8\theta}{2}} = \sin
\dfrac{8\theta}{2} = \sin 4\theta$$

We know that the Addition Formula for Sine is $$\sin (x + y) = \sin x \cos y + \cos
x \sin y$$\\ When $y = x$, i.e. the angle gets doubled, then \begin{align*}
\sin (x + x) & = \sin x \cos x + \cos x \sin x\\
\sin 2x & = 2 \sin x \cos x
\end{align*}
which is Double-Angle Formula for Sine.\\
\\
Hence Proved

$$\sin x = \dfrac{3}{5},\ \ \ 0^o < x < 90^o$$\\


Since $\cos x$ is positive in Quadrant I, we get $$\cos x = \sqrt{1 - \sin^2 x}
= \sqrt{1 - \dfrac{9}{25}} = \dfrac{4}{5}$$\\
Now, \begin{align*}
\sin \dfrac{x}{2} & = \pm \sqrt{\dfrac{ 1 - \cos x}
{2}}\ \ \ \ \ \ \ \ \ \ \ \ \ \ \ \ [\ \text{Half Angle Formula}\ ]\\
& = \sqrt{\dfrac{ 1 - \dfrac{4}{5}}
{2}}\ \ \ \ \ \ \ \ \ \ \ \ \ \ \ \ \ \ \ \ \ \bigg[\ \text{Since}\ \sin \dfrac{x}
{2} > 0,\ \text{for}\ 0^o < \dfrac{x}{2} < 45^o\ \bigg]\\
& = \sqrt{\dfrac{1}{10}}\\
\\
\cos \dfrac{x}{2} & = \pm \sqrt{\dfrac{ 1 + \cos x}
{2}}\ \ \ \ \ \ \ \ \ \ \ \ \ \ \ \ \ [\ \text{Half Angle Formula}\ ]\\
& = \sqrt{\dfrac{ 1 + \dfrac{4}{5}}
{2}}\ \ \ \ \ \ \ \ \ \ \ \ \ \ \ \ \ \ \ \ \ \ \bigg[\ \text{Since}\ \cos
\dfrac{x}{2} > 0,\ \text{for}\ 0^o < \dfrac{x}{2} < 45^o\ \bigg]\\
& = \sqrt{\dfrac{9}{10}}\\
& = \dfrac{3}{\sqrt{10}}\\
\\
\tan \dfrac{x}{2} & = \dfrac{\sin \dfrac{x}{2}}{\cos \dfrac{x}{2}}\ \ \ \ \ \ \ \ \
\ \ \ \ \ \ \ \ \ \ \ \ \ \ [\ \text{Reciprocal identities}\ ]\\
& = \dfrac{\sqrt{\dfrac{1}{10}}}{\dfrac{3}{\sqrt{10}}}\\
& = \dfrac{1}{3}
\end{align*}

$$\cos x = -\dfrac{4}{5},\ \ \ 180^o < x < 270^o$$\\


Now, \begin{align*}
\sin \dfrac{x}{2} & = \pm \sqrt{\dfrac{ 1 - \cos x}
{2}}\ \ \ \ \ \ \ \ \ \ \ \ \ \ \ \ [\ \text{Half Angle Formula}\ ]\\
& = \sqrt{\dfrac{ 1 - \bigg(-\dfrac{4}{5}\bigg)}{2}}\ \ \ \ \ \ \ \ \ \ \ \ \ \
\bigg[\ \text{Since}\ \sin \dfrac{x}{2} > 0,\ \text{for}\ 90^o < \dfrac{x}{2} <
135^o\ \bigg]\\
& = \dfrac{3}{\sqrt{10}}\\
\\
\cos \dfrac{x}{2} & = \pm \sqrt{\dfrac{ 1 + \cos x}
{2}}\ \ \ \ \ \ \ \ \ \ \ \ \ \ \ \ \ [\ \text{Half Angle Formula}\ ]\\
& = - \sqrt{\dfrac{ 1 + \bigg(-\dfrac{4}{5}\bigg)}{2}}\ \ \ \ \ \ \ \ \ \ \ \
\bigg[\ \text{Since}\ \cos \dfrac{x}{2} < 0,\ \text{for}\ 90^o < \dfrac{x}{2} <
135^o\ \bigg]\\
& = - \sqrt{\dfrac{1}{10}}\\
& = - \dfrac{1}{\sqrt{10}}\\
\\
\tan \dfrac{x}{2} & = \dfrac{\sin \dfrac{x}{2}}{\cos \dfrac{x}{2}}\ \ \ \ \ \ \ \ \
\ \ \ \ \ \ \ \ \ \ \ \ \ \ \ \ \ [\ \text{Reciprocal identities}\ ]\\
& = \dfrac{\dfrac{3}{\sqrt{10}}}{-\sqrt{\dfrac{1}{10}}}\\
& = -3
\end{align*}

\begin{align*}
\cos \dfrac{x}{2} & = \pm \sqrt{\dfrac{ 1 + \cos x}{2}}\ \ \ \ \ \ \ \ \ \ \ \ \ \
\ \ \ \ \ \ \ \ \ \ [\ \text{Half Angle Formula}\ ]\\
& = \sqrt{\dfrac{ 1 + \bigg(-\dfrac{2\sqrt{2}}{3}\bigg)}{2}}\ \ \ \ \ \ \ \ \ \ \ \
\ \ \ \ \ \ \bigg[\ \text{Since}\ \cos \dfrac{x}{2} > 0,\ \text{for}\ 45^o <
\dfrac{x}{2} < 90^o\ \bigg]\\
& = \sqrt{\dfrac{\dfrac{3-2\sqrt{2}}{3}}{2}}\\
& = \sqrt{\dfrac{3-2\sqrt{2}}{6}}\\
\\
\tan \dfrac{x}{2} & = \dfrac{\sin \dfrac{x}{2}}{\cos \dfrac{x}{2}}\ \ \ \ \ \ \ \ \
\ \ \ \ \ \ \ \ \ \ \ \ \ \ \ \ \ \ \ \ \ \ \ \ \ \ \ [\ \text{Reciprocal
identities}\ ]\\
& = \dfrac{\sqrt{\dfrac{3+2\sqrt{2}}{6}}}{\sqrt{\dfrac{3-2\sqrt{2}}{6}}}\\
& = \sqrt{\dfrac{3+2\sqrt{2}}{3-2\sqrt{2}}}\\
& = \sqrt{\dfrac{3+2\sqrt{2}}{3-2\sqrt{2}} \times \dfrac{3+2\sqrt{2}}
{3+2\sqrt{2}}}\ \ \ \ \ \ \ \ \ \ \ \ \ \ \ [\ \text{Rationalizing}\ ]\\
& = \sqrt{\dfrac{(3+2\sqrt{2})^2}{9-8}}\\
& = 3+2\sqrt{2}
\end{align*}

$$\csc x = 3,\ \ \ 90^o < x < 180^o$$\\


Using reciprocal identities, we get $$\sin x = \dfrac{1}{\csc x} = \dfrac{1}{3}$$
Since $\cos x$ is negative in Quadrant II, we get $$\cos x = -\sqrt{1 - \sin^2 x} =
-\sqrt{1 - \dfrac{1}{9}} = -\dfrac{2\sqrt{2}}{3}$$\\
\\
Now, \begin{align*}
\sin \dfrac{x}{2} & = \pm \sqrt{\dfrac{ 1 - \cos x}
{2}}\ \ \ \ \ \ \ \ \ \ \ \ \ \ \ \ \ \ \ \ \ \ \ \ [\ \text{Half Angle
Formula}\ ]\\
& = \sqrt{\dfrac{ 1 - \bigg(-\dfrac{2\sqrt{2}}{3}\bigg)}{2}}\ \ \ \ \ \ \ \ \ \ \ \
\ \ \ \ \ \ \bigg[\ \text{Since}\ \sin \dfrac{x}{2} > 0,\ \text{for}\ 45^o <
\dfrac{x}{2} < 90^o\ \bigg]\\
& = \sqrt{\dfrac{\dfrac{3+2\sqrt{2}}{3}}{2}}\\
& = \sqrt{\dfrac{3+2\sqrt{2}}{6}}\\
\end{align*}
\begin{align*}
\cos \dfrac{x}{2} & = \pm \sqrt{\dfrac{ 1 + \cos x}
{2}}\ \ \ \ \ \ \ \ \ \ \ \ \ \ \ \ \ [\ \text{Half Angle Formula}\ ]\\
& = -\sqrt{\dfrac{ 1 + \dfrac{2}{3}}
{2}}\ \ \ \ \ \ \ \ \ \ \ \ \ \ \ \ \ \ \ \ \bigg[\ \text{Since}\ \cos \dfrac{x}{2}
< 0,\ \text{for}\ 135^o < \dfrac{x}{2} < 180^o\ \bigg]\\
& = - \sqrt{\dfrac{5}{6}}\\
& = -\dfrac{\sqrt{30}}{6}\ \ \ \ \ \ \ \ \ \ \ \ \ \ \ \ \ \ \ \ \ \ \ \ \ \ [\
\text{Upon rationalizing}\ ]\\
\\
\tan \dfrac{x}{2} & = \dfrac{\sin \dfrac{x}{2}}{\cos \dfrac{x}{2}}\ \ \ \ \ \ \ \ \
\ \ \ \ \ \ \ \ \ \ \ \ \ \ \ \ \ [\ \text{Reciprocal identities}\ ]\\
& = \dfrac{\dfrac{\sqrt{6}}{6}}{-\dfrac{\sqrt{30}}{6}}\\
& = -\dfrac{1}{\sqrt{5}}\\
& = -\dfrac{\sqrt{5}}{5}\ \ \ \ \ \ \ \ \ \ \ \ \ \ \ \ \ \ \ \ \ \ \ \ \ \ [\
\text{Upon rationalizing}\ ]\\
\end{align*}\\

$$\sec x = \dfrac{3}{2},\ \ \ 270^o < x < 360^o$$\\


Using reciprocal identities, we get $$\cos x = \dfrac{1}{\sec x} = \dfrac{2}{3}$$\\
Now, \begin{align*}
\sin \dfrac{x}{2} & = \pm \sqrt{\dfrac{ 1 - \cos x}
{2}}\ \ \ \ \ \ \ \ \ \ \ \ \ \ \ [\ \text{Half Angle Formula}\ ]\\
& = \sqrt{\dfrac{ 1 - \dfrac{2}{3}}{2}}\ \ \ \ \ \ \ \ \ \ \ \ \ \ \ \ \ \ \ \
\bigg[\ \text{Since}\ \sin \dfrac{x}{2} > 0,\ \text{for}\ 135^o < \dfrac{x}{2} <
180^o\ \bigg]\\
& = \dfrac{1}{\sqrt{6}}\\
& = \dfrac{\sqrt{6}}{6} \ \ \ \ \ \ \ \ \ \ \ \ \ \ \ \ \ \ \ \ \ \ \ \ \ \ \ [\
\text{Upon rationalizing}\ ]
\end{align*}

$$\sin (2\tan^{-1} x)$$\\


Let $\theta = \tan^{-1} x$, then $\tan \theta = x$. Sketch a triangle as shown in
the below figure. Now, we need to find $\sin 2 \theta$, but from the triangle we
can find trigonometric functions of $\theta$ only, not $2 \theta$.\\
\\
So, we use the Double-Angle Formula for Sine. \begin{align*}
\sin (2 \tan^{-1} x) & = \sin 2 \theta\ \ \ \ \ \ \ \ \ \ \ \ \ \ \ \ \ \ \ \ \ \ \
\ \ \ \ \ \ \ \ \ \ \ \ \ \ [\ \text{Since}\ \tan^{-1} x = \theta\ ]\\
& = 2 \sin \theta \cos \theta \ \ \ \ \ \ \ \ \ \ \ \ \ \ \ \ \ \ \ \ \ \ \ \ \ \ \
\ \ \ [\ \text{Double-Angle Formula}\ ]\\
& = 2 \bigg(\dfrac{x}{\sqrt{1 + x^2}}\bigg)\bigg(\dfrac{1}{\sqrt{1 +
x^2}}\bigg) \ \ \ \ \ \ \ \ \ \ \ [\ \text{From the triangle}\ ]\\
& = \dfrac{2x}{1 + x^2}
\end{align*}

$$\tan (2\cos^{-1} x)$$\\


Let $\theta = \cos^{-1} x$, then $\cos \theta = x$. Sketch a triangle as shown in
the below figure. Now, we need to find $\sin 2 \theta$, but from the triangle we
can find trigonometric functions of $\theta$ only, not $2 \theta$.\\
\\
So, we use the Double-Angle Formula for Tangent. \begin{align*}
\tan (2 \cos^{-1} x) & = \tan 2 \theta\ \ \ \ \ \ \ \ \ \ \ \ \ \ \ \ \ \ \ \ \ \ \
\ \ \ \ \ \ \ \ \ \ \ \ \ [\ \text{Since}\ \cos^{-1} x = \theta\ ]\\
& = \dfrac{2 \tan \theta}{1 - \tan^2
\theta} \ \ \ \ \ \ \ \ \ \ \ \ \ \ \ \ \ \ \ \ \ \ \ \ \ \ \ \ \ \ [\
\text{Double-Angle Formula}\ ]\\
& = \dfrac{2 \bigg(\dfrac{\sqrt{1 - x^2}}{x}\bigg)}{1 - \bigg(\dfrac{\sqrt{1 -
x^2}}{x}\bigg)^2} \ \ \ \ \ \ \ \ \ \ \ \ \ \ \ \ \ \ \ \ \ [\ \text{From the
triangle}\ ]\\
\\
& = \dfrac{\dfrac{2\sqrt{1 - x^2}}{x}}{\dfrac{x^2 - 1 + x^2}{x^2}} =
\dfrac{\dfrac{2\sqrt{1 - x^2}}{x}}{\dfrac{2x^2 - 1}{x^2}} = \dfrac{2x\sqrt{1 -
x^2}}{2x^2 - 1}
\end{align*}

$$\sin \bigg(\dfrac{1}{2}\cos^{-1} x\bigg)$$\\


Let $\theta = \cos^{-1} x$, then $\cos \theta = x$. Sketch a triangle as shown in
the below figure. Now, we need to find $\sin \dfrac{\theta}{2}$, but from the
triangle we can find trigonometric functions of $\theta$ only, not $\dfrac{\theta}
{2}$.\\
\\
So, we use the Half-Angle Formula for Sine. \begin{align*}
\sin \bigg(\dfrac{1}{2}\cos^{-1} x\bigg) & = \sin \dfrac{\theta}
{2}\ \ \ \ \ \ \ \ \ \ \ \ \ \ \ \ \ \ \ \ \ \ \ \ \ \ \ \ \ [\ \text{Since}\
\cos^{-1} x = \theta\ ]\\
& = \pm \sqrt{\dfrac{ 1- \cos \theta}{2}} \ \ \ \ \ \ \ \ \ \ \ \ \ \ \ \ \ \ [\
\text{Half-Angle Formula}\ ]\\
& = \pm \sqrt{\dfrac{ 1- x}{2}} \ \ \ \ \ \ \ \ \ \ \ \ \ \ \ \ \ \ \ \ \ \ [\
\text{From the triangle}\ ]
\end{align*}\\
Since $x \in [-1,1]$, $1 - x$ will be positive for all values of $x$ and therefore,
$$\sin \bigg(\dfrac{1}{2}\cos^{-1} x\bigg) = \sqrt{\dfrac{ 1- x}{2}}$$

$$\cos (2\sin^{-1} x)$$\\


Let $\theta = \sin^{-1} x$, then $\sin \theta = x$. Sketch a triangle as shown in
the below figure. Now, we need to find $\cos 2 \theta$, but from the triangle we
can find trigonometric functions of $\theta$ only, not $2 \theta$.\\
\\
So, we use the Double-Angle Formula for Tangent. \begin{align*}
\cos (2 \sin^{-1} x) & = \cos 2 \theta\ \ \ \ \ \ \ \ \ \ \ \ \ \ \ \ \ \ \ \ \ \ \
\ \ \ [\ \text{Since}\ \sin^{-1} x = \theta\ ]\\
& = 1 - 2 \sin^2 \theta \ \ \ \ \ \ \ \ \ \ \ \ \ \ \ \ \ \ \ [\ \text{Double-Angle
Formula}\ ]\\
& = 1 - 2x^2 \ \ \ \ \ \ \ \ \ \ \ \ \ \ \ \ \ \ \ \ \ \ \ \ [\ \text{From the
triangle}\ ]\\
\end{align*}

$$\sin \bigg(2\cos^{-1} \frac{7}{25}\bigg)$$\\


Let $\theta = \cos^{-1} \frac{7}{25}$, then $\cos \theta = \frac{7}{25}$. We first
sketch the angle $\theta$ in standard position with terminal side in Quadrant I as
shown in the below figure. Since $\cos \theta = \frac{x}{r} = \frac{7}{25}$, we can
label a side and the hypotenuse of the triangle in the below figure. To find the
remaining side, we use the Pythagorean Theorem. \begin{align*}
x^2 + y^2 & = r^2\ \ \ \ \ \ \ \ \ \ \ \ \ \ \ \ [\ \text{Pythagorean Theorem}\ ]\\
7^2 + y^2 & = 25^2\\
y & = \pm 24\\
y & = 24\ \ \ \ \ \ \ \ \ \ \ \ \ \ \ \ [\ \text{Because}\ y > 0\ ]
\end{align*}\\
Now, we use the Double-Angle Formula for Sine. \begin{align*}
\sin \bigg(2 \cos^{-1} \dfrac{7}{25}\bigg) & = \sin 2 \theta\ \ \ \ \ \ \ \ \ \ \ \
\ \ \ \ \ \ \ \ \ \ \ \ [\ \text{Since}\ \cos^{-1} \frac{7}{25} = \theta\ ]\\
& = 2 \sin \theta \cos \theta \ \ \ \ \ \ \ \ \ \ \ \ \ \ \ \ \ [\ \text{Double-
Angle Formula}\ ]\\
& = 2\bigg(\dfrac{24}{25}\bigg)\bigg(\dfrac{7}
{25}\bigg) \ \ \ \ \ \ \ \ \ \ \ \ \ \ [\ \text{From the triangle}\ ]\\
& = \dfrac{336}{625}
\end{align*}

$$\cos \bigg(2\tan^{-1} \frac{12}{5}\bigg)$$\\


Let $\theta = \tan^{-1} \frac{12}{5}$, then $\tan \theta = \frac{12}{5}$. We first
sketch the angle $\theta$ in standard position with terminal side in Quadrant I as
shown in the below figure. Since $\tan \theta = \frac{y}{x} = \frac{12}{5}$, we can
label a side and the hypotenuse of the triangle in the below figure. To find the
remaining side, we use the Pythagorean Theorem. \begin{align*}
x^2 + y^2 & = r^2\ \ \ \ \ \ \ \ \ \ \ \ \ \ \ \ [\ \text{Pythagorean Theorem}\ ]\\
5^2 + 12^2 & = r^2\\
r & = \pm 13\\
r & = 13\ \ \ \ \ \ \ \ \ \ \ \ \ \ \ \ [\ \text{Because}\ r > 0\ ]
\end{align*}\\
Now, we use the Double-Angle Formula for Cosine. \begin{align*}
\cos \bigg(2 \tan^{-1} \dfrac{12}{5}\bigg) & = \cos 2 \theta\ \ \ \ \ \ \ \ \ \ \ \
\ \ \ \ \ \ \ \ \ \ \ \ \bigg[\ \text{Since}\ \tan^{-1} \frac{12}{5} = \theta\
\bigg]\\
& = 2 \cos^2 \theta - 1 \ \ \ \ \ \ \ \ \ \ \ \ \ \ \ \ \ [\ \text{Double-Angle
Formula}\ ]\\
& = 2\bigg(\dfrac{5}{13}\bigg)^2 - 1 \ \ \ \ \ \ \ \ \ \ \ \ \ \ \ \ [\ \text{From
the triangle}\ ]\\
& = -\dfrac{119}{169}
\end{align*}

$$\sec \bigg(2\sin^{-1} \frac{1}{4}\bigg)$$\\


Let $\theta = \sin^{-1} \frac{1}{4}$, then $\sin \theta = \frac{1}{4}$. We first
sketch the angle $\theta$ in standard position with terminal side in Quadrant I as
shown in the below figure. Since $\sin \theta = \frac{y}{r} = \frac{1}{4}$, we can
label a side and the hypotenuse of the triangle in the below figure. To find the
remaining side, we use the Pythagorean Theorem. \begin{align*}
x^2 + y^2 & = r^2\ \ \ \ \ \ \ \ \ \ \ \ \ \ \ \ \ [\ \text{Pythagorean
Theorem}\ ]\\
1^2 + y^2 & = 4^2\\
y & = \pm \sqrt{15}\\
y & = \sqrt{15}\ \ \ \ \ \ \ \ \ \ \ \ \ \ \ \ [\ \text{Because}\ y > 0\ ]
\end{align*}\\
Now, we know that $\sec 2 \theta = \dfrac{1}{\cos 2\theta}$, and we need to find
$\sec 2 \theta$.\\
\\
\\
Now, we use the Double-Angle Formula for Cosine. \begin{align*}
\sec \bigg(2 \sin^{-1} \dfrac{1}{4}\bigg) & = \sec 2 \theta \ \ \ \ \ \ \ \ \ \ \ \
\ \ \ \ \ \ \ \ \ \ \ \ \bigg[\ \text{Since}\ \sin^{-1} \frac{1}{4} = \theta\
\bigg]\\
& = \dfrac{1}{cos 2 \theta}\\
& = \dfrac{1}{2 \cos^2 \theta - 1} \ \ \ \ \ \ \ \ \ \ \ \ \ \ \ \ \ [\
\text{Double-Angle Formula}\ ]\\
& = \dfrac{1}{2\bigg(\dfrac{\sqrt{15}}{4}\bigg)^2 - 1} \ \ \ \ \ \ \ \ \ \ \ \ \ [\
\text{From the triangle}\ ]\\
& = \dfrac{1}{\dfrac{15}{8} - 1}\\
& = \dfrac{8}{7}
\end{align*}

$$\tan \bigg(\dfrac{1}{2}\cos^{-1} \frac{2}{3}\bigg)$$\\


Let $\theta = \cos^{-1} \frac{2}{3}$, then $\cos \theta = \frac{2}{3}$. We first
sketch the angle $\theta$ in standard position with terminal side in Quadrant I as
shown in the below figure. Since $\cos \theta = \frac{x}{r} = \frac{2}{3}$, we can
label a side and the hypotenuse of the triangle in the below figure. To find the
remaining side, we use the Pythagorean Theorem. \begin{align*}
x^2 + y^2 & = r^2\ \ \ \ \ \ \ \ \ \ \ \ \ \ \ \ [\ \text{Pythagorean Theorem}\ ]\\
2^2 + y^2 & = 3^2\\
y & = \pm \sqrt{5}\\
y & = \sqrt{5}\ \ \ \ \ \ \ \ \ \ \ \ \ \ \ \ [\ \text{Because}\ y > 0\ ]
\end{align*}\\
\\
Now, we use the Half-Angle Formula for Tangent. \begin{align*}
\tan \bigg(\dfrac{1}{2}\cos^{-1} \frac{2}{3}\bigg) & = \tan \dfrac{\theta}
{2}\ \ \ \ \ \ \ \ \ \ \ \ \ \ \ \ \ \ \ \ \ \ \bigg[\ \text{Since}\ \cos^{-1}
\frac{2}{3} = \theta\ \bigg]\\
& = \dfrac{1 - \cos \theta}{\sin \theta} \ \ \ \ \ \ \ \ \ \ \ \ \ \ \ \ \ \ [\
\text{Half-Angle Formula}\ ]\\
& = \dfrac{1-\frac{2}{3}}{\frac{\sqrt{5}}
{3}} \ \ \ \ \ \ \ \ \ \ \ \ \ \ \ \ \ \ \ \ \ \ [\ \text{From the triangle}\ ]\\
& = \dfrac{1}{\sqrt{5}}
\end{align*}

$$\cos 2 \theta;\ \ \ \sin \theta = -\dfrac{3}{5},\ \ \ \theta\ \text{in Quadrant


III}$$ \begin{align*}
\cos 2 \theta & = 1 - 2\sin^2 \theta\ \ \ \ \ \ \ \ \ \ \ \ \ \ \ [\ \text{Double-
Angle Formula}\ ]\\
& = 1 - 2\bigg(-\dfrac{3}{5}\bigg)^2\\
& = 1 - \dfrac{18}{25}\\
& = \dfrac{7}{25}
\end{align*}

$$\sin \dfrac{\theta}{2};\ \ \ \tan \theta = -\dfrac{5}{12},\ \ \ \theta\ \text{in


Quadrant IV}$$\\
We first sketch the angle $\theta$ in standard position with terminal side in
Quadrant IV as shown in the below figure. Since $\tan \theta = \frac{y}{x} =
-\frac{5}{12}$, we can label a side and the hypotenuse of the triangle in the below
figure. To find the remaining side, we use the Pythagorean Theorem. \begin{align*}
x^2 + y^2 & = r^2\ \ \ \ \ \ \ \ \ \ \ \ \ \ \ [\ \text{Pythagorean Theorem}\ ]\\
(12)^2 + (-5)^2 & = r^2\\
r & = \pm 13\\
r & = 13\ \ \ \ \ \ \ \ \ \ \ \ \ \ \ \ [\ \text{Because}\ r > 0\ ]
\end{align*}\\
Since $\theta$ lies in Quadrant IV, i.e. $270^o < \theta < 360^o$, we have $$135^o
< \dfrac{\theta}{2} < 180^o,\ \text{i.e.}\ \dfrac{\theta}{2}\ \text{lies in
Quadrant II}$$
Then $\sin \dfrac{\theta}{2} > 0$ \begin{align*}
\sin \dfrac{\theta}{2} & = \sqrt{\dfrac{1 - \cos \theta}
{2}}\ \ \ \ \ \ \ \ \ \ \ \ \ \ \ [\ \text{Half-Angle Formula}\ ]\\
& = \sqrt{\dfrac{1 - \frac{12}{13}}{2}}\ \ \ \ \ \ \ \ \ \ \ \ \ \ \ \ \ \ [\
\text{From the triangle}\ ]\\
& = \dfrac{1}{\sqrt{26}}
\end{align*}\\

$$\sin 2 \theta;\ \ \ \sin \theta = \dfrac{1}{7},\ \ \ \theta\ \text{in Quadrant


II}$$\\ Since $\cos \theta$ is negative in Quadrant II, we have $$\cos \theta =
-\sqrt{1 - \sin^2 \theta} = -\sqrt{1 - \dfrac{1}{49}} = -\dfrac{4\sqrt{3}}{7}$$\\
\begin{align*}
\sin 2 \theta & = 2\sin \theta \cos \theta\ \ \ \ \ \ \ \ \ \ \ \ \ \ \ [\
\text{Double-Angle Formula}\ ]\\
& = 2 \bigg(\dfrac{1}{7}\bigg)\bigg(-\dfrac{4\sqrt{3}}{7}\bigg)\\
& = -\dfrac{8\sqrt{3}}{49}
\end{align*}

$$\tan 2\theta;\ \ \ \cos \theta = \dfrac{3}{5},\ \ \ \theta\ \text{in Quadrant I}


$$\\
We first sketch the angle $\theta$ in standard position with terminal side in
Quadrant I as shown in the below figure. Since $\cos \theta = \frac{x}{r} =
\frac{3}{5}$, we can label a side and the hypotenuse of the triangle in the below
figure. To find the remaining side, we use the Pythagorean Theorem. \begin{align*}
x^2 + y^2 & = r^2\ \ \ \ \ \ \ \ \ \ \ \ \ \ \ \ [\ \text{Pythagorean Theorem}\ ]\\
3^2 + y^2 & = 5^2\\
y & = \pm \sqrt{16}\\
y & = 4\ \ \ \ \ \ \ \ \ \ \ \ \ \ \ \ \ [\ \text{Because}\ y > 0\ ]
\end{align*}\\
\begin{align*}
\tan 2 \theta & = \dfrac{2 \tan \theta}{1 - \tan^2
\theta} \ \ \ \ \ \ \ \ \ \ \ \ \ \ \ \ \ [\ \text{Double-Angle Formula}\ ]\\
& = \dfrac{2 \bigg(\dfrac{4}{3}\bigg)}{1 - \bigg(\dfrac{4}
{3}\bigg)^2} \ \ \ \ \ \ \ \ \ \ \ \ \ \ \ \ [\ \text{From the triangle}\ ]\\
& = \dfrac{\dfrac{8}{3}}{-\dfrac{7}{9}} = -\dfrac{24}{7}
\end{align*}

$$\sin 2x \cos 3x$$\\


The first Product-To-Sum Formula gives $$\sin u \cos v = \dfrac{1}{2}[\sin (u + v)
+ \sin (u - v)]$$
Let $u = 2x$ and $v = 3x$, then we get \begin{align*}
\sin 2x \cos 3x & = \dfrac{1}{2}[\sin (2x + 3x) + \sin (2x - 3x)]\\
& = \dfrac{1}{2}[\sin 5x + \sin (-x)]\\
& = \dfrac{1}{2}[\sin 5x - \sin x]\ \ \ \ \ \ \ \ \ \ \ \ \ \ \ \ \ \ \ \ \ \ [\
\text{Even-Odd Identities}\ ]
\end{align*}
$$\sin x \sin 5x$$\\
The fourth Product-To-Sum Formula gives $$\sin u \sin v = \dfrac{1}{2}[\cos (u - v)
- \cos (u + v)]$$
Let $u = x$ and $v = 5x$, then we get \begin{align*}
\sin x \sin 5x & = \dfrac{1}{2}[\cos (x - 5x) - \cos (x + 5x)]\\
& = \dfrac{1}{2}[\cos (-4x) - \cos 6x]\\
& = \dfrac{1}{2}[\cos 4x - \cos 6x]\ \ \ \ \ \ \ \ \ \ \ \ \ \ \ \ \ \ \ \ \ \
[\ \text{Even-Odd Identities}\ ]
\end{align*}

$$\cos x \sin 4x$$\\


The second Product-To-Sum Formula gives $$\cos u \sin v = \dfrac{1}{2}[\sin (u + v)
- \sin (u - v)]$$
Let $u = x$ and $v = 4x$, then we get \begin{align*}
\cos x \sin 4x & = \dfrac{1}{2}[\sin (x + 4x) - \sin (x - 4x)]\\
& = \dfrac{1}{2}[\sin 5x - \sin (-3x)]\\
& = \dfrac{1}{2}[\sin 5x + \sin 3x]\ \ \ \ \ \ \ \ \ \ \ \ \ \ \ \ \ \ \ \ \ \
[\ \text{Even-Odd Identities}\ ]
\end{align*}

$$\cos 5x \cos 3x$$\\


The third Product-To-Sum Formula gives $$\cos u \cos v = \dfrac{1}{2}[\cos (u + v)
+ \cos (u - v)]$$
Let $u = 5x$ and $v = 3x$, then we get \begin{align*}
\cos 5x \cos 3x & = \dfrac{1}{2}[\cos (5x + 3x) + \cos (5x - 3x)]\\
& = \dfrac{1}{2}[\cos 8x + \cos 2x]\\
\end{align*}

$$3\cos 4x \cos 7x$$\\


The third Product-To-Sum Formula gives $$\cos u \cos v = \dfrac{1}{2}[\cos (u + v)
+ \cos (u - v)]$$
Let $u = 4x$ and $v = 7x$, then we get \begin{align*}
\cos 4x \cos 7x & = \dfrac{1}{2}[\cos (4x + 7x) + \cos (4x - 7x)]\\
& = \dfrac{1}{2}[\cos 11x + \cos (-3x)]\\
& = \dfrac{1}{2}[\cos 11x + \cos 3x]\ \ \ \ \ \ \ \ \ \ \ \ \ \ \ \ \ [\
\text{Even-Odd Identities}\ ]\\
\text{Thus}\ 3\cos 4x \cos 7x & = \dfrac{3}{2}[\cos 11x + \cos 3x]
\end{align*}

$$11\sin \dfrac{x}{2} \cos \dfrac{x}{4}$$\\


The first Product-To-Sum Formula gives $$\sin u \cos v = \dfrac{1}{2}[\sin (u + v)
+ \sin (u - v)]$$
Let $u = \dfrac{x}{2}$ and $v = \dfrac{x}{4}$, then we get \begin{align*}
\sin \dfrac{x}{2} \cos \dfrac{x}{4} & = \dfrac{1}{2}\bigg[\sin \bigg(\dfrac{x}{2} +
\dfrac{x}{4}\bigg) + \sin \bigg(\dfrac{x}{2} - \dfrac{x}{4}\bigg)\bigg]\\
& = \dfrac{1}{2}\bigg(\sin \dfrac{3x}{4} + \sin \dfrac{x}{4}\bigg)\\
\text{Thus}\ 11 \sin \dfrac{x}{2} \cos \dfrac{x}{4} & = \dfrac{11}{2}\bigg(\sin
\dfrac{3x}{4} + \sin \dfrac{x}{4}\bigg)
\end{align*}

$$\sin 5x + \sin 3x$$\\


The first Sum-To-Product Formula gives $$\sin u + \sin v = 2\sin \bigg(\dfrac{u +
v}{2}\bigg) \cos \bigg(\dfrac{u - v}{2}\bigg)$$
Let $u = 5x$ and $v = 3x$, then we get \begin{align*}
\sin 5x + \sin 3x & = 2\sin \bigg(\dfrac{5x + 3x}{2}\bigg) \cos \bigg(\dfrac{5x -
3x}{2}\bigg)\\
& = 2\sin \bigg(\dfrac{8x}{2}\bigg) \cos \bigg(\dfrac{2x}{2}\bigg)\\
& = 2\sin 4x \cos x\\
\end{align*}

$$\sin x - \sin 4x$$\\


The second Sum-To-Product Formula gives $$\sin u - \sin v = 2\cos \bigg(\dfrac{u +
v}{2}\bigg) \sin \bigg(\dfrac{u - v}{2}\bigg)$$
Let $u = x$ and $v = 4x$, then we get \begin{align*}
\sin x - \sin 4x & = 2\cos \bigg(\dfrac{x + 4x}{2}\bigg) \sin \bigg(\dfrac{x - 4x}
{2}\bigg)\\
& = 2\cos \bigg(\dfrac{5x}{2}\bigg) \sin \bigg(\dfrac{-3x}{2}\bigg)\\
& = -2\cos \bigg(\dfrac{5x}{2}\bigg) \sin \bigg(\dfrac{3x}
{2}\bigg)\ \ \ \ \ \ \ \ \ \ \ \ \ [\ \text{Even-Odd Identities}\ ]
\end{align*}

$$\cos 4x - \cos 6x$$\\


The fourth Sum-To-Product Formula gives $$\cos u - \cos v = -2\sin \bigg(\dfrac{u +
v}{2}\bigg) \sin \bigg(\dfrac{u - v}{2}\bigg)$$
Let $u = 4x$ and $v = 6x$, then we get \begin{align*}
\cos 4x - \cos 6x & = -2\sin \bigg(\dfrac{4x + 6x}{2}\bigg) \sin \bigg(\dfrac{4x -
6x}{2}\bigg)\\
& = -2\sin \bigg(\dfrac{10x}{2}\bigg) \sin \bigg(\dfrac{-2x}{2}\bigg)\\
& = -2 \sin 5x \sin (-x)\\
& = \ \ 2 \sin 5x \sin x\ \ \ \ \ \ \ \ \ \ \ \ \ \ \ \ \ \ \ [\ \text{Even-Odd
Identities}\ ]
\end{align*}

$$\cos 9x + \cos 2x$$\\


The third Sum-To-Product Formula gives $$\cos u + \cos v = 2\cos \bigg(\dfrac{u +
v}{2}\bigg) \cos \bigg(\dfrac{u - v}{2}\bigg)$$
Let $u = 9x$ and $v = 2x$, then we get \begin{align*}
\cos 9x + \cos 2x & = 2\cos \bigg(\dfrac{9x + 2x}{2}\bigg) \cos \bigg(\dfrac{9x -
2x}{2}\bigg)\\
& = 2\cos \bigg(\dfrac{11x}{2}\bigg) \cos \bigg(\dfrac{7x}{2}\bigg)\\
\end{align*}

$$\sin 2x - \sin 7x$$\\


The second Sum-To-Product Formula gives $$\sin u - \sin v = 2\cos \bigg(\dfrac{u +
v}{2}\bigg) \sin \bigg(\dfrac{u - v}{2}\bigg)$$
Let $u = 2x$ and $v = 7x$, then we get \begin{align*}
\sin 2x - \sin 7x & = 2\cos \bigg(\dfrac{2x + 7x}{2}\bigg) \sin \bigg(\dfrac{2x -
7x}{2}\bigg)\\
& = 2\cos \bigg(\dfrac{9x}{2}\bigg) \sin \bigg(\dfrac{-5x}{2}\bigg)\\
& = -2\cos \bigg(\dfrac{9x}{2}\bigg) \sin \bigg(\dfrac{5x}
{2}\bigg)\ \ \ \ \ \ \ \ \ \ \ \ \ [\ \text{Even-Odd Identities}\ ]
\end{align*}

$$\sin 3x + \sin 4x$$\\


The first Sum-To-Product Formula gives $$\sin u + \sin v = 2\sin \bigg(\dfrac{u +
v}{2}\bigg) \cos \bigg(\dfrac{u - v}{2}\bigg)$$
Let $u = 3x$ and $v = 4x$, then we get \begin{align*}
\sin 3x + \sin 4x & = 2\sin \bigg(\dfrac{3x + 4x}{2}\bigg) \cos \bigg(\dfrac{3x -
4x}{2}\bigg)\\
& = 2\sin \bigg(\dfrac{7x}{2}\bigg) \cos \bigg(\dfrac{-x}{2}\bigg)\\
& = 2\sin \bigg(\dfrac{7x}{2}\bigg) \cos \bigg(\dfrac{x}
{2}\bigg)\ \ \ \ \ \ \ \ \ \ \ [\ \text{Even-Odd Identities}\ ]
\end{align*}

$$2\sin 52.5^o \sin 97.5^o$$\\


The fourth Product-To-Sum Formula gives $$\sin u \sin v = \dfrac{1}{2}[\cos (u - v)
- \cos (u + v)]$$
Let $u = 52.5^o$ and $v = 97.5^o$, then we get \begin{align*}
\sin 52.5^o \sin 97.5^o & = \dfrac{1}{2}[\cos (52.5^o - 97.5^o) - \cos (52.5^o +
97.5^o)]\\
& = \dfrac{1}{2}[\cos (-45^o) - \cos 150^o]\\
& = \dfrac{1}{2}(\cos 45^o - \cos (90 + 60)^o)\ \ \ \ \ \ \ \ \ \ [\ \text{Even-Odd
Identities}\ ]\\
& = \dfrac{1}{2}(\cos 45^o + \sin 60^o)\ \ \ \ \ \ \ \ \ \ \ \ \ \ \ \ \ \ \ [\
\text{Addition Formula for Cosine}\ ]\\
& = \dfrac{1}{2}\bigg(\dfrac{\sqrt{2}}{2} + \dfrac{\sqrt{3}}{2}\bigg)\\
& = \dfrac{1}{2}\bigg(\dfrac{\sqrt{2} + \sqrt{3}}{2}\bigg)\\
\text{Thus}\ 2\sin 52.5^o \sin 97.5^o & = 2\bigg[\dfrac{1}{2}\bigg(\dfrac{\sqrt{2}
+ \sqrt{3}}{2}\bigg)\bigg]\\
& = \dfrac{\sqrt{2} + \sqrt{3}}{2}
\end{align*}

$$\cos 37.5^o \sin 7.5^o$$\\


The second Product-To-Sum Formula gives $$\cos u \sin v = \dfrac{1}{2}[\sin (u + v)
- \sin (u - v)]$$
Let $u = 37.5^o$ and $v = 7.5^o$, then we get \begin{align*}
\cos 37.5^o \sin 7.5^o & = \dfrac{1}{2}[\sin (37.5^o + 7.5^o) - \sin (37.5^o -
7.5^o)]\\
& = \dfrac{1}{2}(\sin 45^o - \sin 30^o)\\
& = \dfrac{1}{2}\bigg(\dfrac{\sqrt{2}}{2} - \dfrac{1}{2}\bigg)\\
& = \dfrac{1}{2}\bigg(\dfrac{\sqrt{2} -1}{2}\bigg)\\
& = \dfrac{\sqrt{2} -1}{4}\\
\end{align*}

$$\sin 75^o + \sin 15^o$$\\


The first Sum-To-Product Formula gives $$\sin u + \sin v = 2\sin \bigg(\dfrac{u +
v}{2}\bigg) \cos \bigg(\dfrac{u - v}{2}\bigg)$$
Let $u = 75^o$ and $v = 15^o$, then we get \begin{align*}
\sin 75^o + \sin 15^o & = 2\sin \bigg(\dfrac{75^o + 15^o}{2}\bigg) \cos
\bigg(\dfrac{75^o - 15^o}{2}\bigg)\\
& = 2\sin \bigg(\dfrac{90^o}{2}\bigg) \cos \bigg(\dfrac{60^o}{2}\bigg)\\
& = 2\sin 45^o \cos 30^o\\
& = 2 \bigg(\dfrac{\sqrt{2}}{2}\bigg)\bigg(\dfrac{\sqrt{3}}{2}\bigg)\\
& = \dfrac{\sqrt{6}}{2}
\end{align*}

$$\cos 255^o - \cos 195^o$$\\


The fourth Sum-To-Product Formula gives $$\cos u - \cos v = -2\sin \bigg(\dfrac{u +
v}{2}\bigg) \sin \bigg(\dfrac{u - v}{2}\bigg)$$
Let $u = 255^o$ and $v = 195^o$, then we get \begin{align*}
\cos 255^o - \cos 195^o & = -2\sin \bigg(\dfrac{255^o + 195^o}{2}\bigg) \sin
\bigg(\dfrac{255^o - 195^o}{2}\bigg)\\
& = -2\sin \bigg(\dfrac{450^o}{2}\bigg) \sin \bigg(\dfrac{60^o}{2}\bigg)\\
& = -2 \sin 225^o \sin 30^o\\
& = -2 \sin (180^o + 45^o) \sin 30^o\\
& = 2 \sin 45^o \sin 30^o\ \ \ \ \ \ \ \ \ \ \ \ \ \ [\ \text{Addition Formula for
Sine}\ ]\\
& = 2\bigg(\dfrac{\sqrt{2}}{2}\bigg)\bigg(\dfrac{1}{2}\bigg)\\
& = \dfrac{\sqrt{2}}{2}
\end{align*}

$$\cos \dfrac{\pi}{12} + \cos \dfrac{5\pi}{12}$$\\


The third Sum-To-Product Formula gives $$\cos u + \cos v = 2\cos \bigg(\dfrac{u +
v}{2}\bigg) \cos \bigg(\dfrac{u - v}{2}\bigg)$$
Let $u = \dfrac{\pi}{12}$ and $v = \dfrac{5\pi}{12}$, then we get \begin{align*}
\cos \dfrac{\pi}{12} + \cos \dfrac{5\pi}{12} & = 2\cos \Bigg(\dfrac{\frac{\pi}{12}
+ \frac{5\pi}{12}}{2}\Bigg) \cos \Bigg(\dfrac{\frac{\pi}{12} - \frac{5\pi}{12}}
{2}\Bigg)\\
& = 2\cos \bigg(\dfrac{\pi}{4}\bigg) \cos \bigg(-\dfrac{\pi}{6}\bigg)\\
& = 2\cos \dfrac{\pi}{4} \cos \dfrac{\pi}{6}\ \ \ \ \ \ \ \ \ \ \ \ \ \ \ \ [\
\text{Even-Odd Identities}\ ]\\
& = 2 \bigg(\dfrac{\sqrt{2}}{2}\bigg)\bigg(\dfrac{\sqrt{3}}{2}\bigg)\\
& = \dfrac{\sqrt{6}}{2}
\end{align*}

$$\sin 8x = 2 \sin 4x \cos 4x$$\\


We can simplify LHS as \begin{align*}
\text{LHS} & = \sin 8x\\
& = \sin 2(4x)\\
& = 2 \sin 4x \cos 4x\ \ \ \ \ \ \ \ \ \ \ \ [\ \text{Double-Angle Formula}\ ]\\
& = \text{RHS}
\end{align*}

$$(\sin x + \cos x)^2 = 1 + \sin 2x$$\\


We can simplify LHS as \begin{align*}
\text{LHS} & = (\sin x + \cos x)^2\\
& = \sin^2 x + \cos^2 x + 2 \sin x \cos x\\
& = 1 + 2 \sin x \cos x\ \ \ \ \ \ \ \ \ \ \ \ [\ \text{Pythagorean identity}\ ]\\
& = 1 + \sin 2x\ \ \ \ \ \ \ \ \ \ \ \ \ \ \ \ \ \ \ [\ \text{Double-Angle
Formula}\ ]\\
& = \text{RHS}
\end{align*}

$$\cos^4 x - \sin^4 x = \cos 2x$$\\


We can simplify LHS as \begin{align*}
\text{LHS} & = \cos^4 x - \sin^4 x\\
& = (\cos^2 x)^2 - (\sin^2 x)^2\\
& = (\cos^2 x + \sin^2 x)(\cos^2 x - \sin^2 x)\ \ \ \ \ \ \ \ \ \ [\ \text{Using}\
a^2 - b^2 = (a+b)(a-b)\ ]\\
& = 1(\cos^2 x - \sin^2 x)\ \ \ \ \ \ \ \ \ \ \ \ \ \ \ \ \ \ \ \ \ \ \ \ \ \ \ \ \
[\ \text{Pythagorean identity}\ ]\\
& = \cos
2x\ \ \ \ \ \ \ \ \ \ \ \ \ \ \ \ \ \ \ \ \ \ \ \ \ \ \ \ \ \ \ \ \ \ \ \ \ \ \ \ \
\ \ [\ \text{Double-Angle Formula}\ ]\\
& = \text{RHS}
\end{align*}

$$ \dfrac{2 \tan x}{1 + \tan^2 x} = \sin 2x$$\\


We can simplify LHS as \begin{align*}
\text{LHS} & = \dfrac{2 \tan x}{1 + \tan^2 x}\\
& = \dfrac{2 \tan x}{\sec^2 x}\ \ \ \ \ \ \ \ \ \ \ \ [\ \text{Pythagorean
Identity}\ ]\\
& = \dfrac{2 \dfrac{\sin x}{\cos x}}{\dfrac{1}{\cos^2 x}}\ \ \ \ \ \ \ \ \ \ \ \ [\
\text{Reciprocal Identity}\ ]\\
& = 2 \sin x \cos x\\
& = \sin 2x \ \ \ \ \ \ \ \ \ \ \ \ \ [\ \text{Double-Angle Formula}\ ]\\
& = \text{RHS}
\end{align*}

$$\dfrac{1 - \cos 2x}{\sin 2x} = \tan x$$\\


We can simplify LHS as \begin{align*}
\text{LHS} & = \dfrac{1 - \cos 2x}{\sin 2x}\\
& = \dfrac{1 - (1 - 2\sin^2 x)}{2 \sin x \cos x}\ \ \ \ \ \ \ \ \ \ \ \ [\
\text{Double-Angle Formula}\ ]\\
& = \dfrac{1 - 1 + 2\sin^2 x}{2 \sin x \cos x}\\
& = \dfrac{2\sin^2 x}{2 \sin x \cos x}\\
& = \tan x\ \ \ \ \ \ \ \ \ \ \ \ \ \ \ \ \ \ \ \ \ \ \ \ \ \ \ \ [\
\text{Reciprocal Identities}\ ]\\
& = \text{RHS}
\end{align*}

$$\tan \bigg(\dfrac{x}{2}\bigg) + \cos x \tan \bigg(\dfrac{x}{2}\bigg) = \sin x$$\\


We can simplify LHS as \begin{align*}
\text{LHS} & = \tan \bigg(\dfrac{x}{2}\bigg) + \cos x \tan \bigg(\dfrac{x}
{2}\bigg)\\
& = \tan \bigg(\dfrac{x}{2}\bigg)(1 + \cos x )\\
& = \bigg(\dfrac{\sin x}{1 + \cos x}\bigg)(1 + \cos x )\ \ \ \ \ \ \ \ \ \ \ [\
\text{Half-Angle Formula}\ ]\\
& = \sin x\\
& = \text{RHS}
\end{align*}

$$\tan \bigg(\dfrac{x}{2}\bigg) + \csc x = \dfrac{2 - \cos x}{\sin x}$$\\


We can simplify LHS as \begin{align*}
\text{LHS} & = \tan \bigg(\dfrac{x}{2}\bigg) + \csc x\\
& = \dfrac{1 - \cos x}{\sin x} + \csc x\ \ \ \ \ \ \ \ \ \ \ [\ \text{Half-Angle
Formula}\ ]\\
& = \dfrac{1 - \cos x}{\sin x} + \dfrac{1}{\sin x}\ \ \ \ \ \ \ \ \ \ \ [\
\text{Reciprocal Identity}\ ]\\
& = \dfrac{1 - \cos x + 1}{\sin x}\\
& = \dfrac{2 - \cos x}{\sin x}\\
& = \text{RHS}
\end{align*}

$$\dfrac{\sin 4x}{\sin x} = 4 \cos x \cos 2x$$\\


We can simplify LHS as \begin{align*}
\text{LHS} & = \dfrac{\sin 4x}{\sin x}\\
& = \dfrac{\sin 2(2x)}{\sin x}\\
& = \dfrac{2 \sin 2x \cos 2x}{\sin x}\ \ \ \ \ \ \ \ \ \ \ \ \ \ \ \ \ \ \ \ [\
\text{Double-Angle Formula}\ ]\\
& = \dfrac{2 (2 \sin x \cos x) \cos 2x}{\sin x}\ \ \ \ \ \ \ \ \ \ \ [\
\text{Double-Angle Formula}\ ]\\
& = 4 \cos x \cos 2x\\
& = \text{RHS}
\end{align*}

$$\dfrac{2(\tan x - \cot x)}{\tan^2 x - \cot^2 x} = \sin 2x$$\\


We can simplify LHS as \begin{align*}
\text{LHS} & = \dfrac{2(\tan x - \cot x)}{\tan^2 x - \cot^2 x}\\
& = \dfrac{2(\tan x - \cot x)}{(\tan x + \cot x)(\tan x - \cot x)}\ \ \ \ \ \ \ \ \
\ [\ \text{Using}\ a^2 - b^2 = (a+b)(a-b)\ ]\\
& = \dfrac{2}{\tan x + \cot x}\\
& = \dfrac{2}{\tan x + \dfrac{1}{\tan
x}}\ \ \ \ \ \ \ \ \ \ \ \ \ \ \ \ \ \ \ \ \ \ \ \ \ \ \ \ \ \ [\ \text{Reciprocal
Identities}\ ]\\
& = \dfrac{2\tan x}{1 + \tan^2 x}\\
& = \dfrac{2\tan x}{\sec^2
x}\ \ \ \ \ \ \ \ \ \ \ \ \ \ \ \ \ \ \ \ \ \ \ \ \ \ \ \ \ \ \ \ \ \ \ \ \ \ \
[\ \text{Pythagorean Identity}\ ]\\
& = \dfrac{\dfrac{2\sin x}{\cos x}}{\dfrac{1}{\cos^2 x}}\ \ \ \ \ \ \ \ \ \ \ \ \ \
\ \ \ \ \ \ \ \ \ \ \ \ \ \ \ \ \ \ \ \ \ \ \ \ \ [\ \text{Reciprocal
Identity}\ ]\\
& = 2\sin x \cos x\\
& = \sin
2x\ \ \ \ \ \ \ \ \ \ \ \ \ \ \ \ \ \ \ \ \ \ \ \ \ \ \ \ \ \ \ \ \ \ \ \ \ \ \ \ \
[\ \text{Double-Angle Formula}\ ]\\
& = \text{RHS}
\end{align*}
$$\tan x = \dfrac{\sin 2x}{1 + \cos 2x}$$\\
We can simplify RHS as \begin{align*}
\text{RHS} & = \dfrac{\sin 2x}{1 + \cos 2x}\\
& = \dfrac{2 \sin x \cos x}{1 + (2 \cos^2 x - 1)}\ \ \ \ \ \ \ \ \ \ \ \ \ \ [\
\text{Double-Angle Formula}\ ]\\
& = \dfrac{2 \sin x \cos x}{2 \cos^2 x}\\
& = \dfrac{\sin x}{\cos x}\\
& = \tan x\ \ \ \ \ \ \ \ \ \ \ \ \ \ \ \ \ \ \ \ \ \ \ \ \ \ \ \ \ \ [\
\text{Pythagorean Identity}\ ]\\
& = \text{LHS}
\end{align*}

$$\cot 2x = \dfrac{1 - \tan^2 x}{2 \tan x}$$\\


We can simplify LHS as \begin{align*}
\text{LHS} & = \cot 2x\\
& = \dfrac{1}{\tan 2x}\\
& = \dfrac{1}{\dfrac{2\tan x}{1 - \tan^2 x}} \ \ \ \ \ \ \ \ \ \ \ \ \ \ \ [\
\text{Double-Angle Formula}\ ]\\
& = \dfrac{1 - \tan^2 x}{2\tan x}\\
& = \text{RHS}
\end{align*}

$$4(\sin^6 x + \cos^6 x) = 4 - 3 \sin^2 2x$$\\


We can simplify LHS as \begin{align*}
\text{LHS} & = 4(\sin^6 x + \cos^6 x)\\
& = 4(\sin^6 x + (\cos^2 x)^3)\\
& = 4(\sin^6 x + (1 - \sin^2 x)^3)\ \ \ \ \ \ \ \ \ \ \ \ \ \ \ \ \ \ \ \ \ \ \ \ \
\ \ \ [\ \text{Pythagorean Identity}\ ]\\
& = 4(\sin^6 x + 1 - \sin^6 x - 3\sin^2 x + 3\sin^4 x)\ \ \ \ \ \ [\
\text{Simplify}\ ]\\
& = 4(1 - 3\sin^2 x + 3\sin^4 x)\\
& = 4(1 - 3\sin^2 x(1 - \sin^2 x))\ \ \ \ \ \ \ \ \ \ \ \ \ \ \ \ \ \ \ \ \ \ \ \
\ \ [\ \text{Taking common factor}\ ]\\
& = 4(1 - 3\sin^2 x \cos^2
x)\ \ \ \ \ \ \ \ \ \ \ \ \ \ \ \ \ \ \ \ \ \ \ \ \ \ \ \ \ \ \ \ \ [\
\text{Pythagorean Identity}\ ]\\
& = 4 - 12\sin^2 x \cos^2 x\\
& = 4 - 3(4\sin^2 x \cos^2 x)\\
& = 4 - 3(2\sin x \cos x)^2\\
& = 4 - 3(\sin 2x)^2\ \ \ \ \ \ \ \ \ \ \ \ \ \ \ \ \ \ \ \ \ \ \ \ \ \ \ \ \ \ \ \
\ \ \ \ \ \ \ \ \ \ [\ \text{Double-Angle Formula}\ ]\\
& = 4 - 3\sin^2 2x\\
& = \text{RHS}
\end{align*}

$$$\dfrac{\sin 3x + \cos 3x}{\cos x - \sin x} = 1 + 4 \sin x \cos x$$\\


First using, Addition Formula for Sine and Cosine, we find \begin{align*}
\sin 3x & = \sin (2x + x)\\
& = \sin 2x \cos x + \cos 2x \sin x\\
\\
\cos 3x & = \cos (2x + x)\\
& = \cos 2x \cos x - \sin 2x \sin x
\end{align*}
Now, we can simplify LHS as \begin{align*}
\text{LHS} & = \dfrac{\sin 3x + \cos 3x}{\cos x - \sin x}\\
& = \dfrac{\sin 2x \cos x + \cos 2x \sin x + \cos 2x \cos x - \sin 2x \sin x}{\cos
x - \sin x}\\
& = \dfrac{\sin 2x(\cos x - \sin x) + \cos 2x ( \sin x + \cos x)}{\cos x - \sin
x}\\
& = \dfrac{\sin 2x(\cos x - \sin x)}{\cos x - \sin x} + \dfrac{\cos 2x ( \sin x
+ \cos x)}{\cos x - \sin x}\\
& = \sin 2x + \dfrac{(\cos^2 x - \sin^2 x)(\sin x + \cos x)}{\cos x - \sin x}\\
& = 2\sin x \cos x + \dfrac{(\cos x + \sin x)(\cos x - \sin x)(\sin x + \cos x)}
{\cos x - \sin x}\\
& = 2\sin x \cos x + (\cos x + \sin x)^2\\
& = 2\sin x \cos x + \cos^2 x + \sin^2 x + 2 \cos x \sin x\\
& = 2\sin x \cos x + 1 + 2 \cos x \sin x\\
& = 1 + 4\sin 2x \\
& = \text{RHS}
\end{align*}

$$\dfrac{\sin x + \sin 5x}{\cos x + \cos 5x} = \tan 3x$$\\


We can simplify LHS as \begin{align*}
\text{LHS} & = \dfrac{\sin x + \sin 5x}{\cos x + \cos 5x}\\
& = \dfrac{2\sin \bigg(\dfrac{x+5x}{2}\bigg) \cos \bigg(\dfrac{x-5x}{2}\bigg)}
{2\cos \bigg(\dfrac{x+5x}{2}\bigg) \cos \bigg(\dfrac{x-5x}{2}\bigg)}\ \ \ \ \ \ \ \
\ [\ \text{Sum-To-Product Formula}\ ]\\
& = \dfrac{\sin \bigg(\dfrac{6x}{2}\bigg)}{\cos \bigg(\dfrac{6x}{2}\bigg) }\\
& = \dfrac{\sin 3x}{\cos 3x }\\
& = \tan 3x \ \ \ \ \ \ \ \ \ \ \ \ \ \ \ \ \ \ \ \ \ \ \ \ \ \ \ \ \ \ \ \ \ \ \ \
\ \ \ [\ \text{Reciprocal Identity}\ ]\\
& = \text{RHS}
\end{align*}

$$\dfrac{\sin 3x + \sin 7x}{\cos 3x - \cos 7x} = \cot 2x$$\\


We can simplify LHS as \begin{align*}
\text{LHS} & = \dfrac{\sin 3x + \sin 7x}{\cos 3x - \cos 7x}\\
& = \dfrac{2\sin \bigg(\dfrac{3x+7x}{2}\bigg) \cos \bigg(\dfrac{3x-7x}{2}\bigg)}{-
2\sin \bigg(\dfrac{3x+7x}{2}\bigg) \sin \bigg(\dfrac{3x-7x}
{2}\bigg)}\ \ \ \ \ \ \ \ \ [\ \text{Sum-To-Product Formula}\ ]\\
& = \dfrac{\cos \bigg(\dfrac{-4x}{2}\bigg)}{-\sin \bigg(\dfrac{-4x}{2}\bigg)}\\
& = \dfrac{\cos (-2x)}{-\sin (-2x)}\\
& = \dfrac{\cos 2x}{\sin 2x}\ \ \ \ \ \ \ \ \ \ \ \ \ \ \ \ \ \ \ \ \ \ \ \ \ \ \ \
\ \ \ \ \ \ \ \ \ \ \ \ \ \ \ \ \ [\ \text{Even-Odd Identity}\ ]\\
& = \cot 2x \ \ \ \ \ \ \ \ \ \ \ \ \ \ \ \ \ \ \ \ \ \ \ \ \ \ \ \ \ \ \ \ \ \ \ \
\ \ \ \ \ \ \ \ \ \ [\ \text{Reciprocal Identity}\ ]\\
& = \text{RHS}
\end{align*}

$$\dfrac{\sin 10x}{\sin 9x + \sin x} = \dfrac{\cos 5x}{\cos 4x}$$\\


We can simplify LHS as \begin{align*}
\text{LHS} & = \dfrac{\sin 10x}{\sin 9x + \sin x}\\
& = \dfrac{\sin 2(5x)}{2\sin \bigg(\dfrac{9x+x}{2}\bigg) \cos \bigg(\dfrac{9x-x}
{2}\bigg)}\ \ \ \ \ \ \ \ \ [\ \text{Sum-To-Product Formula}\ ]\\
& = \dfrac{2\sin 5x \cos 5x}{2\sin \bigg(\dfrac{10x}{2}\bigg) \cos \bigg(\dfrac{8x}
{2}\bigg)}\ \ \ \ \ \ \ \ \ \ \ \ \ \ \ \ \ \ [\ \text{Double-Angle Formula}\ ]\\
& = \dfrac{\sin 5x \cos 5x}{\sin 5x \cos 4x}\\
& = \dfrac{\cos 5x}{\cos 4x}\\
& = \text{RHS}
\end{align*}

$$\dfrac{\sin x + \sin y}{\cos x + \cos y} = \tan \bigg(\dfrac{x+y}{2}\bigg)$$\\


We can simplify LHS as \begin{align*}
\text{LHS} & = \dfrac{\sin x + \sin y}{\cos x + \cos y}\\
& = \dfrac{2\sin \bigg(\dfrac{x+y}{2}\bigg) \cos \bigg(\dfrac{x-y}{2}\bigg)}
{2\cos \bigg(\dfrac{x+y}{2}\bigg) \cos \bigg(\dfrac{x-y}{2}\bigg)}\ \ \ \ \ \ \ \ \
[\ \text{Sum-To-Product Formula}\ ]\\
& = \dfrac{\sin \bigg(\dfrac{x+y}{2}\bigg)}{\cos \bigg(\dfrac{x+y}{2}\bigg) }\\
& = \tan \bigg(\dfrac{x+y}{2}\bigg) \ \ \ \ \ \ \ \ \ \ \ \ \ \ \ \ \ \ \ \ \ \ \ \
\ \ \ \ [\ \text{Reciprocal Identity}\ ]\\
& = \text{RHS}
\end{align*}

$$\tan y = \dfrac{\sin (x+y) - \sin (x - y)}{\cos (x+y) + \cos (x - y)}$$\\


We can simplify RHS as \begin{align*}
\text{RHS} & = \dfrac{\sin (x+y) - \sin (x - y)}{\cos (x+y) + \cos (x - y)}\\
& = \dfrac{2 \cos \Bigg(\dfrac{(x+y)+(x-y)}{2}\Bigg) \sin \Bigg(\dfrac{(x+y)-(x-y)}
{2}\Bigg)}{2\cos \Bigg(\dfrac{(x+y)+(x-y)}{2}\Bigg) \cos \Bigg(\dfrac{(x+y)-(x-y)}
{2}\Bigg)}\ \ \ \ \ \ [\ \text{Sum-To-Product Formula}\ ]\\
& = \dfrac{\sin \Bigg(\dfrac{2y}{2}\Bigg)}{\cos \Bigg(\dfrac{2y}{2}\Bigg)}\\
& = \dfrac{\sin y}{\cos y}\\
& = \tan
y \ \ \ \ \ \ \ \ \ \ \ \ \ \ \ \ \ \ \ \ \ \ \ \ \ \ \ \ \ \ \ \ \ \ \ \ \ \ \ \ \
\ \ \ \ \ \ \ \ \ \ \ \ \ \ \ \ \ \ \ \ \ \ \ \ \ \ [\ \text{Reciprocal
Identity}\ ]\\
& = \text{LHS}
\end{align*}

$$\tan^2 \bigg(\dfrac{x}{2} + \dfrac{\pi}{4}\bigg) = \dfrac{1+\sin x}{1 - \sin x}$


$\\
Using Formula for lowering powers, we can simplify LHS as \begin{align*}
\text{LHS} & = \tan^2 \bigg(\dfrac{x}{2} + \dfrac{\pi}{4}\bigg)\\
& = \dfrac{1 - \cos 2\bigg(\dfrac{x}{2} + \dfrac{\pi}{4} \bigg)}{1 + \cos
2\bigg(\dfrac{x}{2} + \dfrac{\pi}{4} \bigg)} \ \ \ \ \ \ \ \ \ \ \ [\ \text{Formula
for lowering powers}\ ]\\
& = \dfrac{1 - \cos \bigg(x + \dfrac{\pi}{2} \bigg)}{1 + \cos \bigg(x + \dfrac{\pi}
{2} \bigg)} \\
& = \dfrac{1 - (-\sin x)}{1 + (-\sin x)} \ \ \ \ \ \ \ \ \ \ \ \ \ \ \ \ \ \ \
[\ \text{Cofunction Identities}\ ]\\
& = \dfrac{1 + \sin x}{1 - \sin x} \\
& = \text{RHS}
\end{align*}
$$(1 - \cos 4x)(2 + \tan^2 x + \cot^2 x) = 8$$\\
We can simplify LHS as \begin{align*}
\text{LHS} & = (1 - \cos 4x)(2 + \tan^2 x + \cot^2 x)\\
& = \big(1 - \cos 2(2x)\big)(1 + \tan^2 x + 1 + \cot^2 x)\\
& = \big(1 - (1 - 2\sin^2 2x)\big)\big((1 + \tan^2 x) + (1 + \cot^2 x)\big) \ \ \ \
\ \ \ \ [\ \text{Double-Angle Formula}\ ]\\
& = (1 - 1 + 2\sin^2 2x)(\sec^2 x + \csc^2 x) \ \ \ \ \ \ \ \ \ \ \ \ \ \ \ \ \ \ \
\ \ \ \ \ \ \ \ [\ \text{Pythagorean Identity}\ ]\\
& = 2\sin^2 2x\bigg(\dfrac{1}{\cos^2 x} + \dfrac{1}{\sin^2 x}\bigg) \ \ \ \ \ \ \ \
\ \ \ \ \ \ \ \ \ \ \ \ \ \ \ \ \ \ \ \ \ \ \ \ \ \ \ \ [\ \text{Reciprocal
Identity}\ ]\\
& = 2(\sin 2x)^2\bigg(\dfrac{\sin^2 x + \cos^2 x}{\cos^2 x \sin^2 x}\bigg)\\
& = 2(2\sin x \cos x)^2\bigg(\dfrac{\sin^2 x + \cos^2 x}{\cos^2 x \sin^2
x}\bigg)\ \ \ \ \ \ \ \ \ \ \ \ \ \ \ \ \ \ \ \ \ \ \ \ \ \ \ \ [\ \text{Double-
Angle Formula}\ ]\\
& = 8\sin^2 x \cos^2 x\bigg(\dfrac{1}{\cos^2 x \sin^2 x}\bigg)\ \ \ \ \ \ \ \ \ \ \
\ \ \ \ \ \ \ \ \ \ \ \ \ \ \ \ \ \ \ \ \ \ \ [\ \text{Pythagorean Identity}\ ]\\
& = 8\\
& = \text{RHS}
\end{align*}

$$\sin 130^o - \sin 110^o = -\sin 10^o$$\\


The second Sum-To-Product Formula gives $$\sin u - \sin v = 2\cos \bigg(\dfrac{u +
v}{2}\bigg) \sin \bigg(\dfrac{u - v}{2}\bigg)$$
Let $u = 130^o$ and $v = 110^o$, then we get \begin{align*}
\text{LHS} & = \sin 130^o - \sin 110^o\\
& = 2\cos \bigg(\dfrac{130^o + 110^o}{2}\bigg) \sin \bigg(\dfrac{130^o - 110^o}
{2}\bigg)\\
& = 2\cos \bigg(\dfrac{240^o}{2}\bigg) \sin \bigg(\dfrac{20^o}{2}\bigg)\\
& = 2\cos 120^o \sin 10^o\\
& = 2\cos (90^o + 30^o) \sin 10^o\\
& = 2 (-\sin 30^o) \sin 10^o\ \ \ \ \ \ \ \ \ \ \ \ \ \ \ [\ \text{Cofunction
Identity}\ ]\\
& = 2 \bigg(-\dfrac{1}{2}\bigg) \sin 10^o\\
& = -\sin 10^o\\
& = \text{RHS}
\end{align*}

$$\cos 100^o - \cos 200^o = \sin 50^o$$\\


The fourth Sum-To-Product Formula gives $$\cos u - \cos v = -2\sin \bigg(\dfrac{u +
v}{2}\bigg) \sin \bigg(\dfrac{u - v}{2}\bigg)$$
Let $u = 100^o$ and $v = 200^o$, then we get \begin{align*}
\text{LHS} & = \cos 100^o - \cos 200^o\\
& = -2\sin \bigg(\dfrac{100^o + 200^o}{2}\bigg) \sin \bigg(\dfrac{100^o - 200^o}
{2}\bigg)\\
& = -2\sin \bigg(\dfrac{300^o}{2}\bigg) \sin \bigg(\dfrac{-100^o}{2}\bigg)\\
& = -2 \sin 150^o \sin (-50^o)\\
& = 2 \sin (90^o + 60^o) \sin 50^o\ \ \ \ \ \ \ \ \ \ \ \ \ \ \ \ \ \ \ [\
\text{Even-Odd Identities}\ ]\\
& = 2 (\cos 60^o) \sin 50^o\ \ \ \ \ \ \ \ \ \ \ \ \ \ \ \ \ \ \ \ \ \ \ \ \ \ \ [\
\text{Cofunction Identity}\ ]\\
& = 2 \bigg(\dfrac{1}{2}\bigg) \sin 50^o\\
& = \sin 50^o\\
& = \text{RHS}
\end{align*}

$$\sin 45^o + \sin 15^o = \sin 75^o$$\\


The first Sum-To-Product Formula gives $$\sin u + \sin v = 2\sin \bigg(\dfrac{u +
v}{2}\bigg) \cos \bigg(\dfrac{u - v}{2}\bigg)$$
Let $u = 45^o$ and $v = 15^o$, then we get \begin{align*}
\text{LHS} & = \sin 45^o + \sin 15^o\\
& = 2\sin \bigg(\dfrac{45^o + 15^o}{2}\bigg) \cos \bigg(\dfrac{45^o - 15^o}
{2}\bigg)\\
& = 2\sin \bigg(\dfrac{60^o}{2}\bigg) \cos \bigg(\dfrac{30^o}{2}\bigg)\\
& = 2\sin 30^o \cos 15^o\\
& = 2 \sin 30^o \cos (90^o - 75^o)\\
& = 2 \sin 30^o \sin 75^o\ \ \ \ \ \ \ \ \ \ \ \ \ \ \ \ \ \ \ \ \ \ \ \ \ \ \
[\ \text{Cofunction Identity}\ ]\\
& = 2 \bigg(\dfrac{1}{2}\bigg) \sin 75^o\\
& = \sin 75^o\\
& = \text{RHS}
\end{align*}

$$\cos 87^o + \cos 33^o = \sin 63^o$$\\


The third Sum-To-Product Formula gives $$\cos u + \cos v = 2\cos \bigg(\dfrac{u +
v}{2}\bigg) \cos \bigg(\dfrac{u - v}{2}\bigg)$$
Let $u = 87^o$ and $v = 33^o$, then we get \begin{align*}
\text{LHS} & = \cos 87^o + \cos 33^o\\
& = 2\cos \bigg(\dfrac{87^o + 33^o}{2}\bigg) \cos \bigg(\dfrac{87^o - 33^o}
{2}\bigg)\\
& = 2\cos \bigg(\dfrac{120^o}{2}\bigg) \cos \bigg(\dfrac{54^o}{2}\bigg)\\
& = 2\cos 60^o \cos 27^o\\
& = 2 \cos 60^o \cos (90^o - 63^o)\\
& = 2 \cos 60^o \sin 63^o\ \ \ \ \ \ \ \ \ \ \ \ \ \ \ \ \ \ \ \ \ \ \ \ \ \ \
[\ \text{Cofunction Identity}\ ]\\
& = 2 \bigg(\dfrac{1}{2}\bigg) \sin 63^o\\
& = \sin 63^o\\
& = \text{RHS}
\end{align*}

$$\dfrac{\sin x + \sin 2x + \sin 3x + \sin 4x + \sin 5x}{\cos x + \cos 2x + \cos 3x


+ \cos 4x + \cos 5x} = \tan 3x$$\\
Using Sum-To-Product Formula, we can simplify LHS as \begin{align*}
\text{LHS} & = \dfrac{\sin x + \sin 2x + \sin 3x + \sin 4x + \sin 5x}{\cos x + \cos
2x + \cos 3x + \cos 4x + \cos 5x}\\
& = \dfrac{(\sin x + \sin 5x) + (\sin 2x + \sin 4x) + \sin 3x}{(\cos x + \cos 5x) +
(\cos 2x + \cos 4x) + \cos 3x}\\
& = \dfrac{2\sin \dfrac{x+5x}{2} \cos \dfrac{x-5x}{2} + 2\sin \dfrac{2x+4x}{2} \cos
\dfrac{2x-4x}{2} + \sin 3x}{2\cos \dfrac{x+5x}{2} \cos \dfrac{x-5x}{2} + 2\cos
\dfrac{2x+4x}{2} \cos \dfrac{2x-4x}{2} + \cos 3x}\\
& = \dfrac{2\sin \dfrac{6x}{2} \cos \bigg(\dfrac{-4x}{2}\bigg) + 2\sin \dfrac{6x}
{2} \cos \bigg(\dfrac{-2x}{2}\bigg) + \sin 3x}{2\cos \dfrac{6x}{2} \cos
\bigg(\dfrac{-4x}{2}\bigg) + 2\cos \dfrac{6x}{2} \cos \bigg(\dfrac{-2x}{2}\bigg)
+ \cos 3x}\\
& = \dfrac{2\sin 3x \cos (-2x) + 2\sin 3x \cos (-x) + \sin 3x}{2\cos 3x \cos (-2x)
+ 2\cos 3x \cos (-x) + \cos 3x}\\
& = \dfrac{2\sin 3x \cos 2x + 2\sin 3x \cos x + \sin 3x}{2\cos 3x \cos 2x + 2\cos
3x \cos x + \cos 3x}\ \ \ \ \ \ \ \ \ \ \ \ \ [\ \text{Even-odd Identity}\ ]\\
& = \dfrac{\sin 3x(2\cos 2x + 2\cos x + 1)}{\cos 3x (2\cos 2x + 2\cos x + 1)} \ \ \
\ \ \ \ \ \ \ \ \ \ \ \ \ \ \ \ \ \ \ \ \ \ \ [\ \text{Taking common factor}\ ]\\
& = \dfrac{\sin 3x}{\cos 3x}\\
& = \tan
3x\ \ \ \ \ \ \ \ \ \ \ \ \ \ \ \ \ \ \ \ \ \ \ \ \ \ \ \ \ \ \ \ \ \ \ \ \ \ \ \ \
\ \ \ \ \ \ \ \ \ \ \ \ \ \ [\ \text{Reciprocal Identity}\ ]\\
& = \text{RHS}
\end{align*}

$$\dfrac{\sin x + \sin 3x + \sin 5x}{\cos x + \cos 3x + \cos 5x} = \tan 3x$$\\


Using Sum-To-Product Formula, we can simplify LHS as \begin{align*}
\text{LHS} & = \dfrac{\sin x + \sin 3x + \sin 5x}{\cos x + \cos 3x + \cos 5x}\\
& = \dfrac{(\sin x + \sin 5x) + \sin 3x}{(\cos x + \cos 5x) + \cos 3x}\\
& = \dfrac{2\sin \dfrac{x+5x}{2} \cos \dfrac{x-5x}{2} + \sin 3x}{2\cos \dfrac{x+5x}
{2} \cos \dfrac{x-5x}{2} + \cos 3x}\\
& = \dfrac{2\sin \dfrac{6x}{2} \cos \bigg(\dfrac{-4x}{2}\bigg) + \sin 3x}{2\cos
\dfrac{6x}{2} \cos \bigg(\dfrac{-4x}{2}\bigg) + \cos 3x}\\
& = \dfrac{2\sin 3x \cos (-2x) + \sin 3x}{2\cos 3x \cos (-2x) + \cos 3x}\\
& = \dfrac{2\sin 3x \cos 2x + \sin 3x}{2\cos 3x \cos 2x + \cos
3x}\ \ \ \ \ \ \ \ \ \ \ \ \ \ \ \ \ \ \ \ \ \ \ \ [\ \text{Even-odd
Identity}\ ]\\
& = \dfrac{\sin 3x(2\cos 2x + 1)}{\cos 3x (2\cos 2x + 1)} \ \ \ \ \ \ \ \ \ \ \ \ \
\ \ \ \ \ \ \ \ \ \ \ \ \ \ \ [\ \text{Taking common factor}\ ]\\
& = \dfrac{\sin 3x}{\cos 3x}\\
& = \tan
3x\ \ \ \ \ \ \ \ \ \ \ \ \ \ \ \ \ \ \ \ \ \ \ \ \ \ \ \ \ \ \ \ \ \ \ \ \ \ \ \ \
\ \ \ \ [\ \text{Reciprocal Identity}\ ]\\
& = \text{RHS}
\end{align*}

The Double-Angle Formula states that \begin{align*}


\sin 2x & = 2 \sin x \cos x\\
\\
\text{For}\ n = 1, \sin (2^nx) & = \sin 2x = 2 \sin x \cos x\\
\\
\text{For}\ n = 2, \sin (2^nx) & = \sin 4x\\
& = \sin 2(2x) = 2 \sin 2x \cos 2x\\
& = 2 (2\sin x \cos x) \cos 2x\\
& = 2^2 \sin x \cos x \cos 2^{2-1}x\\
\\
\text{For}\ n = 3, \sin (2^nx) & = \sin 8x\\
& = \sin 2(4x) = 2 \sin 4x \cos 4x\\
& = 2 (\sin 2(2x)) \cos 4x\\
& = 2 (2\sin 2x \cos 2x) \cos 4x\\
& = 4(2\sin x \cos x) \cos 2x \cos 4x\\
& = 2^3 \sin x \cos x \cos 2x \cos 2^{3-1}x\\
\end{align*}
From the above pattern, we can predict that $$\sin(2^n)x = 2^n \sin x \cos x \cos
2x \cos 4x .\ .\ .\ .\ .\ \cos 2^{n-1} x$$

$$2 \sin^{-1} x = \cos^{-1}(1 - 2x^2),\ \ \ 0 \leq x \leq 1$$\\


Let $u = \sin^{-1} x$, so that $x = \sin u$. Then the RHS changes to \begin{align*}
\text{RHS}\ & = \cos^{-1}(1 - 2x^2)\\
& = \cos^{-1}(1 - 2(\sin u)^2)\\
& = \cos^{-1}(1 - 2\sin^2 u)\\
& = \cos^{-1}(\cos 2u)\ \ \ \ \ \ \ \ \ \ \ \ \ \ [\ \text{Double-Angle
Formula}\ ]\\
& = 2u\\
& = 2\sin^{-1} x\\
& = \text{LHS}\\
\end{align*}

$$2 \tan^{-1} \bigg(\dfrac{1}{x}\bigg) = \cos^{-1}\bigg(\dfrac{x^2 - 1}


{x^2+1}\bigg)$$\\
Let $u = \tan^{-1} \bigg(\dfrac{1}{x}\bigg)$, so that $x = \dfrac{1}{\tan u} = \cot
u$. Then the RHS changes to \begin{align*}
\text{RHS}\ & = \cos^{-1}\bigg(\dfrac{x^2 - 1}{x^2+1}\bigg)\\
& = \cos^{-1}\bigg(\dfrac{\cot^2 u - 1}{\cot^2 u + 1}\bigg)\\
& = \cos^{-1}\Bigg(\dfrac{\cot^2 u - 1}{\csc^2 u}\Bigg)\ \ \ \ \ \ \ \ \ \ \ \ \ [\
\text{Pythagorean Identity}\ ]\\
& = \cos^{-1}\Bigg(\dfrac{\frac{\cos^2 u}{\sin^2 u} - 1}{\frac{1}{\sin^2 u}}\Bigg)\
\ \ \ \ \ \ \ \ \ \ \ \ \ \ [\ \text{Reciprocal Identity}\ ]\\
& = \cos^{-1}\Bigg(\dfrac{\frac{\cos^2 u - \sin^2 u}{\sin^2 u}}{\frac{1}{\sin^2
u}}\Bigg)\\
& = \cos^{-1}(\cos 2u)\ \ \ \ \ \ \ \ \ \ \ \ \ \ \ \ \ \ \ \ \ \ [\ \text{Double-
Angle Formula}\ ]\\
& = 2u\\
& = 2\tan^{-1} \bigg(\dfrac{1}{x}\bigg)\\
& = \text{LHS}\\
\end{align*}

(b)\ We can prove the above conjecture by expanding $f(x)$ to get $f(x) = 2$.\\
\begin{align*}
f(x) & = \dfrac{\sin 3x}{\sin x} - \dfrac{\cos 3x}{\cos x}\\
& = \dfrac{\sin 3x \cos x - \cos 3x \sin x}{\sin x \cos x}\\
& = \dfrac{\sin (3x - x)}{\sin x \cos x}\ \ \ \ \ \ \ \ \ \ \ \ \ \ \ \ [\
\text{Addition-Formula for Sine}\ ]\\
& = \dfrac{\sin 2x}{\sin x \cos x}\\
& = \dfrac{2 \sin x \cos x}{\sin x \cos x}\ \ \ \ \ \ \ \ \ \ \ \ \ \ \ \ [\
\text{Double-Angle Formula}\ ]\\
& = 2
\end{align*}
Hence Proved

(b)\ We can prove the above conjecture by expanding $f(x)$ to get $f(x) = 1$.\\
\begin{align*}
f(x) & = \cos 2x + 2 \sin^2 x\\
& = (1 - 2 \sin^2 x) + 2 \sin^2 x\ \ \ \ \ \ \ \ \ \ \ \ \ \ \ \ \ [\ \text{Double-
Angle Formula}\ ]\\
& = 1
\end{align*}
Hence Proved

(b)\ To verify that $f(x) = 2\cos \frac{1}{2}x \sin \frac{13}{2}x$, we use the
first Sum-To-Product Formula which is $$\sin u + \sin v = 2\sin \bigg(\dfrac{u + v}
{2}\bigg) \cos \bigg(\dfrac{u - v}{2}\bigg)$$
Given function is $$f(x) = \sin 6x + \sin 7x$$
Let $u = 6x$ and $v = 7x$, then we get \begin{align*}
\sin 6x + \sin 7x & = 2\sin \bigg(\dfrac{6x + 7x}{2}\bigg) \cos \bigg(\dfrac{6x -
7x}{2}\bigg)\\
& = 2\sin \bigg(\dfrac{13x}{2}\bigg) \cos \bigg(\dfrac{-x}{2}\bigg)\\
& = 2\sin \bigg(\dfrac{13x}{2}\bigg) \cos \bigg(\dfrac{x}{2}\bigg)\ \ \ \ \ \ \ \ \
\ \ [\ \text{Even-Odd Identities}\ ]
\end{align*}
Hence Proved

Let $3x = \pi/3$, and let $y = \cos x$. \\


\\
The result of of Example 2 is the Triple-Angle Formula for Cosine, i.e. $$\cos 3x =
4 \cos^3 x - 3 \cos x$$\\ Substituting $3x = \pi/3$ and $y = \cos x$ in the above
result, we get \begin{align*}
\cos 3x & = 4 \cos^3 x - 3 \cos x\\
\cos \bigg(\dfrac{\pi}{3}\bigg) & = 4 (\cos x)^3 - 3 \cos x\\
\dfrac{1}{2} & = 4y^3 - 3y\\
1 & = 8y^3 - 6y\\
8y^3 - 6y - 1 & = 0
\end{align*}\\
Hence Proved

Using Addition-Formula for Cosine again, we get \begin{align*}


\cos 5x & = \cos (3x + 2x)\\
& = \cos 3x \cos 2x - \sin 3x \sin 2x\\
& = (4 \cos^3 x - 3 \cos x)(2\cos^2 x - 1) - (3 \sin x - 4 \sin^3 x)(2\sin x \cos
x)\\
& = 8 \cos^5 x - 4 \cos^3 x - 6\cos^3 x + 3\cos x - 6 \sin^2 x \cos x + 8 \sin^4
x \cos x\\
& = 8 \cos^5 x - 10 \cos^3 x + 3\cos x - 6(1 - \cos^2 x) \cos x + 8 (\sin^2 x)^2
\cos x\\
& = 8 \cos^5 x - 10 \cos^3 x + 3\cos x - 6 \cos x + 6\cos^3 x + 8 (1 - \cos^2
x)^2 \cos x\\
& = 8 \cos^5 x - 4 \cos^3 x - 3\cos x + 8 (1 + \cos^4 x - 2\cos^2 x) \cos x\\
& = 8 \cos^5 x - 4 \cos^3 x - 3\cos x + 8\cos x + 8\cos^5 x - 16\cos^3 x\\
& = 16 \cos^5 x - 20 \cos^3 x + 5\cos x\\
\end{align*}
Therefore, we see that there is a polynomial $P(\cos^4 x) = 16 \cos^5 x - 20 \cos^3
x + 5\cos x$ such that $\cos 5x = P(cos^5 x)$
The result of Example 2 is the Triple-Angle Formula for Cosine, i.e. $$\cos 3x =
4 \cos^3 x - 3 \cos x$$\\
Now, using Addition-Formula for Cosine, we get \begin{align*}
\cos 4x & = \cos (3x + x)\\
& = \cos 3x \cos x - \sin 3x \sin x\\
& = (4 \cos^3 x - 3 \cos x)\cos x - (3 \sin x - 4 \sin^3 x)\sin x\\
& = 4 \cos^4 x - 3 \cos^2 x - 3 \sin^2 x + 4 \sin^4 x\\
& = 4 \cos^4 x - 3 \cos^2 x - 3 (1 - \cos^2 x) + 4 (\sin^2 x)^2\\
& = 4 \cos^4 x - 3 \cos^2 x - 3 + 3\cos^2 x + 4 (1 - \cos^2 x)^2\\
& = 4 \cos^4 x - 3 + 4 (1 + \cos^4 x - 2\cos^2 x)\\
& = 4 \cos^4 x - 3 + 4 + 4\cos^4 x - 8\cos^2 x\\
& = 8 \cos^4 x - 8\cos^2 x + 1\\
\end{align*}
Therefore, we see that there is a polynomial $P(\cos^4 x) = 8 \cos^4 x - 8\cos^2 x
+ 1$ such that $\cos 4x = P(cos^4 x)$

Given that $A,\ B$ and $C$ are the angles in a triangle. Then, \begin{align*}
\text{LHS} & = \sin 2A + \sin 2B + \sin 2C\\
& = 2\sin \bigg(\dfrac{2A+2B}{2}\bigg) \cos \bigg(\dfrac{2A-2B}{2}\bigg) + \sin 2C\
\ \ \ \ \ \ \ \ \ \ \ \ \ \ \ \ \ \ \ \ [\ \text{Sum-To-Product Formula}\ ]\\
& = 2\sin (A+B) \cos (A-B) + 2 \sin C \cos C\ \ \ \ \ \ \ \ \ \ \ \ \ \ \ \ \ \ \ \
\ \ \ \ \ [\ \text{Double-Angle Formula}\ ]\\
& = 2\sin (180^o - C) \cos (A-B) + 2\sin C \cos (180^o - (A+B))\\
& = 2\sin C \cos (A-B) - 2\sin C \cos
(A+B)\ \ \ \ \ \ \ \ \ \ \ \ \ \ \ \ \ \ \ \ \ \ \ \ \ \ [\ \text{Cofunction
Identities}\ ]\\
& = 2\sin C(\cos (A-B) - \cos (A+B))\\
& = 2\sin C \bigg[-2 \sin \bigg(\dfrac{(A-B)+(A+B)}{2}\bigg) \sin \bigg(\dfrac{(A-
B)-(A+B)}{2}\bigg)\bigg]\\
& = -4 \sin C \bigg[\sin \bigg(\dfrac{2A}{2}\bigg) \sin \bigg(\dfrac{-2B}
{2}\bigg)\bigg]\\
& = -4 \sin C [\sin A \sin (-B)]\\
& = 4 \sin C [\sin A \sin B]\ \ \ \ \ \ \ \ \ \ \ \ \ \ \ \ \ \ \ \ \ \ \ \ \ \ \ \
\ \ \ \ \ \ \ \ \ \ \ \ \ \ \ \ \ \ \ \ \ \ \ \ [\ \text{Even-Odd Identity}\ ]\\
& = 4 \sin A \sin B \sin C\\
& = \text{RHS}
\end{align*}
Hence Proved

Given that a rectangle is inscribed in a semicircle of radius 5 cm as shown in the


figure below.\\
\\
(a)\ Now, in the figure, $OA = 5$ cm since it is the radius of the circle. Then,
using trigonometric ratios for the $\Delta$OAB, we have $$OB = 5 \cos \theta\
\text{and}\ AB = 5 \sin \theta$$
Now, the dimensions of the rectangle will be $$\text{length}\ = 2 OB = 10 \cos
\theta\ \text{and width}\ = AB = 5 \sin \theta$$
Therefore, area of the rectangle is \begin{align*}
\text{Area}\ & = \text{length}\ \times \text{width}\\
& = 10 \cos \theta \times 5 \sin \theta\\
& = 25 \times 2 \sin \theta \cos \theta\\
A(\theta) & = 25 \sin 2 \theta \ \ \ \ \ \ \ \ \ \ \ \ \ \ \ \ \ \ \ \ \ \ \ [\
\text{Double-Angle Formula}\ ]
\end{align*}

(b)\ From the above part, we saw that the area of rectangle is modeled by the
function $$A(\theta) = 25 \sin 2 \theta$$
It will be maximum when $\sin 2 \theta$ is maximum.\\
\\
Now, we know that $\sin u$ achieves its maximum value at $u = \pi/2$. So, $\sin
2 \theta$ will achieve its maximum value at $$2 \theta = \pi/2\ \Rightarrow \theta
= \pi/4$$ Therefore the largest possible area for such inscribed rectangle will be
$$\text{Area} = 25 \sin 2(\pi/4) = 25 \sin \pi/2 = 25$$\\
\\
(c)\ Dimensions of the inscribed rectangle with the largest possible area are
\begin{align*}
\text{length}\ & = 10 \cos \theta\\
& = 10 \cos \pi/4\\
& = \dfrac{10}{\sqrt{2}}\\
\\
\text{width}\ & = 5 \sin \theta\\
& = 5 \sin \pi/4\\
& = \dfrac{5}{\sqrt{2}}
\end{align*}

(b)\ From the above part, we saw that the area of cross-section of rectangular beam
is modeled by the function $$A(\theta) = 200 \sin 2 \theta$$
It will be maximum when $\sin 2 \theta$ is maximum.\\
\\
Now, we know that $\sin u$ achieves its maximum value at $u = \pi/2$. So, $\sin
2 \theta$ will achieve its maximum value at $$2 \theta = \pi/2\ \Rightarrow \theta
= \pi/4$$ Therefore the maximum possible area for such a beam will be $$\text{Area}
= 200 \sin 2(\pi/4) = 200 \sin \pi/2 = 200\ \text{in.}^2$$\\

Given that a rectangular beam is cut from a cylindrical log of diameter 20 in. in
one of the ways as shown in the figure below.\\
\\
(a)\ The cross-sectional area $A$ of the beam is \begin{align*}
A & = \text{Area of the rectangle}\ ABCD \\
& = (\text{length})(\text{depth})
\end{align*}\\
In the $\Delta$ACD, we have \begin{align*}
\text{width} & = 20 \cos \theta \\
\text{length} & = 20 \sin \theta
\end{align*}\\
Therefore, area of the cross-section of the beam is \begin{align*}
A(\theta) & = (20 \sin \theta)(20 \cos \theta)\\
& = 200 (2\cos \theta \sin \theta)\\
A(\theta) & = 200 \sin 2 \theta \ \ \ \ \ \ \ \ \ \ \ \ \ \ \ \ \ \ \ \ \ \ \ [\
\text{Double-Angle Formula}\ ]
\end{align*}\\

The given situation is drawn and labelled as shown below. Since the paper is folded
from the corner $D$ to the left-hand edge at $A$, we have the following equal
segments. $$CD = AC\ \ \text{and}\ \ \ AB = BD$$
Since segment $BC$ is common for the triangles $ABC$ and $BCD$, we have $$\Delta
ABC \cong \Delta BCD$$ Therefore, by CPCTC, we have $$\angle ABC \cong \angle
DBC \cong \theta$$
Also, width of paper is $AE = 6$ in.\\
\\
Now, in $\Delta$BCD, we have \begin{align*}
\cos \theta & = \dfrac{BD}{BC}\\
BD & = BC\cos \theta\\
& = L \cos \theta\ .\ .\ .\ .\ .\ .\ .\ .\ .\ .\ .\ (1)
\end{align*}\\
\\
Now, in $\Delta$ABE, we have \begin{align*}
\sin 2\theta & = \dfrac{AE}{AB}\\
2\sin \theta \cos \theta & = \dfrac{6}{BD}\ \ \ \ \ \ \ \ \ \ \ \ \ \ \ \ [\
\text{Since}\ AB = BD\ ]\\
\sin \theta \cos \theta & = \dfrac{3}{L \cos \theta}\ \ \ \ \ \ \ \ \ \ \ \ [\
\text{from}\ (1)\ ]\\
L & = \dfrac{3}{\sin \theta \cos^2 \theta}
\end{align*}
Hence Proved

(b)\ To verify that $f(x) = 2\cos t \cos 12t$, we use the first Sum-To-Product
Formula which is $$\cos u + \cos v = 2\cos \bigg(\dfrac{u + v}{2}\bigg) \cos
\bigg(\dfrac{u - v}{2}\bigg)$$
Given function is $$f(t) = \cos 11t + \cos 13t$$
Let $u = 11t$ and $v = 13t$, then we get \begin{align*}
\cos 11t + \cos 13t & = 2\cos \bigg(\dfrac{11t + 13t}{2}\bigg) \cos
\bigg(\dfrac{11t - 13t}{2}\bigg)\\
& = 2\cos \bigg(\dfrac{24t}{2}\bigg) \cos \bigg(\dfrac{-2t}{2}\bigg)\\
& = 2\cos 12t \cos (-t)\\
& = 2\cos 12t \cos t\ \ \ \ \ \ \ \ \ \ \ [\ \text{Even-Odd Identities}\ ]
\end{align*}
Hence Proved

(c)\ The graphs of $y = 2 \cos t$ and $y = -2 \cos t$ together with the graph in
part (a) is shown below.\\
\\
From the below graph, we see that the graph of $f$ lies between the graphs of $y =
2 \cos t$ and $y = -2 \cos t$ and therefore, the loudness of the sound varies
between $-2 \cos t$ and $2 \cos t$.

(a)\ From the given information in the figure, when the 4 key is pressed, a tone of
low frequency $f_1 = 770$ Hz is produced along with a high frequency $f_2 = 1209$
Hz.\\
\\
It is given that pressing a key produces the sound wave $$y = \sin(2 \pi f_1 t)
+ \sin (2 \pi f_2 t)$$
Therefore, the function that models the sound produced when 4 key is pressed will
be $$y = \sin(2 \pi 770 t) + \sin (2 \pi 1209 t)$$ or $$y = \sin(1540\pi t) + \sin
(2418 \pi t)$$

(b)\ The first Sum-To-Product Formula gives $$\sin u + \sin v = 2\sin


\bigg(\dfrac{u + v}{2}\bigg) \cos \bigg(\dfrac{u - v}{2}\bigg)$$
Let $u = 1540 \pi t$ and $v = 2418 \pi t$, then we get \begin{align*}
\text{LHS} & = \sin (1540 \pi t) + \sin (2418 \pi t)\\
& = 2\sin \bigg(\dfrac{1540 \pi t + 2418 \pi t}{2}\bigg) \cos \bigg(\dfrac{1540 \pi
t - 2418 \pi t}{2}\bigg)\\
& = 2\sin \bigg(\dfrac{ 3958 \pi t}{2}\bigg) \cos \bigg(\dfrac{-878 \pi t}
{2}\bigg)\\
& = 2\sin (1979 \pi t) \cos (-439 \pi t)\\
& = 2\sin (1979 \pi t) \cos (439 \pi
t)\ \ \ \ \ \ \ \ \ \ \ \ \ \ \ \ \ \ \ \ \ \ \ \ \ \ \ [\ \text{Even-Odd
Identity}\ ]\\
\end{align*}

In the below figure, radius $OA = OB = 1$ unit and $\angle$BAC = $\theta$ is given.
\\
\\
Now, we know that an angle inscribed in a semicircle is a right angle, so
$\angle$ACB = $90^o$. Therefore, in $\Delta$ABC, we have \begin{align*}
\cos \theta & = \dfrac{AC}{AB}\\
AC & = AB \cos \theta\\
& = 2 \cos \theta\\
\\
\sin \theta & = \dfrac{BC}{AB}\\
BC & = AB \sin \theta\\
& = 2 \sin \theta
\end{align*}
Thus, area of the $\Delta$ABC will be \begin{align*}
\text{Area}\ & = \dfrac{1}{2} \times BC \times AC\\
& = \dfrac{1}{2}(2 \sin \theta)(2 \cos \theta)\\
& = 2 \sin \theta \cos \theta\ .\ .\ .\ .\ .\ .\ .\ .\ .\ .\ .\ (1)
\end{align*}\\

Now, we also know that the central angle subtended by the chord of a circle is
twice the angle subtended by the chord on the circle, so $$\angle BOC = 2 \angle
BAC = 2\theta$$\\
Now, let's draw an altitude from $C$ to $D$ on the diameter $AB$. Then, in the
$\Delta$ODC, we have \begin{align*}
\sin 2\theta & = \dfrac{CD}{OC}\\
CD & = OC \sin 2 \theta\\
& = \sin 2 \theta\ \ \ \ \ \ \ \ \ \ \ \ \ \ \ \ \ [\ \text{Since}\ OC = OB = OA =
1\ ]
\end{align*}\\
Thus, area of the $\Delta$ABC will be \begin{align*}
\text{Area}\ & = \dfrac{1}{2} \times AB \times CD\\
& = \dfrac{1}{2}(2)(\sin 2\theta)\\
& = \sin 2\theta\ .\ .\ .\ .\ .\ .\ .\ .\ .\ .\ (2)
\end{align*}\\
Therefore, from the information (1) and (2), we prove the Double-Angle Formula $
$\sin 2 \theta = 2 \sin \theta \cos \theta$$

Because the trigonometric functions are periodic, if a basic trigonometric equation


has one solution, it has $\mathbf{infintely\ many}$ solutions.\\
\\
For example, for the equation, $\sin x = 0$ when $x = 0$, $x$ has infinitely many
solutions such that $$x = n\pi,\ \ \text{where}\ n = \text{any integer}$$

Since the graph of $\sin x$ lies between $-1$ and $1$, the basic equation $\sin x =
2$ has $\mathbf{no}$ solutions, whereas the basic equation $\sin x = 0.3$ has
$\mathbf{infinitely\ many}$ solutions. \\
\\
This can be seen from the graph of $\sin x$ below.

We can find some of the solutions of $\sin x = 0.3$ graphically by graphing $y =


\sin x$ and $\boxed{y = 2}$ as shown in the figure below.\\
\\
\\
From the below figure, we see that some of the solutions are $$x \approx -9.7,\
-6,\ -3.4,\ 0.3,\ 2.8,\ 6.6\ \text{and}\ 9.1$$

In the given figure, let the line segment $s$ that bisects angle $C$ meet side $AB$
of the triangle at $D$. Let $BD = c_1$ and $AD = c_2$ and $AB = c$.\\
\\
Now, using the Law of Sines for the $\Delta$ABC, we get \begin{align*}
\dfrac{AB}{\sin BCA} & = \dfrac{BC}{\sin A} = \dfrac{AC}{\sin B}\\
\dfrac{c}{\sin 2x} & = \dfrac{a}{\sin A} = \dfrac{b}{\sin B}
\end{align*}\\
Therefore, we have $$\sin A = \dfrac{a \sin 2x}{c}\ \text{and}\ \sin B =
\dfrac{b \sin 2x}{c}$$

Now, using the Law of Sines for the $\Delta$BCD, we get \begin{align*}
\dfrac{BD}{\sin BCD} & = \dfrac{CD}{\sin B}\\
\dfrac{c_1}{\sin x} & = \dfrac{s}{\sin B}\\
\dfrac{c_1}{s} & = \dfrac{\sin x}{\sin B} .\ .\ .\ .\ .\ .\ .\ .\ .\ .\ .\ (1)
\end{align*}\\
Now, using the Law of Sines for the $\Delta$ACD, we get \begin{align*}
\dfrac{AD}{\sin ACD} & = \dfrac{CD}{\sin A}\\
\dfrac{c_2}{\sin x} & = \dfrac{s}{\sin A}\\
\dfrac{c_2}{s} & = \dfrac{\sin x}{\sin A}\ .\ .\ .\ .\ .\ .\ .\ .\ .\ .\ .\ (2)
\end{align*}\\
Adding (1) and (2), we get \begin{align*}
\dfrac{c_1 + c_2}{s} & = \sin x\bigg(\dfrac{1}{\sin A} + \dfrac{1}{\sin B}\bigg)\\
\dfrac{c}{s} & = \sin x \bigg( \dfrac{c}{a\sin 2x} + \dfrac{c}{b\sin 2x} \bigg)\\
& = \dfrac{c \sin x}{\sin 2x} \bigg(\dfrac{a+b}{ab}\bigg)\\
\dfrac{1}{s} & = \dfrac{\sin x}{2\sin x \cos x} \bigg(\dfrac{a+b}{ab}\bigg)\\
& = \dfrac{1}{2\cos x} \bigg(\dfrac{a+b}{ab}\bigg)\\
s & = \dfrac{2ab \cos x}{a + b}
\end{align*}\\

We can find the solutions of $\sin x = 0.3$ algebraically.\\


\\
(a)\ First we find the solutions in the interval $[0,2\pi)$. We get one such
solution by taking $\sin^{-1}$ to get $x \approx 0.30$ as shown below.
\begin{align*}
\sin x & = 0.3\\
\sin^{-1}(\sin x) & = \sin^{-1}(0.3)\ \ \ \ \ \ \ \ \ \ \ \ [\ \text{Take}\ \sin^{-
1}\ \text{of each side}\ ]\\
x & \approx \boxed{0.30}
\end{align*}\\
Since sine is positive in Quadrant II, the solutions in an interval of length $2
\pi$ are $\theta$ and $\pi - \theta$. Therefore, the other solution in this
interval is $$x \approx \pi - 0.3 = 3.14 - 0.3 = \boxed{2.84}$$

Since sine is a periodic function with period $2 \pi$, it means that the graph of
$\sin x$ repeats after a period of $2 \pi$ and so we find all solutions by adding
multiples of $\boxed{2\pi}$ to the solutions in $[0,2\pi)$.\\
\\
The solutions are therefore $$\boxed{0.30 + 2n\pi}\ \text{and}\ \boxed{2.84 +
2n\pi}$$ where $n$ is any integer.

If $v_1$ is the speed of light in one medium and $v_2$ is its speed in another
medium, then according to Snell's Law, $$\dfrac{\sin \theta_1}{\sin \theta_2} =
\dfrac{v_1}{v_2}$$ where $\theta_1$ is the angle of incidence and $\theta_2$ is the
angle of refraction. The number $v_1/v_2$ is called the index of refraction. \\
\\
Now, it is given that a ray of light passes through the surface of a lake at an
angle of incidence of $70^o$. Also, index of refraction from air to water is equal
to $1.33$. Then, according to the Snell's Law, we have \begin{align*}
\dfrac{\sin 70^0}{\sin \theta_2} & = 1.33\\
\sin \theta_2 & = \dfrac{\sin 70^o}{1.33}\\
& = \dfrac{0.94}{1.33} = 0.70676691729\\
\theta & = \sin^{-1} (0.70676691729)\\
& \approx 44.97^o
\end{align*}\\
Therefore, the angle of refraction is approximately 45 degrees.

If $v_1$ is the speed of light in one medium and $v_2$ is its speed in another
medium, then according to Snell's Law, $$\dfrac{\sin \theta_1}{\sin \theta_2} =
\dfrac{v_1}{v_2}$$ where $\theta_1$ is the angle of incidence and $\theta_2$ is the
angle of refraction. The number $v_1/v_2$ is called the index of refraction. \\
\\
Now, when $\theta_2$ is $90^o$ or larger, the light beam is reflected back and this
phenomenon is called $total\ internal\ reflection$. Let $\theta_1 = \theta_c$ be
the critical angle of incidence at which this phenomenon occurs. Now, we know that
when light passes from air to glass, index of refraction $v_1/v_2 = 1.52$, so, when
light passes from denser (glass) to rarer (air) medium, the index of refraction
becomes $v_1/v_2 = \dfrac{1}{1.52}$.\\
\\
Let $\theta_2 = 90^o$, i.e. the angle of refraction at which the total internal
reflection begins to occur when light passes from glass to air. Then, according to
the Snell's Law, we have \begin{align*}
\dfrac{\sin \theta_1}{\sin \theta_2} & = \dfrac{v_1}{v_2}\\
\dfrac{\sin \theta_c}{\sin 90^o} & = \dfrac{1}{1.52}\\
\dfrac{\sin \theta_c}{1} & = 0.658\\
\theta_c & = \sin^{-1} (0.658)\\
& \approx 41.15^o
\end{align*}\\
Therefore, the critical angle of incidence is approximately 41.15 degrees.

An astronomical measure of phase is given by the fraction $F$ of the lunar disc
that is lit. When the angle between the sun, earth and the moon is $\theta (0
\leq \theta \leq 360^o)$, then $$F = \dfrac{1}{2}(1 - \cos \theta)$$\\
\\
(a)\ $F= 0$ (new moon)\\
\\
Substituting $F = 0$, in the equation, we get \begin{align*}
F & = \dfrac{1}{2}(1 - \cos \theta)\\
0 & = \dfrac{1}{2}(1 - \cos \theta)\\
1 - \cos \theta & = 0\\
\cos \theta & = 1\\
\theta & = \cos^{-1} (1)\\
& = 0^o\ \text{or}\ 360^o
\end{align*}\\

An $\mathbf{equation}$ is a statement that two mathematical expressions are equal.


For example, the following are equations : $$ 2x^2 + 4x + 3 = 0$$
$$e^x = 2$$
$$\sin^2 x + \cos^2 x = 1$$\\
On the other hand, an $\mathbf{identity}$ is an equation that is true for all
values of the variable(s). For example, the last equation in the above example is
an identity as it is true for all values of $x$ whereas the other two equations are
not.\\
\\
Therefore, the statement $A$ is true whereas the statement $B$ is not.

The equation $$\sin x + \sin 2x = 0$$ is equivalent to the equation \begin{align*}


\sin x + \sin 2x & = 0\\
\boxed{\sin x + 2 \sin x \cos x = 0} & \ \ \ \ \ \ \ \ \ \ \ \ [\ \text{Double-
Angle Formula}\ ]\\
\sin x(1 + 2 \cos x) & = 0\\
\end{align*}
Upon factoring, we see that solving this equation is equivalent to solving the two
basic equations $$\boxed{\sin x= 0}\ \text{and}\ \boxed{1 + 2 \cos x = 0}$$

$$2 \sin 2 \theta = 1$$\\


From previous exercise, we saw that $$\boxed{\theta = \dfrac{\pi}{12} + n\pi}\
\text{or}\ \boxed{\theta = \dfrac{5\pi}{12} + n \pi}$$ where $n$ is any integer.\\
\\
For $n = 0$, \begin{align*}
\theta & = \dfrac{\pi}{12} + 0 = \dfrac{\pi}{12}\\
\\
\theta & = \dfrac{5\pi}{12} + 0 = \dfrac{5\pi}{12}
\end{align*}
For $n = 1$, \begin{align*}
\theta & = \dfrac{\pi}{12} + \pi = \dfrac{13\pi}{12}\\
\\
\theta & = \dfrac{5\pi}{12} + \pi = \dfrac{17\pi}{12}
\end{align*}
For $n = 2$, \begin{align*}
\theta & = \dfrac{\pi}{12} + 2\pi = \dfrac{25\pi}{12}\\
\\
\theta & = \dfrac{5\pi}{12} + 2\pi = \dfrac{29\pi}{12}\\
\end{align*}
Therefore, from above, we see that the solutions in the interval $[0,2\pi)$ are $
$\theta = \dfrac{\pi}{12},\ \dfrac{5\pi}{12},\ \dfrac{13\pi}{12},\ \dfrac{17\pi}
{12}$$ For $n \geq 2$, the solutions lie outside the interval $[0,2\pi)$.

$$\csc 3 \theta = 5 \sin 3 \theta$$\\


(a)\ Using Reciprocal identity, we solve the above equation as \begin{align*}
\dfrac{1}{\sin 3 \theta} & = 5 \sin 3 \theta\\
5\sin^2 3 \theta & = 1\\
\sin^2 3 \theta & = \dfrac{1}{5}\\
\sin 3 \theta & = \pm \dfrac{1}{\sqrt{5}}\\
3 \theta & = \sin^{-1} \bigg(\dfrac{1}{ \sqrt{5}}\bigg)\ \ \text{or}\ \ \sin^{-
1} \bigg(-\dfrac{1}{ \sqrt{5}}\bigg)\\
& \approx 0.464\ \text{radians}\ \text{and}\ 2.677\ \text{radians}\\
&\ \text{or}\ 5.819\ \text{radians}\ \text{and}\ 3.616\ \text{radians}
\end{align*}\\
Now, we can get the general solution for $3\theta$ by adding $2k\pi$ because the
period of the sine function is $2\pi$ where $k$ is any integer. \\ \begin{align*}
3 \theta & \approx 0.464 + 2k\pi\ \ \ \text{and}\ \ \ 2.677 + 2k\pi\\
\text{or}\ & \approx 5.819 + 2k\pi\ \ \ \text{and}\ \ \ 3.616 + 2k\pi\\
\\
\text{Thus}\ & \boxed{\theta \approx 0.155 + \dfrac{2k\pi}{3}\ \ \ \text{and}\ \ \
0.893 + \dfrac{2k\pi}{3}}\\
\text{or}\ & \boxed{ \theta \approx 1.94 + \dfrac{2k\pi}{3}\ \ \ \text{and}\ \ \
1.202 + \dfrac{2k\pi}{3}}
\end{align*}

(b)\ The solutions in the interval $[0,2\pi)$ can be found out by substituting
different integer values for $k$.\\
\begin{align*}
\text{For}\ k = 0,\ \ \ \ \theta = 0.155 + 0 & = 0.155\ \text{radians}\\
\theta = 0.893 + 0 & = 0.893\ \text{radians}\\
& \text{or}\ \\
\theta = 1.94 + 0 & = 1.94\ \text{radians}\\
\theta = 1.202 + 0 & = 1.202\ \text{radians}\\
\\
\text{For}\ k = 1,\ \ \ \ \theta = 0.155 + \dfrac{2\pi}{3} & = 2.25\
\text{radians}\\
\theta = 0.893 + \dfrac{2\pi}{3} & = 2.986\ \text{radians}\\
& \text{or}\ \\
\theta = 1.94 + \dfrac{2\pi}{3} & = 4.034\ \text{radians}\\
\theta = 1.202 + \dfrac{2\pi}{3} & = 3.296\ \text{radians}\\
\\
\text{For}\ k = 2,\ \ \ \ \theta = 0.155 + \dfrac{4\pi}{3} & = 4.344\
\text{radians}\\
\theta = 0.893 + \dfrac{4\pi}{3} & = 5.08\ \text{radians}\\
& \text{or}\ \\
\theta = 1.94 + \dfrac{4\pi}{3} & = 6.129\ \text{radians}\\
\theta = 1.202 + \dfrac{4\pi}{3} & = 5.391\ \text{radians}
\end{align*}
For $n \geq 3$, the solutions lie outside the interval $[0,2\pi)$.\\
\\
\\
Therefore, from above, we see that the solutions in the interval $[0,2\pi)$ are $
$\theta = 0.155,\ 0.893,\ 1.94,\ 1.202,\ 2.25,\ 2.986,\ 4.034,\ 3.296,\ 4.344,\
5.08,\ 6.129,\ 5.391$$ where $\theta\ \text{is in radians}$.\\

$$1 - 2 \sin \theta = \cos 2 \theta$$\\


(a)\ Using Double-Angle Formula, we solve the above equation as \begin{align*}
1 - 2 \sin \theta = \cos 2 \theta\\
1 - 2 \sin \theta = 1 - 2\sin^2 \theta\\
\sin \theta - \sin^2 \theta & = 0\\
\sin \theta(1 - \sin \theta) & = 0\\
\sin \theta = 0\ \text{or}\ & \sin \theta = 1\\
\theta = \sin^{-1} (0)\ & \ \text{or}\ \ \sin^{-1} (1)\\
= 0\ \text{and}\ \pi\ & \text{radians} \ \text{or}\ \pi/2\ \text{radians}\\
\end{align*}
Now, we can get the general solution for $\theta$ by adding $2k\pi$ because the
period of the sine function is $2\pi$ where $k$ is any integer. \\ \begin{align*}
\theta & = 0 + 2k\pi\ \text{and}\ \pi + 2k\pi \ \ \ \text{or}\ \ \ \dfrac{\pi}{2}
+ 2k\pi
\end{align*}
It can be rewritten as $$\boxed{ \theta = k\pi\ \ \ \text{or}\ \ \ \dfrac{\pi}{2} +
2k\pi}$$ where $k$ is any integer.\\

$$1 - 2\sin \theta = \cos 2\theta$$\\


From previous exercise, we saw that $$\boxed{\theta = k\pi}\ \text{or}\
\boxed{\theta = \dfrac{\pi}{2} + 2k \pi}$$ where $k$ is any integer.\\
\\
For $k = 0$, \begin{align*}
\theta = 0 \ \text{or}\ \ & \theta = \dfrac{\pi}{2} + 0 = \dfrac{\pi}{2}
\end{align*}
For $k = 1$, \begin{align*}
\theta & = \pi\ \text{or}\ \ \theta = \dfrac{\pi}{2} + 2\pi = \dfrac{5\pi}{2}
\end{align*}\\
Therefore, from above, we see that the solutions in the interval $[0,2\pi)$ are $
$\theta = 0,\ \dfrac{\pi}{2},\ \pi$$ For $k \geq 2$, the solutions lie outside the
interval $[0,2\pi)$.\\

$$\tan 3 \theta + 1 = \sec 3 \theta$$\\


(a)\ Using Reciprocal identity, we solve the above equation as \begin{align*}
\dfrac{\sin 3 \theta}{\cos 3 \theta} + 1 & = \dfrac{1}{\cos 3 \theta}\\
\dfrac{\sin 3 \theta + \cos 3 \theta}{\cos 3 \theta} & = \dfrac{1}{\cos 3 \theta}\\
\sin 3 \theta + \cos 3 \theta & = 1\\
\dfrac{1}{\sqrt{2}}\sin 3 \theta + \dfrac{1}{\sqrt{2}}\cos 3 \theta & = \dfrac{1}
{\sqrt{2}}\\
\sin \bigg(3 \theta + \dfrac{\pi}{4}\bigg) & = \dfrac{1}{\sqrt{2}}\ \ \ \ \ \ \ \ \
\ \ \ [\ \text{Sum-To-Product Formula}\ ]\\
3 \theta + \dfrac{\pi}{4} & = \sin^{-1}\bigg(\dfrac{1}{\sqrt{2}}\bigg)\\
& = \dfrac{\pi}{4}\ \text{or}\ \dfrac{3\pi}{4}\\
3\theta & = 0\ \text{or}\ \dfrac{\pi}{2}
\end{align*}\\
Now, we can get the general solution for $3\theta$ by adding $2k\pi$ because the
period of the sine function is $2\pi$ where $k$ is any integer. \\ \begin{align*}
3 \theta & = 0 + 2k\pi\ \ \ \text{and}\ \ \ \dfrac{\pi}{2} + 2k\pi\\
\\
\text{Thus}\ & \boxed{\theta = \dfrac{2k\pi}{3}\ \ \ \text{and}\ \ \ \dfrac{\pi}{6}
+ \dfrac{2k\pi}{3}}\\
\end{align*}\\

(b)\ The solutions in the interval $[0,2\pi)$ can be found out by substituting
different integer values for $k$.\\
\begin{align*}
\text{For}\ k = 0,\ \ \ \ \theta = 0\\
\theta = \dfrac{\pi}{6} + 0 & = \dfrac{\pi}
{6}\ \ \ \ \ \ \ \ \ \ \ \ \ \ \ \ \ \ \ \ \ \ \ \ \ \ [\ \tan 3\theta\ \text{is
not defined}\ ]\\
\\
\text{For}\ k = 1,\ \ \ \ \theta & = \dfrac{2\pi}{3} \\
\theta & = \dfrac{\pi}{6} + \dfrac{2\pi}{3} = \dfrac{5\pi}{6}\ \ \ \ \ \ \ \ \ \ [\
\tan 3\theta\ \text{is not defined}\ ]\\
\\
\text{For}\ k = 2,\ \ \ \ \theta & = \dfrac{4\pi}{3} \\
\theta & = \dfrac{\pi}{6} + \dfrac{4\pi}{3} = \dfrac{9\pi}{6}\ \ \ \ \ \ \ \ \ \ [\
\tan 3\theta\ \text{is not defined}\ ]\\
\end{align*}
For $n \geq 3$, the solutions lie outside the interval $[0,2\pi)$.\\
\\
\\
Therefore, from above, we see that the solutions in the interval $[0,2\pi)$ are $
$\theta = 0,\ \dfrac{2\pi}{3},\ \dfrac{4\pi}{3}$$ where $\theta\ \text{is in
radians}$.\\

$$3 \tan^3 \theta - 3\tan^2 \theta - \tan \theta + 1 = 0$$\\


We factor the left-hand side of the equation. \begin{align*}
3 \tan^3 \theta - 3\tan^2 \theta - \tan \theta + 1 & = 0\\
3 \tan^2 \theta(\tan \theta - 1) - (\tan \theta - 1) & = 0\\
(3\tan^2 \theta - 1)(\tan \theta - 1) & = 0\\
3 \tan^2 \theta - 1 = 0\ \ \text{or}\ \ & \tan \theta -1 = 0\\
\tan \theta = \pm \dfrac{1}{\sqrt{3}}\ \ \text{or}\ \ & \tan \theta = 1\\
\theta = \tan^{-1} \bigg( \pm \dfrac{1}{\sqrt{3}}\bigg)\ \ \text{or}\ \ & \theta
= \tan^{-1} (1)\\
\end{align*}
Because tangent has period $\pi$, we first find the solutions in any interval of
length $\pi$. In the interval $(0,\pi)$, the solutions for the first equation are
$\theta = \dfrac{\pi}{6}$ and $\dfrac{5\pi}{6}$ and for the second equation, the
solution is $\theta = \dfrac{\pi}{4}$.\\
\\
\\
To get all solutions, we add integer multiples of $\pi$ to these solutions : $
$\theta = \dfrac{\pi}{6} + k\pi\ \text{and}\ \ \dfrac{5\pi}{6} + k\pi\ \ \
\text{or}\ \ \ \theta = \dfrac{\pi}{4} + k\pi$$
where $k$ is any integer.\\
\\
All the solutions in the interval $[0,2\pi)$ will be \begin{align*}
\text{For}\ k = 0,\ \ \theta & = \dfrac{\pi}{6}\ \text{and}\ \dfrac{5\pi}{6}\\
\theta & = \dfrac{\pi}{4}\\
\\
\text{For}\ k = 1,\ \ \theta & = \dfrac{\pi}{6} + \pi\ \text{and}\ \dfrac{5\pi}{6}
+ \pi\\
& = \dfrac{7\pi}{6}\ \text{and}\ \dfrac{11\pi}{6}\\
\theta & = \dfrac{\pi}{4} + \pi = \dfrac{5\pi}{4}
\end{align*}

$$4 \sin \theta \cos \theta + 2\sin \theta - 2\cos \theta - 1 = 0$$\\
We factor the left-hand side of the equation. \begin{align*}
4 \sin \theta \cos \theta + 2\sin \theta - 2\cos \theta - 1 & = 0\\
2 \sin \theta (2\cos \theta + 1) - (2\cos \theta + 1) & = 0\\
(2\sin \theta - 1)(2\cos \theta + 1) & = 0\\
2 \sin \theta - 1 = 0\ \ \text{or}\ \ & 2\cos \theta + 1 = 0\\
\sin \theta = \dfrac{1}{2}\ \ \text{or}\ \ & \cos \theta = -\dfrac{1}{2}\\
\theta = \sin^{-1} \bigg( \dfrac{1}{2}\bigg)\ \ \text{or}\ \ & \theta = \cos^{-
1} \bigg(-\dfrac{1}{2}\bigg)\\
\end{align*}
Because sine and cosine have period $2\pi$, we first find the solutions in any
interval of length $2\pi$. In the interval $(0,2\pi)$ the solutions for the first
equation are $\theta = \dfrac{\pi}{6}$ and $\dfrac{5\pi}{6}$ and the solution for
the second equation is $\theta = \dfrac{2\pi}{3}$ and $\dfrac{4\pi}{3}$\\
\\
\\
To get all solutions, we add integer multiples of $2\pi$ to these solutions : $
$\theta = \dfrac{\pi}{6} + 2k\pi\ \text{and}\ \ \dfrac{5\pi}{6} + 2k\pi\ \ \
\text{or}\ \ \ \theta = \dfrac{2\pi}{3} + 2k\pi\ \text{and}\ \dfrac{4\pi}{3} +
2k\pi$$
where $k$ is any integer.\\
\\
All the solutions in the interval $[0,2\pi)$ will be \begin{align*}
\text{For}\ k = 0,\ \ \theta & = \dfrac{\pi}{6}\ \text{and}\ \dfrac{5\pi}{6}\\
\theta & = \dfrac{2\pi}{3}\ \text{and}\ \dfrac{4\pi}{3}\\
\end{align*}

$$2 \sin \theta \tan \theta - \tan \theta = 1 - 2\sin \theta$$\\


We factor the left-hand side of the equation. \begin{align*}
2 \sin \theta \tan \theta - \tan \theta & = 1 - 2\sin \theta\\
2 \sin \theta \tan \theta - \tan \theta - 1 + 2\sin \theta & = 0\\
2 \sin \theta(\tan \theta + 1) - (\tan \theta + 1) & = 0\\
(2\sin \theta - 1)(\tan \theta + 1) & = 0\\
2 \sin \theta - 1 = 0\ \ \text{or}\ \ & \tan \theta + 1 = 0\\
\sin \theta = \dfrac{1}{2}\ \ \text{or}\ \ & \tan \theta = -1\\
\theta = \sin^{-1} \bigg( \dfrac{1}{2}\bigg)\ \ \text{or}\ \ & \theta = \tan^{-1}
(-1)\\
\end{align*}
Because sine has period $2\pi$, we first find the solutions in any interval of
length $2\pi$. In the interval $(0,2\pi)$ the solutions for the first equation are
$\theta = \dfrac{\pi}{6}$ and $\dfrac{5\pi}{6}$.\\
\\
Because tangent has period $\pi$, we find the solutions in any interval of length
$\pi$. In the interval $(0,\pi)$ the solutions for the second equation are $\theta
= \dfrac{3\pi}{4}$.\\
\\
To get all solutions, we add integer multiples of $2\pi$ and $\pi$ to the first and
second equation solutions respectively: $$\theta = \dfrac{\pi}{6} + 2k\pi\
\text{and}\ \ \dfrac{5\pi}{6} + 2k\pi\ \ \ \text{or}\ \ \ \theta = \dfrac{3\pi}{4}
+ k\pi$$
where $k$ is any integer.\\
\\
\\
All the solutions in the interval $[0,2\pi)$ will be \begin{align*}
\text{For}\ k = 0,\ \ \theta & = \dfrac{\pi}{6}\ \text{and}\ \dfrac{5\pi}{6}\\
\theta & = \dfrac{3\pi}{4}\\
\\
\text{For}\ k = 1,\ \ \theta & = \dfrac{3\pi}{4} + \pi = \dfrac{7\pi}{4}
\end{align*}

$$\sec \theta \tan \theta - \cos \theta \cot \theta = \sin \theta$$\\
We factor the left-hand side of the equation. \begin{align*}
\sec \theta \tan \theta - \cos \theta \cot \theta & = \sin \theta\\
\dfrac{\tan \theta}{\cos \theta} - \dfrac{\cos \theta}{\tan \theta} & = \sin
\theta\\
\dfrac{\tan^2 \theta - \cos^2 \theta}{\cos \theta \tan \theta} & = \sin \theta\\
\dfrac{\tan^2 \theta - \cos^2 \theta}{\cos \theta \bigg(\dfrac{\sin \theta}{\cos
\theta}\bigg)} & = \sin \theta\\
\dfrac{\tan^2 \theta - \cos^2 \theta}{\sin \theta} & = \sin \theta\\
\tan^2 \theta - \cos^2 \theta & = \sin^2 \theta\\
\tan^2 \theta & = \sin^2 \theta + \cos^2 \theta = 1\\
\tan \theta & = \pm 1\\
\theta & = \tan^{-1} (\pm 1)\\
\end{align*}
Because tangent has period $\pi$, we find the solutions in any interval of length
$\pi$. In the interval $(0,\pi)$ the solutions for the equation are $\theta =
\dfrac{\pi}{4}$ or $\theta = \dfrac{3\pi}{4}$.\\
\\
\\
To get all solutions, we add integer multiples of $\pi$ to the equation solutions:
$$\theta = \dfrac{\pi}{4} + k\pi\ \text{and}\ \ \dfrac{3\pi}{4} + k\pi$$
where $k$ is any integer.\\
\\
All the solutions in the interval $[0,2\pi)$ will be \begin{align*}
\text{For}\ k = 0,\ \ \theta & = \dfrac{\pi}{4}\ \text{and}\ \dfrac{3\pi}{4}\\
\\
\text{For}\ k = 1,\ \ \theta & = \dfrac{\pi}{4} + \pi = \dfrac{5\pi}{4}\\
\theta & = \dfrac{3\pi}{4} + \pi = \dfrac{7\pi}{4}
\end{align*}

$$\tan \theta + \cot \theta = 4\sin 2\theta$$\\


We simplify both the sides of the equation. \begin{align*}
\tan \theta + \cot \theta = 4\sin 2\theta\\
\dfrac{\sin \theta}{\cos \theta} + \dfrac{\cos \theta}{\sin \theta} & = 4\sin
2\theta\\
\dfrac{\sin^2 \theta + \cos^2 \theta}{\sin \theta \cos \theta} & = 4\sin 2\theta \\
\dfrac{2}{2\sin \theta \cos \theta} & = 4\sin 2\theta \\
\dfrac{1}{\sin 2\theta} & = 2\sin 2\theta \\
\dfrac{1}{2} & = \sin^2 2\theta \\
\sin 2\theta & = \pm \dfrac{1}{\sqrt{2}}\\
2\theta & = \sin^{-1} \bigg(\pm \dfrac{1}{\sqrt{2}}\bigg)\\
\end{align*}
Because sine has period $2\pi$, we find the solutions in any interval of length
$2\pi$. In the interval $[0,2\pi)$ the solutions for the equation are
\begin{align*}
2\theta &= \dfrac{\pi}{4},\ \dfrac{3\pi}{4},\ \dfrac{5\pi}{4},\ \dfrac{7\pi}{4}
\end{align*}
To get all solutions, we add integer multiples of $2\pi$ to these solutions : $
$2\theta = \dfrac{\pi}{4} + 2k\pi,\ \dfrac{3\pi}{4} + 2k\pi,\ \dfrac{5\pi}{4} +
2k\pi,\ \dfrac{7\pi}{4} + 2k\pi$$
$$\theta = \dfrac{\pi}{8} + k\pi,\ \dfrac{3\pi}{8} + k\pi,\ \dfrac{5\pi}{8} +
k\pi,\ \dfrac{7\pi}{8} + k\pi$$
where $k$ is any integer.\\
\\
Therefore, all the solutions in the interval $[0,2\pi)$ are \begin{align*}
\text{For}\ k = 0,\ \ \theta & = \dfrac{\pi}{8},\ \dfrac{3\pi}{8},\ \dfrac{5\pi}
{8},\ \dfrac{7\pi}{8}\\
\text{For}\ k = 1,\ \ \theta & = \dfrac{\pi}{8} + \pi,\ \dfrac{3\pi}{8} + \pi,\
\dfrac{5\pi}{8} + \pi,\ \dfrac{7\pi}{8} + \pi\\
& = \dfrac{9\pi}{8},\ \dfrac{11\pi}{8},\ \dfrac{13\pi}{8},\ \dfrac{15\pi}{8}
\end{align*}

$$\cos \theta - \sin \theta = \sqrt{2}\sin \dfrac{\theta}{2}$$\\


We start by simplifying RHS of the equation by using Half-Angle Formula
\begin{align*}
\cos \theta - \sin \theta & = \sqrt{2} \sqrt{\dfrac{1 - \cos \theta}{2}}\\
\cos \theta - \sin \theta & = \sqrt{1 - \cos \theta}\\
(\cos \theta - \sin \theta)^2 & = 1 - \cos \theta\\
\cos^2 \theta + \sin^2 \theta - 2\sin \theta \cos \theta & = 1 - \cos \theta\\
1 - 2\sin \theta \cos \theta & = 1 - \cos \theta\\
\cos \theta ( 1 - 2\sin \theta) & =0\\
\cos \theta = 0\ \ \ \ \ & \text{or}\ \ \ \ \sin \theta = \dfrac{1}{2}\\
\end{align*}
Because sine and cosine have period $2\pi$, we find the solutions in any interval
of length $2\pi$. In the interval $[0,2\pi)$ the solutions for the first equation
are $\theta = \dfrac{\pi}{2}$ and $\dfrac{3\pi}{2}$ and the solution for the second
equation are $\theta = \dfrac{\pi}{6}$ and $\dfrac{5\pi}{6}$\\
\\
\\
The solutions $\theta = \dfrac{\pi}{2}$ and $\dfrac{5\pi}{6}$ does not satisfy the
equation and therefore, all the solutions in the interval $[0,2\pi)$ will be $
$\theta = \dfrac{\pi}{6}\ \text{and}\ \dfrac{3\pi}{2}$$

$$\sin \theta - \cos \theta = \dfrac{1}{2}$$\\


We start by dividing both sides of the equation by $\sqrt{2}$ \begin{align*}
\dfrac{1}{\sqrt{2}} \sin \theta - \dfrac{1}{\sqrt{2}} \cos \theta & = \dfrac{1}
{2\sqrt{2}}\\
\cos \dfrac{\pi}{4} \sin \theta - \sin \dfrac{\pi}{4} \cos \theta & = \dfrac{1}
{2\sqrt{2}}\\
\cos \bigg(\dfrac{\pi}{4} + \theta \bigg) & = \dfrac{1}{2\sqrt{2}}\\
\pm \bigg(\dfrac{\pi}{4} + \theta\bigg) & = \cos^{-1} \bigg(\dfrac{1}
{2\sqrt{2}}\bigg)\\
\theta = \cos^{-1} \bigg(\dfrac{1}{2\sqrt{2}}\bigg) - \dfrac{\pi}{4}\ \ &
\text{or}\ \ \ \theta = -\cos^{-1} \bigg(\dfrac{1}{2\sqrt{2}}\bigg) - \dfrac{\pi}
{4}\\
\end{align*}
Cosine has period $2\pi$. To get all solutions, we add integer multiples of $2\pi$
to this solution : $$\theta = \cos^{-1} \bigg(\dfrac{1}{2\sqrt{2}}\bigg) -
\dfrac{\pi}{4} + 2k\pi$$ or $$\theta = -\cos^{-1} \bigg(\dfrac{1}{2\sqrt{2}}\bigg)
- \dfrac{\pi}{4} + 2k\pi$$
where $k$ is any integer.\\
\\
\\
Therefore, all the solutions in the interval $[0,2\pi)$ are \begin{align*}
\text{For}\ k = 0,\ \ \theta & = \cos^{-1} \bigg(\dfrac{1}{2\sqrt{2}}\bigg) -
\dfrac{\pi}{4}\\
\text{For}\ k = 1,\ \ \theta & = -\cos^{-1} \bigg(\dfrac{1}{2\sqrt{2}}\bigg) -
\dfrac{\pi}{4} + 2\pi\\
& = -\cos^{-1} \bigg(\dfrac{1}{2\sqrt{2}}\bigg) + \dfrac{7\pi}{4}
\end{align*}

$$\sin 2x = x$$\\
To find the solutions of the given equation, we draw the graphs of $y = x$ and $y =
\sin 2x$ on the same graph and look for the points of intersection. They will be
the solutions of the equation.\\
\\
The two graphs are shown in the figure below. From the graph, we see that the
solutions of the given equation are $$x = -0.95,\ 0,\ 0.95$$

$$2^{\sin x} = x$$\\
To find the solutions of the given equation, we draw the graphs of $y = x$ and $y =
2^{\sin x}$ on the same graph and look for the points of intersection. They will be
the solutions of the equation.\\
\\
The two graphs are shown in the figure below. From the graph, we see that the
solution of the given equation is $$x = 1.92$$

$$\tan^{-1} x + \tan^{-1} 2x = \dfrac{\pi}{4}$$\\


Let $u = \tan^{-1} x$ and $v = \tan^{-1} 2x$. Then $$x = \tan u\ \text{and}\ 2x
= \tan v$$\\
Now the equation becomes $$u + v = \dfrac{\pi}{4}$$
Taking tangent on each side, we get \begin{align*}
\tan ( u + v) & = \tan \dfrac{\pi}{4}\\
\dfrac{\tan u + \tan v}{1 - \tan u \tan v} & = 1\\
\dfrac{x + 2x}{1 - x (2x)} & = 1\\
3x & = 1 - 2x^2\\
2x^2 + 3x - 1 & = 0
\end{align*}\\
Solving using quadratic formula, we get \begin{align*}
x & = \dfrac{-3 \pm \sqrt{(3)^2 - 4(2)(-1)}}{2(2)}\\
& = \dfrac{-3 \pm \sqrt{17}}{4}\\
\end{align*}
Now, taking $x$ negative will give LHS negative and so the equation becomes
invalid. Therefore, $$x = \dfrac{-3 + \sqrt{17}}{4}$$

$$2\sin^{-1} x + \cos^{-1} x = \pi$$\\


Let $u = \sin^{-1} x$. Then $x = \sin u$. Rewriting equation as $$2\sin^{-1} x =
\pi - \cos^{-1} x$$
Taking cosine on each side, we get \begin{align*}
\cos(2\sin^{-1} x) = \cos(\pi - \cos^{-1} x)\\
\cos 2u = -\cos (\cos^{-1} x)\\
1 - 2\sin^2 u = -x\\
1 - 2x^2 = -x\\
2x^2 - x - 1 & = 0
\end{align*}\\
Solving using quadratic formula, we get \begin{align*}
x & = \dfrac{1 \pm \sqrt{(-1)^2 - 4(2)(-1)}}{2(2)}\\
& = \dfrac{1 \pm \sqrt{9}}{4}\\
& = \dfrac{1 \pm 3}{4}\\
& = \dfrac{-1}{2}\ \text{or}\ 1\\
\end{align*}
Now, taking $x = -\dfrac{1}{2}$ does not satisfy the equation. Therefore, $$x = 1$$

It is given that if a projectile is fired with a velocity $v_0$ at angle $\theta$,


then its range, the horizontal distance it travels (in ft), is modeled by the
function $$R(\theta) = \dfrac{v_0^2 \sin 2\theta}{32}$$\\
When the target on the ground $5000$ ft away is needed to be hit using $v_0 = 2200$
ft/s, then the function becomes \begin{align*}
R(\theta) & = \dfrac{v_0^2 \sin 2\theta}{32}\\
5000 & = \dfrac{(2200)^2 \sin 2\theta}{32}\\
160000 & = 4840000 \sin 2\theta\\
\sin 2 \theta & = \dfrac{4}{121}\\
2\theta & = \sin^{-1}\bigg(\dfrac{4}{121}\bigg)\\
\end{align*}
Using calculator, we get \begin{align*}
2 \theta & \approx 1.89^o\ \text{or}\ 178^o\\
\text{Thus}\ \theta & \approx 0.95\ \text{or}\ 89^o
\end{align*}
Therefore, the projectile should be fired at an angle of $0.95^o$ or $89^o$
approximately.
Given that the displacement of a spring vibrating in damped harmonic motion is
given by $$y = 4e^{-3t} \sin 2\pi t$$\\
For the spring to be at its equilibrium position, i.e. $y = 0$, the equation
becomes \begin{align*}
4e^{-3t} \sin 2\pi t & = 0\\
\sin 2\pi t & = 0\ \ \ \ \ \ \ \ \ \ \ \ \ \ \ \ [\ \text{Exponential functions are
always positive}\ ]\\
2 \pi t & = \sin^{-1} (0)\\
2 \pi t & = n\pi \ \ \ \ \ \ \ \ \ \ \ \ \ \ \ [\ n\ \text{be any integer}\ ]\\
t & = \dfrac{n}{2}
\end{align*}\\
Therefore, the spring is at its equilibrium position at times $$t= 0,\ 0.5,\ 1,\
1.5,\ 2,\ .\ .\ .\ .\ .\ .\ .\ $$

Given that in Philadelphia the number of hours of daylight on day $t$ (where $t$ is
the number of days after January 1) is modeled by the function $$L(t) = 12 +
2.83\sin \bigg(\dfrac{2\pi}{365}(t - 80)\bigg) $$\\
(a)\ When there are 10 hours of daylight, the function becomes \begin{align*}
12 + 2.83\sin \bigg(\dfrac{2\pi}{365}(t - 80)\bigg) & = 10\\
2.83\sin \bigg(\dfrac{2\pi}{365}(t - 80)\bigg) & = -2\\
\sin \bigg(\dfrac{2\pi}{365}(t - 80)\bigg) & = -0.7067\\
\dfrac{2\pi}{365}(t - 80) & = \sin^{-1}(-0.7067)\\
\dfrac{2\pi}{365}(t - 80) & = \frac{5\pi}{4} \text{or}\ -\frac{\pi}{4}\\
t - 80 & = \frac{1825}{8}\ \text{or}\ -\frac{365}{8}\\
& = 228.125\ \text{or}\ -45.625\\
t & = 308.125\ \text{or}\ 34.375
\end{align*}\\
Therefore, 34th day (February 3) and 308th day (November 4) approximately will have
10 hours of daylight.

(b)\ When there are more than 10 hours of daylight, the function becomes
\begin{align*}
12 + 2.83\sin \bigg(\dfrac{2\pi}{365}(t - 80)\bigg) & > 10\\
2.83\sin \bigg(\dfrac{2\pi}{365}(t - 80)\bigg) & > -2\\
\sin \bigg(\dfrac{2\pi}{365}(t - 80)\bigg) & > -0.7067\\
\end{align*}
Now, using the information from part (a), we see that the daylight is more than 10
hours between days $t = 34.375$ and $t = 308.125$. Therefore, the number of days
with more than 10 hours of daylight are $$t = 308.125 - 34.375 \approx 274\
\text{days} $$

Given figure is labelled and redrawn as shown below. The belt of length $L$
surrounds two pulleys of radii $R$ and $r$ with angle $\theta$ where the belt
crosses itself.\\
\\
\\
(a)\ Using angle sum property, we can say that $$\angle PAN = \angle OBQ = \pi -
\theta$$
Now using the formula for arc length, we have $$\widehat{PN} = R(\pi -
\theta)\ \ \text{and}\ \ \widehat{OQ} = r(\pi - \theta)$$\\
Now, length of belt on the pulleys will be the difference between circumferences of
pulleys and the arc length. So, $$\widehat{PLN} = 2\pi R - R(\pi - \theta) = R(\pi
+ \theta)$$
Similarly, $$\widehat{OMQ} = 2\pi r - r(\pi - \theta) = r(\pi + \theta)$$\\
Now, in $\Delta$APT, we have $$\cot \dfrac{\theta}{2} = \dfrac{PT}{AP} \Rightarrow
PT = R \cot \dfrac{\theta}{2}$$
Similarly, in $\Delta$BOT, we have $$\cot \dfrac{\theta}{2} = \dfrac{OT}{OB}
\Rightarrow OT = r \cot \dfrac{\theta}{2}$$

We also have $$PT = NT\ \text{and}\ OT = QT$$ Since, total length of belt is $L$,
we have \begin{align*}
\widehat{PLN} + PT + NT + OT + QT + \widehat{OMQ} & = L\\
R( \pi + \theta) + R\cot \frac{\theta}{2} + R\cot \frac{\theta}{2} + r\cot
\frac{\theta}{2} + r\cot \frac{\theta}{2} + r(\pi + \theta) & = L\\
(\pi + \theta) (R + r) + 2R\cot \frac{\theta}{2} + 2r\cot \frac{\theta}{2} & = L\\
(\pi + \theta)(R + r) + 2\cot \frac{\theta}{2}(R + r) & = L\\
\bigg(\pi + \theta + 2\cot \frac{\theta}{2} \bigg)(R + r) & = L\\
\pi + \theta + 2\cot \frac{\theta}{2} & = \dfrac{L}{R + r}\\
\theta + 2\cot \frac{\theta}{2} & = \dfrac{L}{R + r} - \pi
\end{align*}

(b)\ From the above part, we got the equation $$\theta + 2\cot \dfrac{\theta}{2}
= \dfrac{L}{R+r} - \pi$$
For $R = 2.42$ ft, $r = 1.21$ ft and $L = 27.78$ ft, the equation becomes
\begin{align*}
\theta + 2\cot \dfrac{\theta}{2} & = \dfrac{L}{R+r} - \pi\\
\theta + 2\cot \dfrac{\theta}{2} & = \dfrac{27.78}{2.42+1.21} - \pi\\
\theta + 2\cot \dfrac{\theta}{2} & = 4.511\\
\end{align*}
Let's draw the graphs of $y = \theta + 2 \cot \dfrac{\theta}{2}$ and $y = 4.511$ on
the same window as shown below. From the below graph, we see that the solution of
the equation is $\theta = 1.05$ radians.\\
\\
To convert radians into degrees, we multiply it by $\dfrac{\pi}{180^o} $
$$1.05 \times \dfrac{180^o}{\pi} = 60.16^o$$\\

The equation $\sin (\cos x ) = 0$ is different from all the other equations we have
looked at in this section since the function sine is itself a function of a
variable $\cos x$ here.\\
\\
Upon solving the equation, we get \begin{align*}
\sin (\cos x) & = 0\\
\cos x & = n\pi
\end{align*}
where $n$ can be any integer.\\
\\
Since $-1 \leq \cos x \leq 1$, the only possibility of $\cos x$ is when $n = 0$,
i.e. \begin{align*}
\cos x & = 0\\
x & \cos^{-1} (0)\\
& = \dfrac{\pi}{2} + k\pi
\end{align*}
where $k$ is any integer.

$$\dfrac{1 + \sec x}{\sec x} = \dfrac{\sin^2 x}{1 - \cos x}$$


\begin{align*}
\text{LHS}\ & = \dfrac{1 + \sec x}{\sec x}\\
& = \dfrac{1 + \dfrac{1}{\cos x}}{\dfrac{1}{\cos x}}\ \ \ \ \ \ \ \ \ \ \ \ \ \
[\ \text{Reciprocal Identities}\ ]\\
& = \dfrac{\dfrac{1 + \cos x}{\cos x}}{\dfrac{1}{\cos x}}\\
& = 1 + \cos x \end{align*}
Upon multiplying both numerator and denominator by $1 - \cos x$, we get
\begin{align*} \text{LHS} & = 1 + \cos x \times \dfrac{1 - \cos x}{1 - \cos x}\\
& = \dfrac{1 - \cos^2 x}{1 - \cos x}\\
& = \dfrac{\sin^2 x}{1 - \cos x} \ \ \ \ \ \ \ \ \ \ \ \ \ \ \ \ [\
\text{Pythagorean Identity}\ ]\\
& = \text{RHS}
\end{align*}

$$\dfrac{\cos^2 x}{1 - \sin x} = \dfrac{\cos x}{\sec x - \tan x}$$


\begin{align*}
\text{LHS}\ & = \dfrac{\cos^2 x}{1 - \sin x}\\
& = \dfrac{\dfrac{\cos^2 x}{\cos x}}{\dfrac{1 - \sin x}{\cos x}}\\
& = \dfrac{\cos x}{\dfrac{1}{\cos x} - \dfrac{\sin x}{\cos x}}\\
& = \dfrac{\cos x}{\sec x - \tan x}\ \ \ \ \ \ \ \ \ \ \ \ \ \ \ \ \ [\
\text{Reciprocal Identities}\ ]\\
& = \text{RHS}
\end{align*}

$$\dfrac{\cos 3x - \cos 7x}{\sin 3x + \sin 7x} = \tan 2x$$\\


We can simplify LHS as \begin{align*}
\text{LHS} & = \dfrac{\cos 3x - \cos 7x}{\sin 3x + \sin 7x}\\
& = \dfrac{-2\sin \bigg(\dfrac{3x+7x}{2}\bigg) \sin \bigg(\dfrac{3x-7x}{2}\bigg)}
{2\sin \bigg(\dfrac{3x+7x}{2}\bigg) \cos \bigg(\dfrac{3x-7x}{2}\bigg)}\ \ \ \ \ \ \
\ \ [\ \text{Sum-To-Product Formula}\ ]\\
& = \dfrac{-\sin \bigg(\dfrac{-4x}{2}\bigg)}{\cos \bigg(\dfrac{-4x}{2}\bigg)}\\
& = \dfrac{-\sin (-2x)}{\cos (-2x)}\\
& = \dfrac{\sin 2x}{\cos 2x}\ \ \ \ \ \ \ \ \ \ \ \ \ \ \ \ \ \ \ \ \ \ \ \ \ \ \ \
\ \ \ \ \ \ \ \ \ \ \ \ \ \ \ \ \ [\ \text{Even-Odd Identity}\ ]\\
& = \tan 2x \ \ \ \ \ \ \ \ \ \ \ \ \ \ \ \ \ \ \ \ \ \ \ \ \ \ \ \ \ \ \ \ \ \ \ \
\ \ \ \ \ \ \ \ \ \ [\ \text{Reciprocal Identity}\ ]\\
& = \text{RHS}
\end{align*}

$$\dfrac{\sin (x+y) + \sin (x - y)}{\cos (x+y) + \cos (x - y)} = \tan x$$\\


We can simplify LHS as \begin{align*}
\text{LHS} & = \dfrac{\sin (x+y) + \sin (x - y)}{\cos (x+y) + \cos (x - y)}\\
& = \dfrac{2 \sin \Bigg(\dfrac{(x+y)+(x-y)}{2}\Bigg) \cos \Bigg(\dfrac{(x+y)-(x-y)}
{2}\Bigg)}{2\cos \Bigg(\dfrac{(x+y)+(x-y)}{2}\Bigg) \cos \Bigg(\dfrac{(x+y)-(x-y)}
{2}\Bigg)}\ \ \ \ \ \ [\ \text{Sum-To-Product Formula}\ ]\\
& = \dfrac{\sin \Bigg(\dfrac{2x}{2}\Bigg)}{\cos \Bigg(\dfrac{2x}{2}\Bigg)}\\
& = \dfrac{\sin x}{\cos x}\\
& = \tan
x \ \ \ \ \ \ \ \ \ \ \ \ \ \ \ \ \ \ \ \ \ \ \ \ \ \ \ \ \ \ \ \ \ \ \ \ \ \ \ \ \
\ \ \ \ \ \ \ \ \ \ \ \ \ \ \ \ \ \ \ \ \ \ \ \ \ \ [\ \text{Reciprocal
Identity}\ ]\\
& = \text{LHS}
\end{align*}
$$\sin (x+y) \sin (x-y) = \sin^2 x - \sin^2 y$$\\
The fourth Product-To-Sum Formula gives $$\sin u \sin v = \dfrac{1}{2}[\cos (u - v)
- \cos (u + v)]$$
\begin{align*}
\text{LHS}\ & = \sin (x+y) \sin (x-y)\\
& = \dfrac{1}{2}[\cos \big((x+y) -(x-y) \big) - \cos \big((x+y) + (x-y) \big)]\\
& = \dfrac{1}{2}[\cos 2y - \cos 2x]\\
& = \dfrac{1}{2}[(1 - 2\sin^2 y) - (1 - 2\sin^2 x)]\ \ \ \ \ \ \ \ \ \ \ \ [\
\text{Double-Angle Formula}\ ]\\
& = \dfrac{1}{2}[1 - 2\sin^2 y - 1 + 2\sin^2 x]\\
& = \dfrac{1}{2}[2(\sin^2 x- \sin^2 y)]\\
& = \sin^2 x - \sin^2 y\\
& = \text{RHS}
\end{align*}

(b)\ From previous exercise, we see that both the graphs are exactly the same which
suggests that the equation $f(x) = g(x)$ is an identity.\\
\\
We can prove this algebraically. Let's simplify $f(x)$.
\begin{align*}
f(x) & = 1 - \bigg( \cos \dfrac{x}{2} - \sin \dfrac{x}{2} \bigg)^2\\
& = 1 - \bigg( \cos^2 \dfrac{x}{2} + \sin^2 \dfrac{x}{2} - 2 \sin \dfrac{x}{2} \cos
\dfrac{x}{2} \bigg)\\
& = 1 - \bigg(1 - 2 \sin \dfrac{x}{2} \cos \dfrac{x}{2} \bigg)\ \ \ \ \ \ \ \ \ \ \
\ \ \ \ \ [\ \text{Pythagorean Identity}\ ]\\
& = 1 - (1 - \sin x )\ \ \ \ \ \ \ \ \ \ \ \ \ \ \ \ \ \ \ \ \ \ \ \ \ \ \ \ [\
\text{Half-Angle Formula}\ ]\\
& = \sin x\\
& = g(x)
\end{align*}
Hence Proved

(b)\ From previous exercise, we see that both the graphs are different which
suggests that the equation $f(x) = g(x)$ is not an identity.\\
\\
We can prove this algebraically. Let's simplify $g(x)$.
\begin{align*}
g(x) & = \sqrt{\sin^2 x + \cos^2 x}\\
& = \sqrt{1}\ \ \ \ \ \ \ \ \ \ \ \ \ \ \ \ \ \ \ \ \ \ [\ \text{Pythagorean
Identity}\ ]\\
& = 1\\
& \ne f(x)
\end{align*}\\
Since $f(x) \ne 1$ for all $x$, we see that $f(x) \ne g(x)$. Therefore, the
equation $f(x) = g(x)$ is not an identity.

(b)\ From previous exercise, we see that both the graphs though are similar, but
are not exactly the same which suggests that the equation $f(x) = g(x)$ is not an
identity.\\
\\
We can prove this algebraically. Let's simplify $f(x)$.
\begin{align*}
f(x) & = \tan x \tan \dfrac{x}{2}\\
& = \dfrac{\sin x}{\cos x} \cdot \tan \dfrac{x}{2}\ \ \ \ \ \ \ \ \ \ \ \ \ \ \ \ \
\ \ \ \ \ [\ \text{Reciprocal Identity}\ ]\\
& = \dfrac{\sin x}{\cos x} \cdot \dfrac{1 - \cos x}{\sin x}\\
& = \dfrac{1 - \cos x}{\cos x}\\
& \ne g(x)
\end{align*}\\
Since $f(x) \ne g(x)$, therefore, the equation $f(x) = g(x)$ is not an identity.

(b)\ From previous exercise, we see that both the graphs are exactly the same which
suggests that the equation $f(x) = g(x)$ is an identity.\\
\\
We can prove this algebraically. Let's simplify $f(x)$.
\begin{align*}
f(x) & = 1 - 8\sin^2 x + 8 \sin^4 x\\
& = 1 - 8\sin^2 x(1 - \sin^2 x)\\
& = 1 - 8\sin^2 x \cos^2 x\ \ \ \ \ \ \ \ \ \ \ \ \ \ \ \ \ \ \ \ \ \ [\
\text{Pythagorean Identity}\ ]\\
& = 1 - 2(2\sin x \cos x)^2\\
& = 1 - 2(\sin 2x)^2\ \ \ \ \ \ \ \ \ \ \ \ \ \ \ \ \ \ \ \ \ \ \ \ \ \ \ [\
\text{Double-Angle Formula}\ ]\\
& = 1 - 2\sin^2 2x\\
& = \cos 2(2x)\ \ \ \ \ \ \ \ \ \ \ \ \ \ \ \ \ \ \ \ \ \ \ \ \ \ \ \ \ \ \ \ \
[\ \text{Double-Angle Formula}\ ]\\
& = \cos 4x\\
& = g(x)
\end{align*}
Hence Proved

(b)\ From previous exercise, we made a conjecture that $$f(x) = 2 \sin^2 3x + \cos
6x = 1$$
We prove this conjecture by expanding $f(x)$. \begin{align*}
f(x) & = 2\sin^2 3x + \cos 6x\\
& = 2\sin^2 3x + \cos 2(3x)\\
& = 2\sin^2 3x + 1 - 2 \sin^2 3x\ \ \ \ \ \ \ \ \ \ \ \ \ \ \ \ \ [\ \text{Double-
Angle Formula}\ ]\\
& = 1
\end{align*}
Hence Proved\\

(b)\ From previous exercise, we made a conjecture that $$f(x) = g(x) + 1$$
We prove this conjecture by simplifying $f(x)$. \begin{align*}
f(x) & = \sin x \cot \dfrac{x}{2}\\
& = 2\sin \dfrac{x}{2} \cos \dfrac{x}{2} \cdot \cot \dfrac{x}{2}\ \ \ \ \ \ \ \ \ \
\ \ \ \ \ \ \ [\ \text{Half-Angle Fomrula}\ ]\\
& = 2\sin \dfrac{x}{2} \cos \dfrac{x}{2} \cdot \dfrac{\cos \dfrac{x}{2}}{\sin
\dfrac{x}{2}}\ \ \ \ \ \ \ \ \ \ \ \ \ \ \ \ [\ \text{Reciprocal Identity}\ ]\\
& = 2 \cos^2 \dfrac{x}{2}\\
& = 2 \cos^2 \dfrac{x}{2} - 1 + 1\\
& = \cos x + 1 \ \ \ \ \ \ \ \ \ \ \ \ \ \ \ \ \ \ \ \ \ \ \ \ \ \ \ \ \ \ [\
\text{Half-Angle Formula}\ ]\\
& = g(x) + 1
\end{align*}
Hence Proved\\

$$\cos x \sin x - \sin x = 0$$\\


We factor the left-hand side of the equation. \begin{align*}
\cos x \sin x - \sin x & = 0\\
\sin x(\cos x - 1 ) & = 0\\
\sin x = 0\ \text{or}\ \cos x & = 1\\
x = \sin^{-1} (0)\ \text{or}\ x & = \cos^{-1} (1)\\
\end{align*}
In the interval $[0,2\pi)$ the solutions for the first equation are $x = 0$ and $x
= \pi$ and the solution for the second equation is $x = 0$\\
\\
Therefore, all the solutions of the equation in the interval $[0,2\pi)$ are $$x =
0\ \text{and}\ \pi$$

$$\sin x - 2\sin^2 x = 0$$\\


We factor the left-hand side of the equation. \begin{align*}
\sin x - 2\sin^2 x & = 0\\
\sin x(1 - 2\sin x ) & = 0\\
\sin x = 0\ \text{or}\ \sin x & = \dfrac{1}{2}\\
x = \sin^{-1} (0)\ \text{or}\ x & = \sin^{-1} \bigg(\dfrac{1}{2}\bigg)\\
\end{align*}
In the interval $[0,2\pi)$ the solutions for the first equation are $x = 0$ and $x
= \pi$ and the solutions for the second equation are $x = \dfrac{\pi}{6}$ and $x
= \dfrac{5\pi}{6}$.\\
\\
Therefore, all the solutions of the equation in the interval $[0,2\pi)$ are $$x =
0,\ \dfrac{\pi}{6},\ \dfrac{5\pi}{6},\ \pi$$

$$2\sin^2 x - 5\sin x + 2 = 0$$\\


We factor the left-hand side of the equation. \begin{align*}
2\sin^2 x - 5\sin x + 2 & = 0\\
2\sin^2 x - 4\sin x - \sin x + 2 & = 0\\
2\sin x(\sin x - 2) - (\sin x - 2) & = 0\\
(\sin x - 2) (2\sin x-1) & = 0\\
\sin x = 2\ \text{or}\ \sin x & = \dfrac{1}{2}\\
x = \sin^{-1} (2)\ \text{or}\ x & = \sin^{-1} \bigg(\dfrac{1}{2}\bigg)\\
\end{align*}
Since sine function lies between $-1$ and $1$, there are no solutions for the first
equation $\sin x = 2$. In the interval $[0,2\pi)$ the solutions for the second
equation are $x = \dfrac{\pi}{6}$ and $x = \dfrac{5\pi}{6}$.\\
\\
Therefore, all the solutions of the equation in the interval $[0,2\pi)$ are $$x
= \dfrac{\pi}{6},\ \dfrac{5\pi}{6}$$

$$\sin x - \cos x - \tan x = -1$$\\


We factor the left-hand side of the equation. \begin{align*}
\sin x - \cos x - \tan x & = -1\\
\sin x - \cos x - \dfrac{\sin x}{\cos x} & = -1\\
\dfrac{\sin x \cos x - \cos^2 x - \sin x}{\cos x} & = -1\\
\sin x \cos x - \cos^2 x - \sin x & = -\cos x\\
\cos x(\sin x - \cos x) - (\sin x - \cos x) & = 0\\
(\sin x - \cos x)(\cos x - 1) & = 0\\
\sin x = \cos x\ \text{or}\ \cos x & = 1\\
\end{align*}
In the interval $[0,2\pi)$ the solutions for the first equation are $x =
\dfrac{\pi}{4}$ and $x = \dfrac{5\pi}{4}$ and the solutions for the second equation
are $x = 0$.\\
\\
Therefore, all the solutions of the equation in the interval $[0,2\pi)$ are $$x =
0,\ \dfrac{\pi}{4},\ \dfrac{5\pi}{4}$$

$$2\cos^2 x - 7\sin x + 3 = 0$$\\


We factor the left-hand side of the equation. \begin{align*}
2\cos^2 x - 7\cos x + 3 & = 0\\
2\cos^2 x - 6\cos x - \cos x + 3 & = 0\\
2\cos x(\cos x - 3) - (\cos x - 3) & = 0\\
(\cos x - 3) (2\cos x-1) & = 0\\
\cos x = 3\ \text{or}\ \cos x & = \dfrac{1}{2}\\
x = \cos^{-1} (3)\ \text{or}\ x & = \cos^{-1} \bigg(\dfrac{1}{2}\bigg)\\
\end{align*}
Since cosine function lies between $-1$ and $1$, there are no solutions for the
first equation $\cos x = 3$. In the interval $[0,2\pi)$ the solutions for the
second equation are $x = \dfrac{\pi}{3}$ and $x = \dfrac{5\pi}{3}$.\\
\\
Therefore, all the solutions of the equation in the interval $[0,2\pi)$ are $$x
= \dfrac{\pi}{3},\ \dfrac{5\pi}{3}$$

$$4\sin^2 x + 2\cos^2 x = 3$$\\


We factor the left-hand side of the equation. \begin{align*}
4\sin^2 x + 2\cos^2 x & = 3\\
4\sin^2 x + 2(1 - \sin^2 x) -3 & = 0\\
4\sin^2 x + 2 - 2\sin^2 x -3 & = 0\\
2\sin^2 x - 1 & = 0\\
\sin^2 x & = \dfrac{1}{2}\\
\sin x & = \pm \dfrac{1}{\sqrt{2}}\\
x & = \sin^{-1} \bigg(\pm \dfrac{1}{\sqrt{2}}\bigg)\\
\end{align*}
In the interval $[0,2\pi)$ the solutions for the above equation are $$x =
\dfrac{\pi}{4},\ \dfrac{3\pi}{4},\ \dfrac{5\pi}{4},\ \dfrac{7\pi}{4}$$

$$\dfrac{1 - \cos x}{1 + \cos x} = 3$$\\


We simplify the equation as \begin{align*}
\dfrac{1 - \cos x}{1 + \cos x} & = 3\\
1 - \cos x & = 3 + 3 \cos x\\
4\cos x & = -2\\
\cos x & = -\dfrac{1}{2}\\
x & = \cos^{-1} \bigg(-\dfrac{1}{2}\bigg)\\
\end{align*}
In the interval $[0,2\pi)$ the solutions for the above equation are $$x =
\dfrac{2\pi}{3},\ \dfrac{4\pi}{3}$$
$$\sin x = \cos 2x$$\\
Using Double-Angle Formula, we simplify the equation as \begin{align*}
\sin x & = \cos 2x\\
\sin x & = 1 - 2\sin^2 x\\
2\sin^2 x + \sin x - 1 & = 0\\
2\sin^2 x + 2\sin x - \sin x - 1 & = 0\\
2\sin x(\sin x + 1) - (\sin x + 1) & = 0\\
(\sin x + 1)(2\sin x - 1) & = 0\\
\sin x = -1\ \text{or}\ \sin x & = \dfrac{1}{2}\\
x = \sin^{-1} (-1)\ \text{or}\ x & = \sin^{-1} \bigg(\dfrac{1}{2}\bigg)\\
\end{align*}
In the interval $[0,2\pi)$ the solution for the first equation is $x = \dfrac{3\pi}
{2}$ and the solutions for the second equation are $x = \dfrac{\pi}{6}$ and $x =
\dfrac{5\pi}{6}$.\\
\\
Therefore, all the solutions of the equation in the interval $[0,2\pi)$ are $$x
= \dfrac{\pi}{6},\ \dfrac{5\pi}{6},\ \dfrac{3\pi}{2}$$

$$\tan^3 x + \tan^2 x - 3\tan x - 3 = 0$$\\


We factor the left-hand side of the equation. \begin{align*}
\tan^3 x + \tan^2 x - 3\tan x - 3 & = 0\\
\tan^2 x(\tan x + 1) - 3 (\tan x + 1) & = 0\\
(\tan^2 x - 3)(\tan + 1) & = 0\\
\tan^2 x - 3 = 0\ \ \text{or}\ \ & \tan x +1 = 0\\
\tan x = \pm \sqrt{3}\ \ \text{or}\ \ & \tan x = -1\\
x = \tan^{-1} ( \pm \sqrt{3})\ \ \text{or}\ \ & x = \tan^{-1} (-1)\\
\end{align*}
In the interval $[0,2\pi)$ the solution for the first equation is $x = \dfrac{\pi}
{3},\ \dfrac{2\pi}{3},\ \dfrac{4\pi}{3},\ \dfrac{5\pi}{3}$ and the solutions for
the second equation are $x = \dfrac{3\pi}{4},\ \dfrac{7\pi}{4}$.\\
\\
Therefore, all the solutions of the equation in the interval $[0,2\pi)$ are $$x
= \dfrac{\pi}{3},\ \dfrac{2\pi}{3},\ \dfrac{3\pi}{4},\ \dfrac{4\pi}{3},\
\dfrac{5\pi}{3},\ \dfrac{7\pi}{4}$$

$$\cos 2x \csc^2 x = 2 \cos 2x$$\\


We simplify the equation as \begin{align*}
\cos 2x \csc^2 x & = 2 \cos 2x\\
\cos 2x \csc^2 x - 2 \cos 2x & = 0\\
\cos 2x(\csc^2 x - 2) & = 0\\
\cos 2x = 0\ \ \text{or}\ \ & \csc^2 x = 2\\
2x = \cos^{-1} (0)\ \ \text{or}\ \ & \csc x = \pm \sqrt{2}\\
2x = \cos^{-1} (0)\ \ \text{or}\ \ & x = \csc^{-1} (\pm \sqrt{2})\\
\end{align*}
The solutions for the first equation are $$2x = \dfrac{\pi}{2},\ \dfrac{3\pi}{2}$$
To get all solutions, we add integer multiples of $2\pi$ to these solutions. So,
the solutions are of the form $$2x = \dfrac{\pi}{2} + 2k\pi\ \ \ \text{and}\ \ \ 2x
= \dfrac{3\pi}{2} + 2k\pi$$
$$x = \dfrac{\pi}{4} + k\pi\ \ \ \text{and}\ \ \ x = \dfrac{3\pi}{4} + k\pi$$
The solutions that are in the interval $[0,2\pi)$ correspond to $k = 0$ and $1$.
For all other values of $k$, the corresponding values of $x$ lie outside this
interval. So, the solutions in the interval $[0,2\pi)$ are $$x = \dfrac{\pi}
{4},\ \dfrac{3\pi}{4},\ \dfrac{5\pi}{4},\ \dfrac{7\pi}{4}$$
In the interval $[0,2\pi)$, the solutions for the second equation are $$x =
\dfrac{\pi}{4},\ \dfrac{3\pi}{4},\ \dfrac{5\pi}{4},\ \dfrac{7\pi}{4}$$\\
\\
Therefore, all the solutions of the equation in the interval $[0,2\pi)$ are $$x
= \dfrac{\pi}{4},\ \dfrac{3\pi}{4},\ \dfrac{5\pi}{4},\ \dfrac{7\pi}{4}$$

$$\tan \dfrac{1}{2}x + 2\sin 2x = \csc x$$\\


We simplify the equation as \begin{align*}
\tan \dfrac{1}{2}x + 2\sin 2x & = \csc x\\
\dfrac{1 - \cos x}{\sin x} + 4\sin x \cos x & = \dfrac{1}{\sin x}\\
\dfrac{1 - \cos x + 4\sin^2 x \cos x}{\sin x} & = \dfrac{1}{\sin x}\\
1 - \cos x + 4\sin^2 x \cos x & = 1\\
4\sin^2 x \cos x - \cos x & = 0\\
\cos x(4\sin^2 x - 1) & = 0\\
\cos x = 0\ \ \text{or}\ \ & 4\sin^2 x = 1\\
x = \cos^{-1} (0)\ \ \text{or}\ \ & \sin x = \pm \dfrac{1}{2}\\
x = \cos^{-1} (0)\ \ \text{or}\ \ & x = \sin^{-1} \bigg(\pm \dfrac{1}{2}\bigg)\\
\end{align*}
In the interval $[0,2\pi)$, the solutions for the first equation are $x =
\dfrac{\pi}{2},\ \dfrac{3\pi}{2}$ and the solutions for the second equation are $$x
= \dfrac{\pi}{6},\ \dfrac{5\pi}{6},\ \dfrac{7\pi}{6},\ \dfrac{11\pi}{6}$$\\
\\
Therefore, all the solutions of the equation in the interval $[0,2\pi)$ are $$x
= \dfrac{\pi}{6},\ \dfrac{\pi}{2},\ \dfrac{5\pi}{6},\ \dfrac{7\pi}{6},\
\dfrac{3\pi}{2},\ \dfrac{11\pi}{6}$$

$$\cos 3x + \cos 2x + \cos x = 0$$\\


Using Sum-To-Product Formula, we can simplify LHS as \begin{align*}
\cos 3x + \cos 2x + \cos x & = 0\\
(\cos 3x + \cos x) + \cos 2x & = 0\\
2\cos \dfrac{3x+x}{2} \cos \dfrac{3x-x}{2} + \cos 2x & = 0\\
2\cos 2x \cos x + \cos 2x & = 0\\
\cos 2x (2\cos x + 1) & = 0\\
\cos 2x = 0\ & \text{or}\ 2\cos x + 1 =0\\
2x = \cos^{-1} (0)\ & \text{or}\ \cos x = -\dfrac{1}{2}\\
2x = \cos^{-1} (0)\ & \text{or}\ x = \cos^{-1} \bigg(-\dfrac{1}{2}\bigg)
\end{align*}
The solutions for the first equation are $$2x = \dfrac{\pi}{2},\ \dfrac{3\pi}{2}$$
To get all solutions, we add integer multiples of $2\pi$ to these solutions. So,
the solutions are of the form $$2x = \dfrac{\pi}{2} + 2k\pi\ \ \ \text{and}\ \ \ 2x
= \dfrac{3\pi}{2} + 2k\pi$$
$$x = \dfrac{\pi}{4} + k\pi\ \ \ \text{and}\ \ \ x = \dfrac{3\pi}{4} + k\pi$$
The solutions that are in the interval $[0,2\pi)$ correspond to $k = 0$ and $1$.
For all other values of $k$, the corresponding values of $x$ lie outside this
interval. So, the solutions in the interval $[0,2\pi)$ are $$x = \dfrac{\pi}
{4},\ \dfrac{3\pi}{4},\ \dfrac{5\pi}{4},\ \dfrac{7\pi}{4}$$
In the interval $[0,2\pi)$, the solutions for the second equation are $$x =
\dfrac{2\pi}{3},\ \dfrac{4\pi}{3}$$\\
Therefore, all the solutions of the equation in the interval $[0,2\pi)$ are $$x
= \dfrac{\pi}{4},\ \dfrac{2\pi}{3},\ \dfrac{3\pi}{4},\ \dfrac{5\pi}{4},\
\dfrac{4\pi}{3},\ \dfrac{7\pi}{4}$$
$$\tan x + \sec x = \sqrt{3}$$\\
We can simplify LHS as \begin{align*}
\tan x + \sec x & = \sqrt{3}\\
\sec x & = \sqrt{3} - \tan x\\
\sec^2 x & = (\sqrt{3} - \tan x)^2\\
\sec^2 x & = 3 + \tan^2 x - 2\sqrt{3}\tan x\\
2\sqrt{3}\tan x & = 3 + \tan^2 x - \sec^2 x\\
2\sqrt{3}\tan x & = 3 - 1\\
\tan x & = \dfrac{1}{\sqrt{3}}\\
x & = \tan^{-1} \bigg(\dfrac{1}{\sqrt{3}}\bigg)
\end{align*}
The solutions for the above equation are $$x = \dfrac{\pi}{6},\ \dfrac{7\pi}{6}$$
But $x = \dfrac{7\pi}{6}$ does not satisfy the equation. \\
\\
Therefore, the only solution of the equation in the interval $[0,2\pi)$ is $$x =
\dfrac{\pi}{6}$$

$$2\cos x - 3\tan x = 0$$\\


We can simplify LHS as \begin{align*}
2\cos x - 3\tan x & = 0\\
2\cos x - \dfrac{3\sin x}{\cos x} & = 0\\
\dfrac{2\cos^2 x - 3\sin x}{\cos x} & = 0\\
2\cos^2 x - 3\sin x = 0\\
2(1 - \sin^2 x) - 3\sin x = 0\\
2 - 2\sin^2 x - 3\sin x = 0\\
2\sin^2 x + 3\sin x - 2 = 0\\
2\sin^2 x + 4\sin x - \sin x - 2 = 0\\
2\sin x(\sin x + 2) - (\sin x + 2) = 0\\
(\sin x + 2)(2\sin x - 1) = 0\\
\sin x + 2 = 0\ & \text{or}\ 2\sin x - 1 = 0\\
\sin x = -2\ & \text{or}\ \sin x = \dfrac{1}{2}\\
\end{align*}
Since sine function lies between $-1$ and $1$, there are no solutions for the first
equation $\sin x = -2$. In the interval $[0,2\pi)$ the solutions for the second
equation are $x = \dfrac{\pi}{6}$ and $x = \dfrac{5\pi}{6}$.\\
\\
Therefore, all the solutions of the equation in the interval $[0,2\pi)$ are $$x
= \dfrac{\pi}{6},\ \dfrac{5\pi}{6}$$

It is given that if a projectile is fired with a velocity $v_0$ at angle $\theta$,


then the maximum height it reaches (in ft), is modeled by the function $$M(\theta)
= \dfrac{v_0^2 \sin^2 \theta}{64}$$\\
When the projectile fired reaches a maximum height of $2000$ ft using\\ $v_0 = 400$
ft/s, then the function becomes \begin{align*}
M(\theta) & = \dfrac{v_0^2 \sin^2 \theta}{64}\\
2000 & = \dfrac{(400)^2 \sin^2 \theta}{64}\\
128000 & = 160000 \sin^2 \theta\\
\sin^2 \theta & = 0.8\\
\sin \theta & = \pm 0.89443\\
\theta & = \sin^{-1} (\pm 0.89443)\\
\end{align*}
Using calculator, we get \begin{align*}
\theta & \approx 63.4^o\ \text{or}\ 243.43^o\\
\end{align*}
For a maximum height, the projectile must be fired between angles $0$ and $90^o$.
Therefore, the projectile should be fired at an angle of $63.4^o$ approximately.

It is given that if a projectile is fired with a velocity $v_0$ at angle $\theta$,


then the maximum height it reaches (in ft), is modeled by the function $$M(\theta)
= \dfrac{v_0^2 \sin^2 \theta}{64}$$\\
When the projectile fired reaches a maximum height of $3000$ ft using\\ $v_0 = 400$
ft/s, then the function becomes \begin{align*}
M(\theta) & = \dfrac{v_0^2 \sin^2 \theta}{64}\\
3000 & = \dfrac{(400)^2 \sin^2 \theta}{64}\\
192000 & = 160000 \sin^2 \theta\\
\sin^2 \theta & = 1.2\\
\sin \theta & = \pm 1.09544\\
\theta & = \sin^{-1} (\pm 1.09544)\\
\end{align*}
But, we know that sine function lies between $-1$ and $1$. Therefore, there is no
value of $\theta$ for which the projectile reaches a maximum height of $3000$ ft.

It is given that if a projectile is fired with a velocity $v_0$ at angle $\theta$,


then the maximum height it reaches (in ft), is modeled by the function $$M(\theta)
= \dfrac{v_0^2 \sin^2 \theta}{64}$$\\
Now, the projectile will travel the highest when $M(\theta)$ is maximum or $
$M(\theta) = \dfrac{v_0^2 \sin^2 \theta}{64}\ \text{is maximum}$$
This happens when $\sin^2 \theta$ is maximum. \\
\\
Now, maximum value of $\sin^2 \theta$ is equal to 1 which happens when
\begin{align*}
\sin^2 \theta & = 1\\
\sin \theta & = \pm 1\\
\theta & = \sin^{-1} (\pm 1)\\
& = 90^o\ \text{or}\ 270^o
\end{align*}
Since projectile can not be fired below the ground, it should be fired vertically
upwards (at an angle of $90^o$ ) so that it reaches a maximum height.

Given that the displacement of an automobile shock absorber is modeled by the


function $$f(t) = 2^{-0.2t} \sin 4\pi t$$\\
For the shock absorber to be at its equilibrium position, i.e. $f(t) = 0$, the
equation becomes \begin{align*}
2^{-0.2t} \sin 4\pi t & = 0\\
\sin 4\pi t & = 0\ \ \ \ \ \ \ \ \ \ \ \ \ \ \ \ [\ \text{Exponential functions are
always positive}\ ]\\
4 \pi t & = \sin^{-1} (0)\\
4 \pi t & = n\pi \ \ \ \ \ \ \ \ \ \ \ \ \ \ \ [\ n\ \text{be any integer}\ ]\\
t & = \dfrac{n}{4}
\end{align*}\\
Therefore, the shock absorber is at its equilibrium position at times $$t= 0,\
0.25,\ 0.5,\ 0.75,\ 1,\ .\ .\ .\ .\ .\ .\ .\ $$
$$\cos 15^o$$\\
Half-Angle Formula for Cosine is $$\cos u = \pm \sqrt{\dfrac{1 + \cos u}{2}}$$\\
Since $15^o$ is half $30^o$, we use Half-Angle Formula for Cosine with $u = 30^o$.
We choose the $+$ sign because $15^o$ is in the first quadrant. \begin{align*}
\cos 15^o & = \cos \dfrac{30^o}{2}\\
& = \sqrt{\dfrac{1 + \cos 30^o}{2}}\ \ \ \ \ \ \ \ \ \ \ \ \ [\ \text{Half-Angle
Formula}\ ]\\
& = \sqrt{\dfrac{1 + \frac{\sqrt{3}}{2}}{2}}\ \ \ \ \ \ \ \ \ \ \ \ \ \ \ \ \
\bigg[\ \cos 30^o = \frac{\sqrt{3}}{2}\ \bigg]\\
& = \sqrt{\dfrac{2 + \sqrt{3}}{4}}\ \ \ \ \ \ \ \ \ \ \ \ \ \ \ \ \ [\ \text{Common
denominator}\ ]\\
& = \dfrac{1}{2}\sqrt{2 + \sqrt{3}}\ \ \ \ \ \ \ \ \ \ \ \ \ \ \ \ [\
\text{Simplify}\ ]\\
\end{align*}

$$\tan \dfrac{\pi}{8}$$\\
Half-Angle Formula for Tangent is $$\tan u = \dfrac{1 - \cos u}{\sin u}$$\\
Since $\dfrac{\pi}{8}$ is half $\dfrac{\pi}{4}$, we use Half-Angle Formula for
Tangent with $u = \dfrac{\pi}{4}$. We choose the $+$ sign because $\dfrac{\pi}{8}$
is in the first quadrant. \begin{align*}
\tan \dfrac{\pi}{8} & = \tan \frac{\frac{\pi}{4}}{2}\\
& = \dfrac{1 - \cos \frac{\pi}{4}}{\sin \frac{\pi}{4}}\ \ \ \ \ \ \ \ \ \ \ \ \ \ \
[\ \text{Half-Angle Formula}\ ]\\
& = \dfrac{1 - \frac{\sqrt{2}}{2}}{\frac{\sqrt{2}}{2}}\ \ \ \ \ \ \ \ \ \ \ \ \ \ \
\ \ \bigg[\ \cos \frac{\pi}{4} = \sin \frac{\pi}{4} = -\frac{\sqrt{2}}{2}\ \bigg]\\
& = \dfrac{2 - \sqrt{2}}{\sqrt{2}}\ \ \ \ \ \ \ \ \ \ \ \ \ \ \ \ \ [\ \text{Common
denominator}\ ]\\
& = \sqrt{2} - 1\ \ \ \ \ \ \ \ \ \ \ \ \ \ \ \ \ \ [\ \text{Simplify}\ ]\\
\end{align*}

$$2\sin \dfrac{\pi}{12} \cos \dfrac{\pi}{12}$$\\


Double-Angle Formula for Sine is $$\sin 2x = 2\sin x \cos x$$ Then, its Half-Angle
Formula will be $$\sin x = 2\sin \dfrac{x}{2} \cos \dfrac{x}{2}$$\\
Since $\dfrac{\pi}{12}$ is half $\dfrac{\pi}{6}$, we use tha above Half-Angle
Formula of Sine with $x = \dfrac{\pi}{6}$. We choose the $+$ sign because
$\dfrac{\pi}{12}$ is in the first quadrant. \begin{align*}
2\sin \dfrac{\pi}{12} \cos \dfrac{\pi}{12} & = \sin \dfrac{\pi}{6}\\
& = \dfrac{1}{2}
\end{align*}

$$\sin \dfrac{5\pi}{12}$$\\
Half-Angle Formula for Sine is $$\sin u = \pm \sqrt{\dfrac{1 - \cos u}{2}}$$\\
Since $\dfrac{5\pi}{12}$ is half $\dfrac{5\pi}{6}$, we use Half-Angle Formula for
Sine with $u = \dfrac{5\pi}{6}$. We choose the $+$ sign because $\dfrac{5\pi}{12}$
is in the first quadrant. \begin{align*}
\sin \dfrac{5\pi}{12} & = \sin \frac{\frac{5\pi}{6}}{2}\\
& = \sqrt{\dfrac{1 - \cos \frac{5\pi}{6}}{2}}\ \ \ \ \ \ \ \ \ \ \ \ \ [\
\text{Half-Angle Formula}\ ]\\
& = \sqrt{\dfrac{1 + \frac{\sqrt{3}}{2}}{2}}\ \ \ \ \ \ \ \ \ \ \ \ \ \ \ \ \
\bigg[\ \cos \frac{5\pi}{6} = -\frac{\sqrt{3}}{2}\ \bigg]\\
& = \sqrt{\dfrac{2 + \sqrt{3}}{4}}\ \ \ \ \ \ \ \ \ \ \ \ \ \ \ \ \ [\ \text{Common
denominator}\ ]\\
& = \dfrac{1}{2}\sqrt{2 + \sqrt{3}}\ \ \ \ \ \ \ \ \ \ \ \ \ \ \ \ [\
\text{Simplify}\ ]\\
\end{align*}

$$\sin 5^o \cos 40^o + \cos 5^o \sin 40^o$$\\


Addition Formula for Sine is $$\sin (x + y) = \sin x \cos y + \cos x \sin y$$\\
Let $x = 5^o$ and $y = 40^o$, then \begin{align*}
\sin 5^o \cos 40^o + \cos 5^o \sin 40^o & = \sin (5^o + 40^o)\\
& = \sin 45^o\\
& = \dfrac{\sqrt{2}}{2}
\end{align*}

$$\dfrac{\tan 66^o - \tan 6^o}{1 + \tan 66^o \tan 6^o}$$\\


Subtraction Formula for Tangent is $$\tan (x - y) = \dfrac{\tan x - \tan y}{1 +
\tan x \tan y}$$\\
Let $x = 66^o$ and $y = 6^o$, then \begin{align*}
\dfrac{\tan 66^o - \tan 6^o}{1 + \tan 66^o \tan 6^o} & = \tan (66^o - 6^o)\\
& = \tan 60^o\\
& = \sqrt{3}
\end{align*}

$$\cos^2 \dfrac{\pi}{8} - \sin^2 \dfrac{\pi}{8}$$\\


Double-Angle Formula for Cosine is $$\cos 2x = \cos^2 x - \sin^2 x$$ Then, its
Half-Angle Formula will be $$\cos x = \cos^2 \dfrac{x}{2} - \sin^2 \dfrac{x}{2}$$\\
Since $\dfrac{\pi}{8}$ is half $\dfrac{\pi}{4}$, we use tha above Half-Angle
Formula of Cosine with $x = \dfrac{\pi}{4}$. \begin{align*}
\cos^2 \dfrac{\pi}{8} - \sin^2 \dfrac{\pi}{8} & = \cos \dfrac{\pi}{4}\\
& = \dfrac{\sqrt{2}}{2}
\end{align*}

$$\dfrac{1}{2} \cos \dfrac{\pi}{12} + \dfrac{\sqrt{3}}{2} \sin \dfrac{\pi}{12}$$\\


We know that for $x = \dfrac{\pi}{6}$, we have $$\sin \dfrac{\pi}{6} = \dfrac{1}
{2}\ \ \ \text{and}\ \ \ \ \cos \dfrac{\pi}{6} = \dfrac{\sqrt{3}}{2}$$ Now, we
replace the terms $\dfrac{1}{2}$ and $\dfrac{\sqrt{3}}{2}$ in the above expression
by $\sin \dfrac{\pi}{6}$ and $\cos \dfrac{\pi}{6}$ respectively. Then, the
expression becomes $$\sin \dfrac{\pi}{6} \cos \dfrac{\pi}{12} + \cos \dfrac{\pi}{6}
\sin \dfrac{\pi}{12}$$\\
Now, we know that the Addition Formula for Sine is $$\sin (x + y) = \sin x \cos y +
\cos x \sin y$$
Let $x = \dfrac{\pi}{6}$ and $y = \dfrac{\pi}{12}$, then \begin{align*}
\dfrac{1}{2} \cos \dfrac{\pi}{12} + \dfrac{\sqrt{3}}{2} \sin \dfrac{\pi}{12} & =
\sin \dfrac{\pi}{6} \cos \dfrac{\pi}{12} + \cos \dfrac{\pi}{6} \sin \dfrac{\pi}
{12}\\
& = \sin \bigg(\dfrac{\pi}{6} + \dfrac{\pi}{12}\bigg)\\
& = \sin \dfrac{3\pi}{12}\\
& = \sin \dfrac{\pi}{4}\\
& = \dfrac{\sqrt{2}}{2}
\end{align*}

$$\cos 37.5^o \cos 7.5^o$$\\


The third Product-To-Sum Formula gives $$\cos u \cos v = \dfrac{1}{2}[\cos (u + v)
+ \cos (u - v)]$$
Let $u = 37.5^o$ and $v = 7.5^o$, then we get \begin{align*}
\cos 37.5^o \cos 7.5^o & = \dfrac{1}{2}[\cos (37.5^o + 7.5^o) + \cos (37.5^o -
7.5^o)]\\
& = \dfrac{1}{2}[\cos 45^o + \cos 30^o]\\
& = \dfrac{1}{2}\bigg[\dfrac{\sqrt{2}}{2} + \dfrac{\sqrt{3}}{2}\bigg]\\
& = \dfrac{1}{2}\bigg[\dfrac{\sqrt{2} + \sqrt{3}}{2}\bigg]\\
& = \dfrac{\sqrt{2} + \sqrt{3}}{4}
\end{align*}

$$\cos 67.5^o + \cos 22.5^o$$\\


The third Sum-To-Product Formula gives $$\cos u + \cos v = 2\cos \bigg(\dfrac{u +
v}{2}\bigg) \cos \bigg(\dfrac{u - v}{2}\bigg)$$\\
Let $u = 67.5^o$ and $v = 22.5^o$, then we get \begin{align*}
\cos 67.5^o + \cos 22.5^o & = 2\cos \bigg(\dfrac{67.5^o + 22.5^o}{2}\bigg) \cos
\bigg(\dfrac{67.5^o - 22.5^o}{2}\bigg)\\
& = 2\cos \bigg(\dfrac{90^o}{2}\bigg) \cos \bigg(\dfrac{45^o}{2}\bigg)\\
& = 2\cos 45^o \cos 22.5^o\\
& = 2 \bigg(\dfrac{\sqrt{2}}{2}\bigg) \bigg[\sqrt{ \dfrac{1 + \cos 45^o}{2}}
\bigg]\ \ \ \ \ \ \ \ \ \ \ \ [\ \text{Half-Angle Formula}\ ]\\
& = \sqrt{2}\bigg[\sqrt{\dfrac{1 + \frac{\sqrt{2}}{2}}{2}}\bigg]\ \ \ \ \ \ \ \ \ \
\ \ \ \ \ \ \ \ \ \ \ \ \ \bigg[\ \cos 45^o = \frac{\sqrt{2}}{2}\ \bigg]\\
& = \sqrt{\dfrac{2 + \sqrt{2}}{2}}\ \ \ \ \ \ \ \ \ \ \ \ \ \ \ \ \ \ \ \ \ \ \ \ \
\ \ \ \ \ [\ \text{Simplify}\ ]
\end{align*}\\

$$\sin (x+y)$$\\
Given $\sec x = \dfrac{3}{2},\ \csc y = 3$, and $x$ and $y$ are in Quadrant I.
Using Reciprocal Identity, we have $$\cos x = \dfrac{1}{\sec x} = \dfrac{2}
{3}\ \ \text{and}\ \ \sin y = \dfrac{1}{\csc y} = \dfrac{1}{3}$$\\
Since both $x$ and $y$ are in Quadrant I, we have $$\sin x = \sqrt{1 - \cos^2 x}
= \sqrt{1 - \bigg(\frac{2}{3}\bigg)^2} = \dfrac{\sqrt{5}}{3}$$
And $$\cos y = \sqrt{1 - \sin^2 y} = \sqrt{1 - \bigg(\frac{1}{3}\bigg)^2} =
\dfrac{2\sqrt{2}}{3}$$\\
\\
Addition Formula for Sine is $$\sin (x+y) = \sin x \cos y + \cos x \sin y$$
Substituting the above values in Addition Formula for Sine, we get \begin{align*}
\sin(x + y) & = \sin x \cos y + \cos x \sin y\\
& = \bigg(\dfrac{\sqrt{5}}{3}\bigg) \bigg(\dfrac{2\sqrt{2}}{3}\bigg) +
\bigg(\dfrac{2}{3}\bigg)\bigg(\dfrac{1}{3}\bigg)\\
& = \dfrac{2\sqrt{10}}{9} + \dfrac{2}{9}\\
& = \dfrac{2\sqrt{10} + 2}{9}\\
& = \dfrac{2}{9}(\sqrt{10} + 1)
\end{align*}

$$\tan (x+y)$$\\
Given $\sec x = \dfrac{3}{2},\ \csc y = 3$, and $x$ and $y$ are in Quadrant I.
Using Reciprocal Identity, we have $$\cos x = \dfrac{1}{\sec x} = \dfrac{2}
{3}\ \ \text{and}\ \ \sin y = \dfrac{1}{\csc y} = \dfrac{1}{3}$$\\
Since both $x$ and $y$ are in Quadrant I, we have $$\sin x = \sqrt{1 - \cos^2 x}
= \sqrt{1 - \bigg(\frac{2}{3}\bigg)^2} = \dfrac{\sqrt{5}}{3}$$
And $$\cos y = \sqrt{1 - \sin^2 y} = \sqrt{1 - \bigg(\frac{1}{3}\bigg)^2} =
\dfrac{2\sqrt{2}}{3}$$\\
\\
Therefore, we have $$\tan x = \dfrac{\sin x}{\cos x} = \dfrac{\sqrt{5}/3}{2/3} =
\dfrac{\sqrt{5}}{2}$$ And $$\tan y = \dfrac{\sin y}{\cos y} = \dfrac{1/3}
{2\sqrt{2}/3} = \dfrac{1}{2\sqrt{2}} = \dfrac{\sqrt{2}}{4}$$ \\

Addition Formula for Tangent is $$\tan (x+y) = \dfrac{\tan x + \tan y}{1 - \tan
x \tan y}$$
Substituting the above values in Addition Formula for Tangent, we get
\begin{align*}
\tan (x+y) & = \dfrac{\tan x + \tan y}{1 - \tan x \tan y}\\
& = \dfrac{\frac{\sqrt{5}}{2} + \frac{\sqrt{2}}{4}}{1 - \frac{\sqrt{5}}
{2}\big(\frac{\sqrt{2}}{4}\big)}\\
& = \dfrac{4\sqrt{5} + 2\sqrt{2}}{8 - \sqrt{10}}\ \ \ \ \ \ \ \ \ \ \ \ \ \ \ \ \ \
\ \ \ \ \ \ \ \ \ \ \ \ \ \ \ [\ \text{Common denominator}\ ]\\
& = \dfrac{4\sqrt{5} + 2\sqrt{2}}{8 - \sqrt{10}} \times \dfrac{8 + \sqrt{10}}{8
+ \sqrt{10}}\ \ \ \ \ \ \ \ \ \ \ \ \ \ \ \ \ \ [\ \text{Rationalize}\ ]\\
& = \dfrac{32\sqrt{5} + 4\sqrt{50} + 16\sqrt{2} + 2\sqrt{20}}{64 -
10} \ \ \ \ \ \ \ \ \ [\ \text{Simplify}\ ]\\
& = \dfrac{32\sqrt{5} + 20\sqrt{2} + 16\sqrt{2} + 4\sqrt{5}}{54}\\
& = \dfrac{36\sqrt{5} + 36\sqrt{2}}{54}\\
& = \dfrac{2}{3}(\sqrt{5} + \sqrt{2})\\
\end{align*}

$$\sin 2x$$\\
Given $\sec x = \dfrac{3}{2}$, and $x$ is in Quadrant I. Using Reciprocal Identity,
we have $$\cos x = \dfrac{1}{\sec x} = \dfrac{2}{3}$$\\
Since $x$ is in Quadrant I, we have $$\sin x = \sqrt{1 - \cos^2 x} = \sqrt{1 -
\bigg(\frac{2}{3}\bigg)^2} = \dfrac{\sqrt{5}}{3}$$\\
Double-Angle Formula for Sine is $$\sin 2x = 2 \sin x \cos x $$
Substituting the above values in Double-Angle Formula for Sine, we get
\begin{align*}
\sin 2x & = 2\sin x \cos x\\
& = 2\bigg(\dfrac{\sqrt{5}}{3}\bigg) \bigg(\dfrac{2}{3}\bigg)\\
& = \dfrac{4\sqrt{5}}{9}\\
\end{align*}

$$\cos \dfrac{y}{2}$$\\
Given $\csc y = 3$ and $y$ is in Quadrant I. Using Reciprocal Identity, we have $
$\sin y = \dfrac{1}{\csc y} = \dfrac{1}{3}$$\\
Since $y$ is in Quadrant I, we have $$\cos y = \sqrt{1 - \sin^2 y} = \sqrt{1 -
\bigg(\frac{1}{3}\bigg)^2} = \dfrac{2\sqrt{2}}{3}$$\\
Half-Angle Formula for Cosine is $$\cos \dfrac{y}{2} = \sqrt{\dfrac{1 + \cos y}{2}}
$$
Substituting the above values in Half-Angle Formula for Cosine, we get
\begin{align*}
\cos \dfrac{y}{2} & = \sqrt{\dfrac{1 + \cos y}{2}}\\
& = \sqrt{\dfrac{1 + \frac{2\sqrt{2}}{3}}{2}}\\
& = \sqrt{\dfrac{\frac{3 + 2\sqrt{2}}{3}}{2}}\\
& = \sqrt{\dfrac{3 + 2\sqrt{2}}{6}}
\end{align*}

$$\tan \dfrac{y}{2}$$\\
Given $\csc y = 3$ and $y$ is in Quadrant I. Using Reciprocal Identity, we have $
$\sin y = \dfrac{1}{\csc y} = \dfrac{1}{3}$$\\
Since $y$ is in Quadrant I, we have $$\cos y = \sqrt{1 - \sin^2 y} = \sqrt{1 -
\bigg(\frac{1}{3}\bigg)^2} = \dfrac{2\sqrt{2}}{3}$$\\
Half-Angle Formula for Tangent is $$\tan \dfrac{y}{2} = \dfrac{1 - \cos y}{\sin u}$
$
Substituting the above values in Half-Angle Formula for Tangnet, we get
\begin{align*}
\tan \dfrac{y}{2} & = \dfrac{1 - \cos y}{\sin y}\\
& = \dfrac{1 - \frac{2\sqrt{2}}{3}}{\frac{1}{3}}\\
& = 3 - 2\sqrt{2}\ \ \ \ \ \ \ \ \ \ \ \ \ \ \ \ \ \ \ \ \ \ \ \ [\ \text{Common
denominator}\ ]
\end{align*}

$$\tan \bigg(2\cos^{-1} \frac{3}{7}\bigg)$$\\


Let $u = \cos^{-1} \frac{3}{7}$, then $\cos u = \frac{3}{7}$. And so, we have $
$\sin u = \sqrt{1 - \cos^2 u} = \sqrt{1 - \bigg(\frac{3}{7}\bigg)^2} =
\dfrac{2\sqrt{10}}{7}$$
Using Reciprocal Identity, we have $$\tan u = \dfrac{\sin u}{\cos u} =
\dfrac{\frac{2\sqrt{10}}{7}}{\frac{3}{7}} = \dfrac{2\sqrt{10}}{3}$$
So, now, we use the Double-Angle Formula for Tangent. \begin{align*}
\tan \bigg(2\cos^{-1} \frac{3}{7}\bigg) & = \tan 2 u\ \ \ \ \ \ \ \ \ \ \ \ \ \ \ \
\ \ \ \ \ \ \ \ \ \ \ \ \ \ \ \bigg[\ \text{Since}\ \cos^{-1} \frac{3}{7} = u\
\bigg]\\
& = \dfrac{2 \tan u}{1 - \tan^2
u} \ \ \ \ \ \ \ \ \ \ \ \ \ \ \ \ \ \ \ \ \ \ \ \ \ \ [\ \text{Double-Angle
Formula}\ ]\\
& = \dfrac{2 \bigg(\dfrac{2\sqrt{10}}{3}\bigg)}{1 - \bigg(\dfrac{2\sqrt{10}}
{3}\bigg)^2}\\
& = \dfrac{\dfrac{4\sqrt{10}}{3}}{\dfrac{9 - 40}{9}}\\
& = -\dfrac{12\sqrt{10}}{31}\\
\end{align*}\\

$$\sin\bigg(\tan^{-1} \dfrac{3}{4} + \cos^{-1} \dfrac{5}{13}\bigg)$$\\


Let $\theta = \tan^{-1} \dfrac{3}{4}$ and $\phi = \cos^{-1} \dfrac{5}{13}$. Using
the methods of Section 6.4, we sketch triangles with angles $\theta$ and $\phi$
such that $\tan \theta = \dfrac{3}{4}$ and $\cos \phi = \dfrac{5}{13}$. From the
triangles, we have $$\sin \theta = \dfrac{3}{5},\ \ \cos \theta = \dfrac{4}
{5}\ \ \text{and}\ \ \sin \phi = \dfrac{12}{13}$$\\
From the Addition Formula For Sine, we have \begin{align*}
\sin\bigg(\tan^{-1} \dfrac{3}{4} + \cos^{-1} \dfrac{5}{13}\bigg) & = \sin(\theta
+ \phi)\\
& = \sin \theta \cos \phi + \cos \theta \sin \phi\\
& = \dfrac{3}{5} \cdot \dfrac{5}{13} + \dfrac{4}{5} \cdot \dfrac{12}{13}\\
& = \dfrac{15}{65} + \dfrac{48}{65}\\
& = \dfrac{15 + 48}{65}\\
& = \dfrac{63}{65}\\
\end{align*}

$$\tan (2\tan^{-1} x)$$\\


Let $u = \tan^{-1} x$, then $\tan u = x$.\\
\\
So, now, we use the Double-Angle Formula for Tangent. \begin{align*}
\tan (2\tan^{-1} u) & = \tan 2 u\ \ \ \ \ \ \ \ \ \ \ \ \ \ \ \ \ \ \ \ \ \ \ \ \
[\ \text{Since}\ \tan^{-1} x = u\ ]\\
& = \dfrac{2 \tan u}{1 - \tan^2 u} \ \ \ \ \ \ \ \ \ \ \ \ \ \ \ \ \ \ \ [\
\text{Double-Angle Formula}\ ]\\
& = \dfrac{2x}{1 - x^2}\\
\end{align*}\\

$$\cos(\sin^{-1} x + \cos^{-1} y)$$\\


Let $\theta = \sin^{-1} x$ and $\phi = \cos^{-1} y$. Using the methods of Section
6.4, we sketch triangles with angles $\theta$ and $\phi$ such that $\sin \theta =
x$ and $\cos \phi = y$. From the triangles, we have $$\cos \theta = \sqrt{1 -
\sin^2 \theta} = \sqrt{1 - x^2}$$
And $$\sin \phi = \sqrt{1 - \cos^2 \phi} = \sqrt{1 - y^2}$$\\
From the Addition Formula For Cosine, we have \begin{align*}
\cos(\sin^{-1} x + \cos^{-1} y) & = \cos(\theta + \phi)\\
& = \cos \theta \cos \phi - \sin \theta \sin \phi\\
& = (\sqrt{1 - x^2})(y) - (x)(\sqrt{1 - y^2})\\
& = y\sqrt{1 - x^2} - x\sqrt{1 - y^2}\\
\end{align*}

Given that a driver approaches a 10-ft-wide highway sign adjacent to a roadway as


shown in the figure below. The viewing angle $\theta$ changes accordingly. \\
\\
From the below figure, in the triangle formed, we have \begin{align*}
\tan \theta & = \dfrac{\text{opp}}{\text{adj}}\\
& = \dfrac{10}{x}\\
\theta & = \tan^{-1} \bigg(\dfrac{10}{x}\bigg)
\end{align*}

From the previous exercise we found out that the viewing angle $\theta$ as a
function of distance $x$ between the driver and the sign is modeled by the function
$$\theta = \tan^{-1} \bigg(\dfrac{10}{x}\bigg)$$\\
Now, it is given that the sign is legible when the viewing angle is $2^o$ or
greater. This happens when \begin{align*}
\theta & \geq 2^o\\
\tan^{-1} \bigg(\dfrac{10}{x}\bigg) & \geq 2\\
\dfrac{10}{x} & \geq \tan 2\\
\dfrac{10}{x} & \geq 0.03492\\
x & \leq \dfrac{10}{0.03492}\\
& \leq 286.4\ \text{ft}
\end{align*}
So, the sign first becomes legible at a distance of 286.4 feet from the sign.

Given that a driver is approaching a $380$-ft-tall building that supports a $40$-ft


communications tower as shown in the figure below. The driver is at a distance $x$
from the building with a viewing angle $\theta$ which changes as he approaches
it. \\
\\
Let $\alpha$ and $\beta$ be the angles of elevations to the top of the tower and
building respectively. Now, from the figure, in the $\Delta$ABD, we have
\begin{align*}
\tan \beta & = \dfrac{\text{opp}}{\text{adj}} = \dfrac{BD}{AB}\\
& = \dfrac{380}{x}\\
\beta & = \tan^{-1} \bigg(\dfrac{380}{x}\bigg)
\end{align*}\\
Now, from the figure, in the $\Delta$ABC, we have \begin{align*}
\tan \alpha & = \dfrac{\text{opp}}{\text{adj}} = \dfrac{BC}{AB}\\
& = \dfrac{420}{x}\\
\alpha & = \tan^{-1} \bigg(\dfrac{420}{x}\bigg)
\end{align*}\\
From the figure, the viewing angle $\theta$ is \begin{align*}
\theta & = \alpha - \beta\\
& = \tan^{-1} \bigg(\dfrac{420}{x}\bigg) - \tan^{-1} \bigg(\dfrac{380}{x}\bigg)
\end{align*}

$$\tan \theta \sin \theta + \cos \theta = \sec \theta$$


\begin{align*}
\text{LHS}\ & = \tan \theta \sin \theta + \cos \theta\\
& = \bigg(\dfrac{\sin \theta}{\cos \theta}\bigg) \sin \theta + \cos \theta\ \ \ \ \
\ \ \ \ \ \ [\ \text{Reciprocal Identity}\ ]\\
& = \dfrac{\sin^2 \theta + \cos^2 \theta}{\cos \theta}\ \ \ \ \ \ \ \ \ \ \ \ \ \ \
\ \ \ \ [\ \text{Common denominator}\ ]\\
& = \dfrac{1}{\cos \theta}\ \ \ \ \ \ \ \ \ \ \ \ \ \ \ \ \ \ \ \ \ \ \ \ \ \ \ \ \
\ \ \ [\ \text{Pythagorean Identity}\ ]\\
& = \sec \theta\ \ \ \ \ \ \ \ \ \ \ \ \ \ \ \ \ \ \ \ \ \ \ \ \ \ \ \ \ \ \ \
[\ \text{Reciprocal Identity}\ ]\\
& = \text{RHS}
\end{align*}

$$\dfrac{\tan x}{1 - \cos x} = \csc x(1 + \sec x)$$


\begin{align*}
\text{LHS}\ & = \dfrac{\tan x}{1 - \cos x}\\
& = \dfrac{\tan x}{1 - \cos x} \times \dfrac{1 + \cos x}{1 + \cos x}\ \ \ \ \ \ \ \
\ \ \ [\ \text{Rationalize}\ ]\\
& = \dfrac{\tan x(1 + \cos x)}{1 - \cos^2 x}\\
& = \dfrac{\frac{\sin x}{\cos x}(1 + \cos x)}{\sin^2
x}\ \ \ \ \ \ \ \ \ \ \ \ \ \ \ \ \ \ \ [\ \text{Pythagorean and Reciprocal
Identities}\ ]\\
& = \dfrac{1}{\sin x}\bigg(\dfrac{1 + \cos x}{\cos
x}\bigg)\ \ \ \ \ \ \ \ \ \ \ \ \ \ \ \ [\ \text{Simplify}\ ]\\
& = \csc x(1 + \sec x)\ \ \ \ \ \ \ \ \ \ \ \ \ \ \ \ \ \ \ \ [\ \text{Reciprocal
Identity}\ ]\\
& = \text{RHS}
\end{align*}
$$\dfrac{2\tan x}{1 + \tan^2 x} = \sin 2x$$
\begin{align*}
\text{LHS}\ & = \dfrac{2\tan x}{1 + \tan^2 x}\\
& = \dfrac{2\tan x}{\sec^2 x}\ \ \ \ \ \ \ \ \ \ \ \ \ \ \ \ \ [\ \text{Pythagorean
Identity}\ ]\\
& = \dfrac{2\frac{\sin x}{\cos x}}{\frac{1}{\cos^2 x}}\ \ \ \ \ \ \ \ \ \ \ \ \ \ \
\ \ \ \ [\ \text{Reciprocal Identity}\ ]\\
& = 2\sin x \cos x\\
& = \sin 2x\ \ \ \ \ \ \ \ \ \ \ \ \ \ \ \ \ \ \ [\ \text{Double-Angle
Formula}\ ]\\
& = \text{RHS}
\end{align*}

$$\sin x \tan \bigg(\dfrac{x}{2}\bigg) = 1 - \cos x$$


\begin{align*}
\text{LHS}\ & = \sin x \tan \bigg(\dfrac{x}{2}\bigg)\\
& = \sin x \bigg(\dfrac{1 - \cos x}{\sin x}\bigg)\ \ \ \ \ \ \ \ \ \ \ \ \ [\
\text{Half-Angle Formula}\ ]\\
& = 1- \cos x\\
& = \text{RHS}
\end{align*}

$$2\sin^2 (3x) = 1 - \cos (6x)$$\\


Double-Angle Formula for Cosine states that $$\cos 2x = 1 - 2 \sin^2 x $$ So, $
$2 \sin^2 x = 1 - \cos 2x$$
\begin{align*}
\text{LHS}\ & = 2\sin^2 (3x)\\
& = 1 - \cos 2(3x)\ \ \ \ \ \ \ \ \ \ \ \ \ [\ \text{Double-Angle Formula}\ ]\\
& = 1- \cos (6x)\\
& = \text{RHS}
\end{align*}

$$\Bigg(\sin \bigg(\dfrac{x}{2}\bigg) + \cos \bigg(\dfrac{x}{2}\bigg)\Bigg)^2 = 1 +


\sin x$$
\begin{align*}
\text{LHS}\ & = \Bigg(\sin \bigg(\dfrac{x}{2}\bigg) + \cos \bigg(\dfrac{x}
{2}\bigg)\Bigg)^2\\
& = \sin^2 \bigg(\dfrac{x}{2}\bigg) + \cos^2 \bigg(\dfrac{x}{2}\bigg) + 2 \sin
\bigg(\dfrac{x}{2}\bigg) \cos \bigg(\dfrac{x}{2}\bigg)\\
& = 1 + 2 \sin \bigg(\dfrac{x}{2}\bigg) \cos \bigg(\dfrac{x}
{2}\bigg)\ \ \ \ \ \ \ \ \ \ \ \ \ \ \ \ \ \ \ \ \ \ \ \ \ \ \ \ \ [\
\text{Pythagorean Identity}\ ]\\
& = 1 + \sin x\ \ \ \ \ \ \ \ \ \ \ \ \ \ \ \ \ \ \ \ \ \ \ \ \ \ \ \ \ \ \ \ \ \ \
\ \ \ \ \ \ \ \ \ \ \ \ \ [\ \text{Half-Angle Formula}\ ]\\
& = \text{RHS}
\end{align*}

$$\sin 75^o$$\\
Half-Angle Formula for Sine is $$\sin u = \pm \sqrt{\dfrac{1 - \cos u}{2}}$$\\
Since $75^o$ is half $150^o$, we use Half-Angle Formula for Sine with $u = 150^o$.
We choose the $+$ sign because $75^o$ is in the first quadrant. \begin{align*}
\sin 75^o & = \sin \dfrac{150^o}{2}\\
& = \sqrt{\dfrac{1 - \sin 150^o}{2}}\ \ \ \ \ \ \ \ \ \ \ \ \ [\ \text{Half-Angle
Formula}\ ]\\
& = \sqrt{\dfrac{1 - \frac{1}{2}}{2}}\ \ \ \ \ \ \ \ \ \ \ \ \ \ \ \ \ \ \ \ \
\bigg[\ \sin 150^o = \frac{1}{2}\ \bigg]\\
& = \sqrt{\dfrac{2 - 1}{4}}\ \ \ \ \ \ \ \ \ \ \ \ \ \ \ \ \ \ \ \ \ \ [\
\text{Common denominator}\ ]\\
& = \dfrac{1}{2} \ \ \ \ \ \ \ \ \ \ \ \ \ \ \ \ \ \ \ \ \ \ \ \ \ \ \ \ \ \ [\
\text{Simplify}\ ]\\
\end{align*}

$$(2\cos \theta - 1)(\sin \theta - 1) = 0$$\\


We simplify the left-hand side of the equation. \begin{align*}
(2\cos \theta - 1)(\sin \theta - 1) & = 0\\
2 \cos \theta - 1 = 0\ \text{or}\ \sin \theta - 1 & = 0\\
\cos \theta = \dfrac{1}{2}\ \text{or}\ \sin \theta & = 1\\
\theta = \cos^{-1} \bigg(\dfrac{1}{2}\bigg)\ \text{or}\ \theta & = \sin^{-1} (1)\\
\end{align*}
In the interval $[0,2\pi)$ the solutions for the first equation are $$x =
\dfrac{\pi}{3}\ \text{and}\ \ \dfrac{5\pi}{3}$$ and the solutions for the second
equation is $$x = \dfrac{\pi}{2}$$\\
\\
Therefore, all the solutions of the equation in the interval $[0,2\pi)$ are $$x
= \dfrac{\pi}{3},\ \dfrac{5\pi}{3}\ \text{and}\ \dfrac{\pi}{2}$$

$$2\tan \bigg(\dfrac{x}{2}\bigg) - \csc x = 0$$\\


We factor the left-hand side of the equation. \begin{align*}
2\tan \bigg(\dfrac{x}{2}\bigg) - \csc x & = 0\\
2\bigg(\dfrac{1 - \cos x}{\sin x}\bigg) - \csc x & = 0\ \ \ \ \ \ \ \ \ \ \ \
[\ \text{Half-Angle Formula}\ ]\\
2\bigg(\dfrac{1 - \cos x}{\sin x}\bigg) - \dfrac{1}{\sin x} & = 0\ \ \ \ \ \ \ \ \
\ \ \ [\ \text{Reciprocal Identity}\ ]\\
\dfrac{2 - 2\cos x - 1}{\sin x} & = 0\\
1 - 2\cos x & = 0\\
\cos x & = \dfrac{1}{2}\\
x & = \cos^{-1} \bigg(\dfrac{1}{2}\bigg)\\
\end{align*}
In the interval $[0,2\pi)$ the solutions for the equation are $$x = \dfrac{\pi}
{3},\ \dfrac{5\pi}{3}$$

$$\sin(\cos^{-1} x - \tan^{-1} y)$$\\


Let $\theta = \cos^{-1} x$ and $\phi = \tan^{-1} y$. Using the methods of Section
6.4, we sketch triangles with angles $\theta$ and $\phi$ such that $\cos \theta =
x$ and $\tan \phi = y$. From the triangles, we have $$\sin \theta = \sqrt{1-
x^2},\ \ \sin \phi = \dfrac{y}{\sqrt{1+y^2}},\ \ \text{and}\ \ \cos \phi =
\dfrac{1}{\sqrt{1+y^2}}$$\\
From the Addition Formula For Sine, we have \begin{align*}
\sin(cos^{-1} x - \tan^{-1} y) & = \sin(\theta - \phi)\\
& = \sin \theta \cos \phi - \cos \theta \sin \phi\\
& = \sqrt{1-x^2} \cdot \dfrac{1}{\sqrt{1+y^2}} - x \cdot \dfrac{y}{\sqrt{1+y^2}}\\
& = \dfrac{\sqrt{1-x^2}}{\sqrt{1+y^2}} - \dfrac{xy}{\sqrt{1+y^2}}\\
& = \dfrac{\sqrt{1-x^2} - xy}{\sqrt{1+y^2}}\\
\end{align*}

It is given that a wave on the surface of a long canal is described by the function
$$y(x,t) = 5\sin \bigg(2x - \dfrac{\pi}{2}t\bigg) \ \ \ \ \ \ \ x \geq 0$$\\
\\
(a)\ When the position of the point is at $x = 0$ at any time $t$, the function
becomes $$y(0,t) = 5\sin \bigg(2(0) - \dfrac{\pi}{2}t\bigg) = 5\sin \bigg( -
\dfrac{\pi}{2}t\bigg)$$
And using Even-Odd Identities, the function gets modeled to $$y(t) = -5\sin
\bigg(\dfrac{\pi}{2}t\bigg)$$

(c)\ To find the velocity of the wave, we express the given function in the
standard form $$y(x,t) = A \sin k(x - vt)$$ where $v$ is the velocity of the
wave.\\
\\
The given function is $$y(x,t) = 5\sin \bigg(2x - \dfrac{\pi}{2}t\bigg)$$ It can be
expressed in standard form as $$y(x,t) = 5\sin 2\bigg(x - \dfrac{\pi}{4}t\bigg)$$
Comparing this to the standard form, we see that the wave is moving with velocity
$v = \dfrac{\pi}{4}$.

Given that travelling waves are generated at each end of a tightly stretched rope
$24$ ft long, with equations $$y = 0.2 \sin (1.047x - 0.524t)$$ and $$y = 0.2 \sin
(1.047x + 0.524t)$$\\
(a)\ The combined wave is obtained by adding the two equations. \begin{align*}
y & = 0.2 \sin (1.047x - 0.524t) + 0.2 \sin (1.047x + 0.524t)\\
\\
& = 0.2 (\sin (1.047x - 0.524t) + \sin (1.047x + 0.524t))
\end{align*}\\
Now, we use the first Sum-To-Product Formula, i.e. $$\sin u + \sin v = 2\sin
\bigg(\dfrac{u + v}{2}\bigg) \cos \bigg(\dfrac{u - v}{2}\bigg)$$
Let $u = 1.047x - 0.524t$ and $v = 1.047x + 0.524t$, then we have $$u + v = 1.047x
- 0.524t + 1.047x - 0.524t = 2.094$$ and $$u - v = 1.047x - 0.524t - 1.047x -
0.524t = -1.048t$$\\
\\
Substituting these values in Sum-To-Product Formula, we get \begin{align*}
y & = 0.2 (\sin (1.047x - 0.524t) + \sin (1.047x + 0.524t))\\
& = 0.2\bigg[2\sin \bigg(\dfrac{2.094x}{2}\bigg) \cos \bigg(\dfrac{-1.048t}
{2}\bigg)\bigg]\\
& = 0.4\sin (1.047x) \cos (-0.524t)\\
& = 0.4\sin (1.047x) \cos (0.524t)\ \ \ \ \ \ \ \ \ \ \ \ \ \ \ [\ \text{Even-Odd
Identity}\ ]\\
\end{align*}

Given that travelling waves are generated at each end of a tightly stretched rope
$24$ ft long, with equations $$y = 0.2 \sin (1.047x - 0.524t)$$ and $$y = 0.2 \sin
(1.047x + 0.524t)$$\\
(a)\ The combined wave is obtained by adding the two equations. \begin{align*}
y & = 0.2 \sin (1.047x - 0.524t) + 0.2 \sin (1.047x + 0.524t)\\
\\
& = 0.2 [\sin (1.047x - 0.524t) + \sin (1.047x + 0.524t)]
\end{align*}\\
Now, we use the first Sum-To-Product Formula, i.e. $$\sin u + \sin v = 2\sin
\bigg(\dfrac{u + v}{2}\bigg) \cos \bigg(\dfrac{u - v}{2}\bigg)$$\\
Let $u = 1.047x - 0.524t$ and $v = 1.047x + 0.524t$, then we have $$u + v = 1.047x
- 0.524t + 1.047x - 0.524t = 2.094$$ and $$u - v = 1.047x - 0.524t - 1.047x -
0.524t = -1.048t$$\\
\\
Substituting these values in Sum-To-Product Formula, we get \begin{align*}
y & = 0.2 [\sin (1.047x - 0.524t) + \sin (1.047x + 0.524t)]\\
& = 0.2\bigg[2\sin \bigg(\dfrac{2.094x}{2}\bigg) \cos \bigg(\dfrac{-1.048t}
{2}\bigg)\bigg]\\
& = 0.4\sin (1.047x) \cos (-0.524t)\\
& = 0.4\sin (1.047x) \cos (0.524t)\ \ \ \ \ \ \ \ \ \ \ \ \ \ \ [\ \text{Even-Odd
Identity}\ ]\\
\end{align*}
The nodes occur at the values of $x$ for which $\sin (1.047x) = 0$, that is, where
$1.047x = k\pi$ ($k$ is an integer). Solving for $x$, we get $x = \dfrac{k\pi}
{1.047} = 3k$. \\
\\
Therefore, the nodes occur at $$x = 0,\ 3,\ 6,\ 9,\ 12,\ 15,\ 18,\ 21,\ 24$$

The position of any point $x$ at time $t$ travelling in a wave is modeled by the
function $$y(x,t) = A \sin (kx - kvt)$$ where $v$ is the velocity of the wave, $A$
is its amplitude and $\dfrac{2\pi}{k}$ is the period.\\
\\
\\
It is given that the wave is moving to the right with velocity $6$, then $$v = 6$$
From the figure, we see that the amplitude and period of wave are $2.7$ and $9.2$
respectively. Therefore, $$A = 2.7\ \ \text{and}\ \ \ \dfrac{2\pi}{k} = 9.2$$
So, we have $$k = \dfrac{2\pi}{9.2} = 0.683$$\\
Therefore, the equation of the given travelling wave is \begin{align*}
y(x,t) & = A \sin (kx - kvt)\\
& = 2.7\sin (0.683x - 0.683(6)t)\\
& = 2.7\sin (0.683x - 4.098t)\\
\end{align*}

The position of any point $x$ at time $t$ travelling in a wave is modeled by the
function $$y(x,t) = A \sin (kx - kvt)$$ where $v$ is the velocity of the wave, $A$
is its amplitude and $\dfrac{2\pi}{k}$ is the period.\\
\\
\\
(a)\ It is given that a travelling wave has period $2\pi/3$, amplitude $5$, and
velocity $0.5$. So, we have $$A = 5,\ v = 0.5\ \ \text{and}\ \ \ \dfrac{2\pi}{k}
= \dfrac{2\pi}{3}\ \Rightarrow k = 3$$
Therefore, the equation of the given travelling wave is \begin{align*}
y(x,t) & = A \sin (kx - kvt)\\
& = 5\sin (3x - 3(0.5)t)\\
& = 5\sin (3x - 1.5t)\\
\end{align*}

Given that a standing wave with amplitude $0.6$ graphed at several times has an
equation of the form $$y(x,t) = A \sin \alpha x \cos \beta t$$ that models this
wave.\\
\\
Now, from the first figure, we see that the position of the wave at $t = 0$ s and
$x = 0.5$ is $0.6$. Substituting these values in the equation, we get
\begin{align*}
y(x,t) & = A \sin \alpha x \cos \beta t\\
y(0.5,0) & = 0.6 \sin (\alpha \cdot 0.5) \cos (\beta \cdot 0)\\
0.6 & = 0.6 \sin \dfrac{\alpha}{2} (1)\\
\sin \dfrac{\alpha}{2} & = 1\\
\dfrac{\alpha}{2} & = \sin^{-1} (1)\\
\dfrac{\alpha}{2} & = \dfrac{\pi}{2}\\
\text{Thus}\ \alpha & = \pi
\end{align*}\\
Now, from the third figure, we see that the position of the wave at $t = 0.025$ s
and $x = 0.5$ is $-0.6$. Substituting these values in the equation, we get
\begin{align*}
y(x,t) & = A \sin \alpha x \cos \beta t\\
y(0.5,0.025) & = 0.6 \sin (\alpha \cdot 0.5) \cos (\beta \cdot 0.025)\\
-0.6 & = 0.6 \sin \dfrac{\pi}{2} \cos \dfrac{\beta}{40} \ \ \ \ \ \ \ \ \ \ \ [\
\text{Since}\ \alpha = \pi\ ]\\
-1& = 1 \cos \dfrac{\beta}{40}\\
\cos \dfrac{\beta}{40} & = -1\\
\dfrac{\beta}{40} & = \pi\\
\text{Thus}\ \beta & = 40\pi
\end{align*}\\
Therefore, the equation that models the given wave is $$y(x,t) = 0.6\sin (\pi
x)\cos (40\pi t)$$

Given that a standing wave has maximum amplitude $A = 7$ and nodes at $0,\
\pi/2,\ \pi,\ 3\pi/2,\ 2\pi$. It is also given that each point that is not a node
moves up and down with period $4\pi$. \\
\\
Now, the function that models the given wave is of the form $$y(x,t) = A \sin
\alpha x \cos \beta t$$\\
Now, from the above given information, we have $$\dfrac{2\pi}{\alpha} = \pi\
\Rightarrow \alpha = 2$$
And $$\dfrac{2\pi}{\beta} = 4\pi\ \Rightarrow \beta = 0.5$$\\
Therefore, the function that models the given wave is $$y(x,t) = 7 \sin (2x) \cos
(0.5t)$$

Given that the length of the string is $\pi$. The possible standing waves are shown
in the figure below.\\
\\
Given that for a fixed $t$, the string has the shape of a sine curve $$y = A
\sin \alpha x$$\\
Now, for figure A, the period is $2\pi$. So, $$\dfrac{2\pi}{\alpha} = 2\pi\
\Rightarrow \boxed{\alpha = 1}$$
Now, for figure B, the period is $\pi$. So, $$\dfrac{2\pi}{\alpha} = \pi\
\Rightarrow \boxed{\alpha = 2}$$
Now, for figure C, the period is $2\pi/3$. So, $$\dfrac{2\pi}{\alpha} =
\dfrac{2\pi}{3}\ \Rightarrow \boxed{\alpha = 3}$$
Now, for figure D, the period is $\pi/2$. So, $$\dfrac{2\pi}{\alpha} = \dfrac{\pi}
{2}\ \Rightarrow \boxed{\alpha = 4}$$
Yes, we see a pattern in the values of $\alpha$. We see that the value of $\alpha$
is equal to the number of segments of ropes in each figure.\\
\\
The next two values of $\alpha$ would be $5$ and $6$. The graphs of standing waves
associated with these values of $\alpha$ are shown in the figure below.

If each point on the string that is not a node vibrates with a frequency of $440$
Hz and if the equation that models this wave is of the form $$y = A \cos \beta t$$
then, we have $$\text{frequency} = \dfrac{1}{\text{period}}$$ $$\dfrac{\beta}{2\pi}
= 440$$ Therefore, $$ \boxed{\beta = 880\pi} $$

If $A = 1$, then using the information from (a) and (c), the function that models
the standing waves in the first figure is $$y(x,t) = \sin x \cos (880\pi t)$$
The function that models the standing waves in the second figure is $$y(x,t) = \sin
2x \cos (880\pi t)$$
The function that models the standing waves in the third figure is $$y(x,t) = \sin
3x \cos (880\pi t)$$
The function that models the standing waves in the fourth figure is $$y(x,t) = \sin
4x \cos (880\pi t)$$

Given that a standing wave in a tube $37.7$ ft long is modeled by the function $
$y(x,t) = 0.3 \cos \frac{1}{2}x \cos 50\pi t$$ where $y(x,t)$ represents the
variation from normal air pressure at the point $x$ feet from the end of the tube,
at time $t$ seconds. \\
\\
\\
(a)\ We know that nodes are the points where $y(x,t) = 0$. So, $$y(x,t) = 0.3
\cos \frac{1}{2}x \cos 50\pi t = 0$$ when $\cos \frac{1}{2} x = 0$, i.e. $
$\dfrac{x}{2} = \dfrac{\pi}{2},\ \dfrac{3\pi}{2},\ \dfrac{5\pi}{2},\ \dfrac{7\pi}
{2}\ .\ .\ .\ .\ .\ $$
So, the points $x$ where the nodes are located are $$x = \pi,\ 3\pi,\ 5\pi,\ 7\pi,\
.\ .\ .\ .\ .\ $$ In other words, $$x = (2n-1)\pi, \ \ \ \ \ n \geq 1$$ where $n$
is an integer.\\
\\
\\
To check if the end-points are nodes, we substitute $x = 0$ and $x = 37.7$ in $\cos
\frac{1}{2}x$. For $x = 0$, we have $$\cos \dfrac{1}{2}(0) = \cos 0 = 1$$ and for
$x = 37.7$, we have $$\cos \dfrac{1}{2}(37.7) = \cos 18.85 = 0.999$$
Since $\cos \frac{1}{2}x \ne 0$ at the end-points, the end-points of the tube are
not nodes.\\

You might also like